ACCSAP 10 Arrhythmia

You might also like

Download as pdf or txt
Download as pdf or txt
You are on page 1of 129

1

Arrhythmias

Question 1
1.A 67-year-old man with coronary artery disease, hypertension, and sleep apnea presented to your
office wishing to discuss rhythm control. Two years ago, he experienced his first episode of atrial
fibrillation, for which he was cardioverted and placed on metoprolol and apixaban. Two weeks ago,
he noted the recurrence of an irregular heart rate and had an electrocardiogram (Figure 1). Since
that time, he has felt an irregular heart rate consistently and his fitness tracker device confirms this.
How would you characterize this patient's current arrhythmia?

(Figure 1)

B. Atrial flutter.
C. Persistent atrial fibrillation.
D. Paroxysmal atrial fibrillation.
E. Permanent atrial fibrillation.

This patient has atrial fibrillation. For his initial episode, this would be characterized as paroxysmal
atrial fibrillation. With the current episode lasting consistently for >7 days, this would be
characterized as persistent atrial fibrillation. Longstanding persistent atrial fibrillation is defined by
atrial fibrillation for >12 months in duration. Permanent atrial fibrillation is defined by cessation of
attempts to control rhythm and acceptance of atrial fibrillation as a permanent state.
KKUH
Collected by:
Dr Hani Abdullah
Dr Fatma Hadi
Dr Salem Boresa
Dr Rami Elesali
Dr Naeif Almagal
1/1/2020
2

1. The correct answer: C

Key Point
AF is currently classified as paroxysmal, persistent, long-standing persistent, and permanent.
Permanent is a circumstance where further attempts at restoration of sinus rhythm are abandoned.

Question 2
2.A 74-year-old female with hypertension presented to the clinic for evaluation of new symptoms of
palpitations. She described this sensation as a feeling of her heart skipping a beat and occasionally
racing. She did not have chest pain or discomfort, but did have dyspnea with more prolonged
episodes. Episodes are not triggered by activity or exercise and most often occur after dinner. She
has not had syncope. Episodes occur several times a month and last for a few minutes to several
hours. Her blood pressure is managed with low-dose atenolol. She has a family history of stroke.
Her electrocardiogram was obtained (Figure 1).

Which of the following is the best next step in her management?

A. Stress test.
B. Reassurance.
C. Holter monitor.
D. Increase atenolol.
E. Event monitor.

KKUH
Collected by:
Dr Hani Abdullah
Dr Fatma Hadi
Dr Salem Boresa
Dr Rami Elesali
Dr Naeif Almagal
1/1/2020
3

When evaluating palpitations, the history is fundamental because it enables assessment of


both the differential diagnosis of symptoms and the possibility of structural heart disease that
put the patient at arrhythmic risk. Palpitations can be of cardiac origin (supraventricular or
ventricular ectopy or arrhythmia, valvular heart disease or cardiomyopathy), psychosomatic
(anxiety or depression), neurologic (autonomic nervous system
dysfunction/neurocardiogenic), druginduced, or due to other pathology (thyroid disorder or
anemia). Evaluation of symptomatology requires assessment of the impact of symptoms on
the patient's life and the potential of risk due to structural heart disease or arrhythmia. In this
case, because the patient has risk factors for heart disease, correlation of symptoms with a
potential arrhythmia is critical, and reassurance without further evaluation would not be
optimal. Increasing atenolol would be difficult due to baseline bradycardia. When deciding
between using a 24-hour Holter and an event monitor to correlate symptoms with a potential
arrhythmia, the frequency of symptoms is important. Overall, ambulatory event monitoring of
a duration of 2-4 weeks has been shown to provide a higher yield of diagnosis than Holter.

2. The correct answer: E

Key Point
When evaluating patients with lesser degrees of bradycardia, symptom-rhythm correlation is
important. This can be achieved with ambulatory monitoring (Holter or similar). More
prolonged monitoring with an implantable subcutaneous monitor may be needed in some
cases.

Question 3

3.A 78-year-old male with a history of coronary artery bypass presented to the emergency
department with symptoms of dizziness and palpitations. He has a history of tobacco use,
diabetes mellitus, and hypertension. His home medications include metoprolol succinate,
lisinopril, aspirin, and atorvastatin. Upon arrival at the emergency department, the patient's
blood pressure was 80/45 mm Hg with a heart rate of 180 bpm. He appeared mildly
distressed. An electrocardiogram (ECG) was obtained (Figure 1).

What is the most likely cause of this patient's arrhythmia?

KKUH
Collected by:
Dr Hani Abdullah
Dr Fatma Hadi
Dr Salem Boresa
Dr Rami Elesali
Dr Naeif Almagal
1/1/2020
4

(Figure 1)

A. Abnormal automaticity,
B. Triggered activity.
C. Scar-mediated re-entry.
D. Early afterdepolarizations.
E. Delayed afterdepolarizations.

This ECG shows monomorphic ventricular tachycardia (VT). Monomorphic VT is most commonly
due to a re-entry mechanism, typically from myocardial scar. Triggered activity encompasses both
delayed and early afterdepolarization and is considered the likely mechanism for right ventricular
outflow tract (RVOT) VT. Early afterdepolarizations are the trigger for torsades de pointes in the
setting of QT prolongation. Delayed afterdepolarizations are associated with catecholaminergic
polymorphic VT. Ischemia is commonly associated with enhanced normal automaticity and abnormal
automaticity.

3. The correct answer: C

Key Point
Monomorphic ventricular tachycardia (MVT) is most commonly due to a re-entry mechanism around
an infarcted scar tissue. Less commonly, MVT is due to a single focus in patients without structural
heart disease.

KKUH
Collected by:
Dr Hani Abdullah
Dr Fatma Hadi
Dr Salem Boresa
Dr Rami Elesali
Dr Naeif Almagal
1/1/2020
5

Question 4
4.A 30-year-old male presented to the cardiology clinic for evaluation after his father had an episode
of syncope and was found to have a Brugada pattern on his electrocardiogram (ECG). His father's
genetic testing revealed an SCN5A gene mutation. You obtained an ECG (Figure 1).

What is the best next step in his care?

(Figure 1)

A. No further evaluation.
B. Echocardiogram.
C. Implantable cardioverter defibrillator.
D. Exercise stress ECG.
E. Genetic testing.
This patient's ECG is normal. Nevertheless, first- and second-degree relatives of patients with
Brugada syndrome and positive genetic screen are recommended to undergo genetic testing even in
the setting of a normal baseline ECG. An echocardiogram may not demonstrate structural heart
disease in a patient with isolated Brugada syndrome. Implantable cardioverter-defibrillator (ICD)
implantation is not indicated for asymptomatic Brugada syndrome patients with a family history of
sudden cardiac death alone. Although an exercise stress test may induce changes in ST segments
and can be supportive of a diagnosis of Brugada syndrome, as this patient's father (the proband)
already has an identified causative mutation, proceeding with genetic testing of the patient would be
the best next step. A negative test would effectively exclude the diagnosis of Brugada syndrome in
him.

4. The correct answer: E


Key Point
Clinical and genetic testing of probands is recommended for family screening along with clinical and
genetic testing of all first- and second-degree relatives, even if phenotype negative, to assist with
counseling.

KKUH
Collected by:
Dr Hani Abdullah
Dr Fatma Hadi
Dr Salem Boresa
Dr Rami Elesali
Dr Naeif Almagal
1/1/2020
6

Question 5
5.A 38-year-old male presented for evaluation of syncope. He has a history of palpitations
associated with lightheadedness. He takes no home medications. His father died suddenly at age
35. An electrocardiogram (ECG) (Figure 1) and echocardiogram (Video 1) were obtained.

In this patient, which of the following measurements is most predicative of a positive genetic screen
for his condition?

(Figure 1)
Video 1

A. Right ventricular end-diastolic dimension.


B. QTc.
KKUH
Collected by:
Dr Hani Abdullah
Dr Fatma Hadi
Dr Salem Boresa
Dr Rami Elesali
Dr Naeif Almagal
1/1/2020
7

C. R-R interval.
D. Left ventricular end-diastolic dimension.
E. Septal wall thickness.

This patient has long QT syndrome (LQTS) based on his ECG and history. The probability of a
positive genetic test for LQTS is greatest when the QTc is >480 msec, especially in the setting of a
positive family history. This diagnosis is not related to the underlying heart rate. The transthoracic
echocardiogram is normal and LQTS is not associated with cardiac structural abnormalities.

5. The correct answer: B

Key Point
The success of genetic testing in LQTS depends in large part on the robustness of the clinical
diagnosis with mutations identified in 70-80% of cases when the diagnosis is certain (QTc >480
msec), but dropping significantly in borderline cases.

Question 6
6.A 60-year-old man was referred by his primary care clinician for evaluation of bradycardia. He is an
avid marathon runner who exercises regularly without limitations. The patient's past medical history
is notable for hypertension, which has been well controlled with lisinopril.

On examination, his heart rate is 46 bpm and blood pressure 112/78 mm Hg. Cardiac auscultation
reveals a bradycardic, regular rhythm. Lungs are clear to auscultation, and he has no lower
extremity edema.

An electrocardiogram was recorded (Figure 1).


Which of the following is the most appropriate next test?

KKUH
Collected by:
Dr Hani Abdullah
Dr Fatma Hadi
Dr Salem Boresa
Dr Rami Elesali
Dr Naeif Almagal
1/1/2020
8

(Figure 1)

A. Treadmill stress test.


B. Pharmacologic nuclear stress test.
C. Electrophysiology study.
D. Borrelia burgdorferi antibody.
E. Transthoracic echocardiogram.

Atrioventricular (AV) block may result from increased parasympathetic nervous system output or
disease of the conduction system. Enhanced vagal tone due to athletic training, sleep, pain, or other
causes may result in slowing of the sinus rate as well as AV conduction abnormalities. More
commonly, this results in first-degree AV block or Mobitz type I second-degree AV block. Higher-
degree AV block, Mobitz type II second-degree AV block, or third degree heart block are suggestive
of underlying infranodal pathology of the conduction system, i.e., below the level of the AV node.

This patient has 2:1 atrioventricular block, which may be due to either type I or type II second-
degree AV block. Given that this patient is able to participate in regular exercise, it is likely that the
AV block is a consequence of high vagal tone rather than due to disease of the conduction
system. An exercise treadmill stress test may help differentiate whether 2:1 atrioventricular block is
due to Mobitz type I or II AV block. Exercise causes withdrawal of vagal tone and increased
sympathetic tone leading to improved atrioventricular nodal conduction. If the baseline
atrioventricular block is infranodal (Mobitz type II), the atrioventricular block will not resolve and will
likely worsen as the sinus rate increases. If the latter ensues, a pacemaker would be indicated. If
the atrioventricular block is Mobitz type I, then increased sympathetic tone would facilitate
conduction through the AV node and may lead to resolution of the AV block.

KKUH
Collected by:
Dr Hani Abdullah
Dr Fatma Hadi
Dr Salem Boresa
Dr Rami Elesali
Dr Naeif Almagal
1/1/2020
9

An echocardiogram would be unlikely to elucidate the cause of his conduction abnormality.

Testing for B. burgdorferi is not indicated without other signs or symptoms of Lyme disease such as
antecedent rash, flu-like symptoms, and joint pains. Later signs of B. burgdorferi infection can
include carditis with AV block.

An electrophysiology study is not indicated without noninvasive evidence of higher-degree AV block.

A pharmacologic nuclear stress test would not be expected to yield the diagnosis, as the patient
does not have any other symptoms of ischemic heart disease.
6. The correct answer: A

Key Point
In patients with block at the AV node level, only block will improve with exercise, but with infra-Hisian
block (i.e., below the AV node), block will get worse with exercise. An exercise test should be
recommended if there is a question regarding whether the bradycardia is intrinsic (heart rate will not
improve or may even worsen with exercise). In contrast, in patients in whom bradycardia is due to
high vagal tone (athletes, for example), there will be a normal and appropriate increase in heart rate
with exercise.

Question 7
A 71-year-old woman with an ischemic cardiomyopathy and permanent atrial fibrillation (AF)
underwent biventricular implantable cardioverter-defibrillator (ICD) implantation 6 months
prior. She has New York Heart Association class III symptoms and continues to complain of
very limited functional capacity, with no change since resynchronization therapy was
initiated.

She is currently taking carvedilol 25 mg BID, lisinopril 10 mg QD, furosemide 40 mg BID,


atorvastatin 40 mg QD, aspirin 81 mg QD, and warfarin 5 mg QD. Her vital signs in the office
were blood pressure 90/60 mm Hg, heart rate 111 bpm, respiration rate 20 breaths per
minute, and oxygen saturation 94% room air. She was afebrile. Her examination was
significant for bibasilar rales, irregular heartbeat, and 1+ bilateral lower extremity edema.

Interrogation of her biventricular ICD revealed good pacing and sensing thresholds. The
battery status and lead impedances were adequate. There had been two episodes of
nonsustained ventricular tachycardia, up to 15 beats, but no shocks had been required. She
was 25% paced over the past 3 months.

KKUH
Collected by:
Dr Hani Abdullah
Dr Fatma Hadi
Dr Salem Boresa
Dr Rami Elesali
Dr Naeif Almagal
1/1/2020
10

In addition to treatment of her heart failure, which of the following should be the next step in her
management?
A. Add sotalol.
B. Referral for heart transplant evaluation.
C. Atrioventricular nodal ablation.
D. Increase carvedilol dose.

The benefit of cardiac resynchronization therapy is highest with maximal biventricular pacing. This
patient is paced only 25% of the time due to AF with rapid ventricular response. Restoration of sinus
rhythm is unlikely to be successful in a patient with permanent AF. Antiarrhythmic drugs for rhythm
control, such as sotalol, should not be continued when AF becomes permanent. Her blood pressure
is tenuous, so increasing her beta-blocker is not recommended. Transplant evaluation prior to
utilizing available guideline-directed therapy is not indicated.

7.The correct answer: C

Key Point
Catheter ablation of the AV junction for nonpharmacologic rate control is highly effective, but renders
the patient pacemaker dependent and is irreversible. It is most suitable for elderly patients with
associated bradycardia or intolerance/inefficacy of rate control agents.

Question 8
8.A 40-year-old female was referred for evaluation of palpitations. She has felt skipped and irregular
beats for the past 20 years, but in the past 6 months, she has found that her exercise tolerance has
decreased due to these symptoms. She denied any chest pain or dyspnea. Prior evaluation included
a Holter monitor, which demonstrated frequent runs of nonsustained ventricular tachycardia (NSVT).
An echocardiogram was technically limited due to body habitus and poor acoustic windows.

Her family history is notable for an uncle who died of a "rhythm problem" that occurred during a
hospitalization 20 years ago.

On physical examination, her blood pressure was 123/70 mm Hg, pulse was 60 bpm, and weight
was 170 pounds (body mass index 30). The jugular venous pulse was at 6 cm, with occasional
cannon A waves present. Lung sounds were clear. Auscultation showed a single S1, single S2, and
frequent ectopic beats.

KKUH
Collected by:
Dr Hani Abdullah
Dr Fatma Hadi
Dr Salem Boresa
Dr Rami Elesali
Dr Naeif Almagal
1/1/2020
11

The remainder of the examination was normal. Her 12-lead electrocardiogram (ECG) rhythm strip is
shown in Figure 1.

Which of the following is the best next step in management?

A. Cardiac catheterization.
B. Cardiac magnetic resonance imaging.
C. Implantable loop recorder.
D. Stress nuclear perfusion study.
E. Thirty-day event monitoring.

This ECG shows premature ventricular contractions (PVCs) of multiple morphologies. PVCs show a
right bundle morphology (namely positive in V1), but with different amplitudes in the late precordial
leads. The PVCs are likely arising from the left ventricle, but from different locations. In a woman
with no risk factors for coronary disease and a longstanding history of symptoms likely related to
these PVCs and NSVT, one should first consider a cardiomyopathy. As an echocardiogram was
previously regarded as suboptimal, the best next imaging study is a cardiac magnetic resonance
image (MRI). This patient’s cardiac MRI revealed left ventricular noncompaction. An event monitor or
implantable loop recorder is not helpful in this case, as the frequent PVCs are already evident on her
office evaluation.

KKUH
Collected by:
Dr Hani Abdullah
Dr Fatma Hadi
Dr Salem Boresa
Dr Rami Elesali
Dr Naeif Almagal
1/1/2020
12

8. The correct answer: B

Key Point
Distinction of ventricular tachycardia in patients with or without structural heart disease is
importantsince therapeutic approaches are very different. In patients with structural heart disease,
an implantable cardioverter-defibrillator is generally recommended (secondary prevention); in
patients without structural heart disease, drug therapy or catheter ablation may be effective in
reducing or eliminating the arrhythmia.

Question 9
9.A 60-year-old male came to the emergency department with palpitations. He described a racing
heart rate and shortness of breath. His past medical history includes a myocardial infarction (MI),
dyslipidemia, hypertension, and gastroesophageal reflux disease. His current medications are
aspirin 81 mg, metoprolol succinate 25 mg, chlorthalidone 50 mg, atorvastatin 80 mg, clopidogrel 75
mg, and omeprazole 20 mg. His pulse was 150 bpm and thready. His blood pressure was 90/60 mm
Hg. On examination, he was diaphoretic and uncomfortable. An electrocardiogram was obtained
(Figure 1).

Which of the following best describes the mechanism of his arrhythmia?

(Figure 1)

KKUH
Collected by:
Dr Hani Abdullah
Dr Fatma Hadi
Dr Salem Boresa
Dr Rami Elesali
Dr Naeif Almagal
1/1/2020
13

A. Re-entry.
B. Early afterdepolarizations.
C. Enhanced automaticity.
D. Delayed afterdepolarizations.
E. Decremental conduction.

Mechanisms of ventricular arrhythmia (VA) include enhanced normal automaticity, abnormal


automaticity, triggered activity induced by early or late afterdepolarizations, and re-entry. Re-entry
requires a trigger to initiate the arrhythmia and a substrate to sustain it. The trigger may be a
premature ventricular complex (PVC), which may be due to automaticity. The substrate may be
structural remodeling secondary to an underlying disease process, and often includes a scar
secondary to a prior MI or surgical repair, or patchy fibrosis in the setting of cardiomyopathy or
hypertrophy. Changes in ion channel or transporter function and/or expression and cell-to-cell
coupling secondary to the underlying pathology may alter the initiation or propagation of the cardiac
action potential. The electrophysiological substrate is dynamically influenced by a variety of factors
including cardiac metabolism, electrolytes, signaling pathways, and autonomic effects.

Re-entry around a scar is the mechanism of monomorphic VAs in patients with ischemic heart
disease.

Decremental conduction is a normal behavior of the atrioventricular node to protect the ventricle
from high atrial rates.

Both early and delayed afterdepolarizations are types of triggered activity and are not expected to be
related to scar-mediated VAs, as seen with monomorphic ventricular tachycardia.

Enhanced automaticity is usually caused by electrolyte abnormalities or ischemia; although it may be


a factor in initiating monomorphic ventricular tachycardia, it is not responsible for sustaining the
arrhythmia.

9. The correct answer: A

Key Point
Monomorphic ventricular tachycardia (MVT) is most commonly due to a re-entry mechanism around
an infarcted scar tissue. Less commonly, MVT is due to a single focus in patients without structural
heart disease.

Question 10
10.A 38-year-old woman presented to the cardiology clinic for evaluation of tachycardia. She has no
prior medical history and is on no medications. She noted intermittent palpitations over the past few
months, usually while watching television. She was referred for an ambulatory electrocardiogram
(ECG) monitor by her primary care clinician, which showed an episode of wide complex tachycardia
at 178 bpm with an inferior QRS axis and left bundle branch block appearance. A transthoracic
echocardiogram showed normal biventricular size and systolic function.

KKUH
Collected by:
Dr Hani Abdullah
Dr Fatma Hadi
Dr Salem Boresa
Dr Rami Elesali
Dr Naeif Almagal
1/1/2020
14

Her resting ECG was obtained (Figure 1).

What is the best next step?

(Figure 1)

A. Implantable cardioverter-defibrillator.
B. Exercise treadmill test.
C. Cardiac magnetic resonance imaging.
D. Catheter ablation.
E. Coronary computed tomography angiogram.

The wide complex tachycardia with a left bundle appearance and inferior axis described above is
suggestive of right ventricular outflow tract (RVOT) ventricular tachycardia (VT). RVOT VT can be
seen in structurally normal hearts and has a more benign course that can respond well to catheter
ablation. However, the patient's baseline ECG shows anterior T-wave inversions, which are
concerning for arrhythmogenic right ventricular cardiomyopathy (ARVC), and therefore she needs
further testing for ARVC.

In patients with suspected ARVC, there is a class I recommendation for cardiac magnetic resonance
imaging (MRI) to establish the diagnosis and for risk stratification. ARVC is characterized by
progressive ventricular myocyte loss with replacement by fatty or fibrous tissue, and is associated
with progressive ventricular dysfunction that may involve both ventricles. Ventricular arrhythmia,
syncope, and sudden cardiac death may occur at a relatively young age, particularly in the second
and third decades of life and often during physical activity. ARVC is often due to a mutation involving
a desmosomal protein, and it usually has autosomal dominant inheritance with variable penetrance.

Catheter ablation can be pursued for patients with ARVC and recurrent VT that is refractory to beta-
blockers. This patient is on no medications, and a beta-blocker should be started prior to ablation.

KKUH
Collected by:
Dr Hani Abdullah
Dr Fatma Hadi
Dr Salem Boresa
Dr Rami Elesali
Dr Naeif Almagal
1/1/2020
15

This is in contrast to RVOT VT, for which catheter ablation is an acceptable first-line therapy.
However, because the diagnosis is not yet established, this is not the correct answer.

Therefore, a coronary computed tomography (CT) angiogram or stress test is not indicated.

Implantable cardioverter-defibrillators (ICDs) may be indicated in certain patients with ARVC, but the
diagnosis has not been established yet.

10. The correct answer: C

Key Point
The important differential diagnosis in right ventricular outflow tract ventricular tachycardia (VT) is VT
occurring in the situation of arrhythmogenic right ventricular cardiomyopathy (ARVC). One important
difference is that the resting 12-lead echocardiogram (ECG) shows marked T-wave inversion in the
right precordial leads (V1-V3/4) and there may be evidence of an epsilon wave. The 12-lead ECG in
patients with normal heart VT is normal. If ARVC is suspected, computed tomography or magnetic
resonance imaging should be performed to rule out this condition, which is a genetic abnormality.

Question 11
11.An 83-year-old male was admitted to the hospital with confusion and near-syncope. He has been
profoundly fatigued for several weeks and feels as though he is going to pass out whenever he tries
to walk any significant distance. His past medical history includes a remote (>5 years ago)
myocardial infarction, hypertension, and osteoarthritis. His current medications are chlorthalidone 25
mg daily, aspirin 81 mg daily, atorvastatin 80 mg daily, and diclofenac as needed. His vital signs
were pulse 62 bpm, blood pressure 126/78 mm Hg, and respiration 12 breaths per minute. A
physical examination revealed prominent A waves, soft s1 and normally split s2, and a soft (II/VI)
holosystolic murmur at the apex. There was 1+ bilateral ankle edema. His electrocardiogram (ECG)
was recorded (Figure 1). A treadmill stress test was performed; he exercised 6 minutes of modified
Bruce protocol, reaching a peak heart rate of 112 bpm and maximum blood pressure of 180/80 mm
Hg, then stopped due to lightheadedness and fatigue.

Which of the following most likely explains his symptoms?

KKUH
Collected by:
Dr Hani Abdullah
Dr Fatma Hadi
Dr Salem Boresa
Dr Rami Elesali
Dr Naeif Almagal
1/1/2020
16

(Figure 1)

A. Complete heart block.


B. Chronotropic incompetence.
C. Cardio-inhibitory syncope.
D. First-degree atrioventricular block.
E. Autonomic dysfunction.

This patient has symptoms from lack of atrioventricular (AV) synchrony, attributable to a markedly
prolonged P-R interval ("marked first-degree AV block"). The diagnosis is made from the surface
ECG.

Although there is little evidence to suggest that pacemakers improve survival in patients with isolated
first-degree AV block, it is now recognized that marked (P-R more than 300 msec) first-degree AV
block can lead to symptoms even in the absence of higher degrees of AV block. When marked first-
degree AV block for any reason causes atrial systole in close proximity to the preceding ventricular
systole and produces hemodynamic consequences usually associated with retrograde
(ventriculoatrial) conduction, including cannon A waves, signs and symptoms similar to the
pacemaker syndrome may occur. With marked first-degree AV block, atrial contraction occurs before
complete atrial filling, ventricular filling is compromised, and an increase in pulmonary capillary
wedge pressure and a decrease in cardiac output follow. Small uncontrolled trials have suggested
some symptomatic and functional improvement by pacing of patients with P-R intervals more than
0.30 sec by decreasing the time for AV conduction.

He has normal heart rate and blood pressure responses to exercise, ruling out autonomic
dysfunction and chronotropic incompetence. The ECG does not show complete heart block. His
symptoms are not classic for cardio-inhibitory syncope, as they are exertional rather than vagally
triggered.
KKUH
Collected by:
Dr Hani Abdullah
Dr Fatma Hadi
Dr Salem Boresa
Dr Rami Elesali
Dr Naeif Almagal
1/1/2020
17

11. The correct answer: D

Key Point
The diagnosis and treatment of SND and AV block includes a thorough history and symptom
evaluation, as well as examination of the electrocardiographic evidence correlating these symptoms
with electrical abnormalities.

Question 12
12.A 34-year-old male presented to the emergency department with nausea and lightheadedness
over the prior hour. He has no past medical history and takes no medications. His family history is
not known because he was adopted from Brazil as a young child.

On examination, his heart rate was 200 bpm and blood pressure was 80/60 mm Hg. He was pale
and fatigued. His lungs were clear and heart was tachycardic with no murmurs appreciated. His
abdomen was soft and extremities were without edema.

An electrocardiogram (ECG) was obtained (Figure 1). He underwent cardioversion and a


subsequent ECG was obtained (Figure 2).

Which of the following is the best next step?

(Figure 1)

KKUH
Collected by:
Dr Hani Abdullah
Dr Fatma Hadi
Dr Salem Boresa
Dr Rami Elesali
Dr Naeif Almagal
1/1/2020
18

(Figure 2)

A. Cardiac magnetic resonance imaging.


B. Electrophysiology study.
C. Fluorodeoxyglucose-positron emission tomography.
D. Trypanosoma cruzi antibody.
E. Treadmill stress testing.

This patient presented with sustained monomorphic ventricular tachycardia (VT) with a left bundle
branch block (LBBB) morphology and inferior axis, suggesting that it originates in the right
ventricular outflow tract (RVOT).

The differential diagnosis of VT arising from the RVOT includes idiopathic VT and arrhythmogenic
right ventricular cardiomyopathy (ARVC). Although both conditions can present with sustained VT,
the treatments and prognosis are significantly different and thus it is essential to differentiate the two
conditions.

The resting ECG of a patient with ARVC will often show an epsilon wave after the QRS and T-wave
inversions in V1-V3, as seen in the postcardioversion ECG (Figure 2). Thus, ARVC is the correct
diagnosis and can be confirmed with cardiac magnetic resonance imaging (MRI).

The resting ECG in idiopathic VT originating in the RVOT is most often normal. Electrophysiologic
studies would demonstrate the origin of the VT but would not reliably differentiate benign RVOT VT
or premature ventricular complexes (PVCs) from ARVC.

The patient has a normal QT interval after cardioversion. Although this does not exclude long QT
syndrome, where the ECG may not be abnormal unless provocative maneuvers are used, it is not
the best answer since the ECG has the classic findings of ARVC. In addition, long QT syndrome

KKUH
Collected by:
Dr Hani Abdullah
Dr Fatma Hadi
Dr Salem Boresa
Dr Rami Elesali
Dr Naeif Almagal
1/1/2020
19

usually presents with polymorphic VT, not monomorphic VT, making this diagnosis unlikely. Thus,
stress testing to assess for long QT is not indicated.

Chagas disease can cause a cardiomyopathy as well as arrhythmias. Virtually all types of atrial and
ventricular arrhythmias occur, and ventricular arrhythmias and atrioventricular (AV) block frequently
occur concurrently. The severity of ventricular arrhythmias tends to correlate with the degree of left
ventricle (LV) dysfunction. However, this patient has no other evidence of heart failure consistent
with a Chagas cardiomyopathy and this condition would not cause the ECG changes seen; thus,
Trypanosoma cruzi (T. cruzi) antibody testing is not the best answer.

Cardiac sarcoidosis (CS) may present with ventricular arrhythmias, sustained or nonsustained
ventricular tachycardia, and ventricular premature beats (VPBs), which are the second most
common clinical presentation of CS, occurring in approximately 30% of cases. Although isolated CS
can occur in 25% of cases, the baseline ECG changes cannot be explained by CS and thus
fluorodeoxyglucose-positron emission tomography (FDG-PET) is not indicated.

12. The correct answer: A

Key Point
The important differential diagnosis in right ventricular outflow tract ventricular tachycardia (VT) is VT
occurring in the situation of arrhythmogenic right ventricular cardiomyopathy (ARVC). One important
difference is that the resting 12-lead echocardiogram (ECG) shows marked T-wave inversion in the
right precordial leads (V1-V3/4) and there may be evidence of an epsilon wave. The 12-lead ECG in
patients with normal heart VT is normal. If ARVC is suspected, computed tomography or magnetic
resonance imaging should be performed to rule out this condition, which is a genetic abnormality.

Question 13
13.An 18-year-old male presented to the clinic referred by his primary care clinician. He denied any
past cardiac issues and has no history of syncope. His electrocardiogram (ECG) showed a pattern of
pseudo right bundle branch block with ST elevation and T-wave inversion in leads V1 and V2. You
interpret this ECG as a type I pattern of Brugada syndrome. His oldest brother died suddenly when
he was 25 years old, and no ECGs of his brother are available.

Which of the following is the most appropriate next step?


A. Implant implantable cardioverter-defibrillator.
B. Provocative testing with sodium channel blockade.
C. Continued clinical follow-up.
D. Advise to avoid strenuous activity.
E. Implant loop recorder.

KKUH
Collected by:
Dr Hani Abdullah
Dr Fatma Hadi
Dr Salem Boresa
Dr Rami Elesali
Dr Naeif Almagal
1/1/2020
20

The patient has a type 1 Brugada syndrome ECG pattern (Figure 1). An older brother who died
suddenly does not increase his risk for sudden death, and because the patient is asymptomatic, he
does not meet criteria for implantable cardioverter-defibrillator (ICD) implantation. The utility of
electrophysiologic study for treatment decision remains controversial. Provocative testing with
sodium channel blockers is not indicated in a type 1 ECG pattern. There is no need for this patient to
refrain from physical activity. At this point, the patient needs to be followed and advised to avoid
drugs with sodium channel blocking activity (see www.brugadadrugs.org).

(Figure 1)
Reproduced with permission from Mizusawa Y, Wilde AA. Brugada syndrome. Circ Arrhythm
Electrophysiol 2012;5:606-16.

13. The correct answer: C


Key Point
Use of a sodium channel-blocking agent (flecainide or ajmaline) can “unmask” BrS and can assist in
the diagnosis of borderline cases. Recording leads V1 and V2 in second and third intercostal space
can also assist in the diagnosis.

Question 14
14.A 34-year-old male presented on Wednesday morning to the emergency department after a
weekend of heavy drinking. He felt his heart racing when he woke up on Saturday morning. The
sensation has persisted. On assessment, he was in an irregularly irregular rhythm with otherwise
normal vital signs. He has no history of hypertension, diabetes mellitus, stroke, or vascular disease.
He was anticoagulated with intravenous unfractionated heparin. He underwent tranesophageal
KKUH
Collected by:
Dr Hani Abdullah
Dr Fatma Hadi
Dr Salem Boresa
Dr Rami Elesali
Dr Naeif Almagal
1/1/2020
21

echocardiography followed by direct-current cardioversion. The left atrial appendage was normal in
size and there was no spontaneous echocardiogram contrast seen. The left ventricular ejection
fraction was normal and there was trace mitral regurgitation.

Which of the following is an appropriate anticoagulation strategy upon discharge?


A. Enoxaparin 1 mg/kg for 5 days.
B. Aspirin 81 mg daily for 1 month.
C. No additional therapy.
D. Warfarin with international normalized ratio (INR) goal 2.0-3.0 for 1 month.
E. Apixaban 5 mg BID for 1 month.

This is a patient with acute onset of atrial fibrillation, likely related to alcohol. The duration of atrial
fibrillation is >48 hours, therefore cardioversion with transesophageal echocardiography (TEE)
guidance is recommended (alternatively 3 weeks of anticoagulation prior to cardioversion may be
considered). The patient has nonvalvular atrial fibrillation and normal left ventricular (LV) systolic
function. The CHADS2VASC score is zero. After cardioversion for atrial fibrillation lasting >48 hours,
guidelines recommend oral anticoagulation for at least 4 weeks; thus, apixaban for 1 month is the
best answer. It would not be appropriate to withhold anticoagulation in this patient. Aspirin offers
inferior thromboembolic prophylaxis and is not the correct answer. Low molecular weight heparin for
5 days as a bridge to a therapeutic international normalized ratio (INR) on warfarin could be
combined to provide safe anticoagulation, but either medication alone would be inadequate for
thromboembolic prophylaxis. Long-term anticoagulation use is determined by aggregate stroke risk
using CHADS2 or CHADS2VASC. In this case, with a CHADS2VASC score of 0, long-term
anticoagulation is not indicated.

14.The correct answer: E

Key Point
Because of the risk of cardioembolic stroke associated with cardioversion, anticoagulation is
recommended independent of the CHA2DS2-VASc score in patients in whom AF has lasted >48
hours or if there is uncertainty regarding onset time.

Question 15
15.A 56-year-old woman with a nonischemic cardiomyopathy with ejection fraction 20%, status
postcardiac resynchronization therapy-defibrillator 6 months ago, presented to your office with a 2-

week history of progressive dyspnea on exertion. She denied any medication or dietary
nonadherence.
Her medications include furosemide 40 mg twice daily, carvedilol 6.25 mg twice daily,
sacubitril/valsartan 24/26 mg twice daily, and spironolactone 12.5 mg daily.

KKUH
Collected by:
Dr Hani Abdullah
Dr Fatma Hadi
Dr Salem Boresa
Dr Rami Elesali
Dr Naeif Almagal
1/1/2020
22

On examination, her heart rate was 88 bpm, blood pressure was 98/60 mm Hg, jugular venous pulse
was 12 cm H20, lungs were clear, and heart was regular with a soft S3 gallop. Her extremities were
warm with trace edema.

An electrocardiogram (ECG) was recorded (Figure 1). The chest X-ray showed mild pulmonary
vascular congestion.

What is the best next step?

(Figure 1)
Reproduced with permission from
https://upload.wikimedia.org/wikipedia/commons/f/ff/Duelchamber.JPG. Accessed 05/14/2018.

A. Complete blood count.


B. Ventilation-perfusion scan.
C. N-terminal pro–B-type natriuretic peptide.
D. Device interrogation.
E. Echocardiogram.

This patient has decompensated heart failure despite optimal guideline-directed medical therapy and
recent cardiac resynchronization therapy-defibrillator (CRT-D) placement. There are a number of
typical causes of heart failure decompensation, including medication and dietary nonadherence,
ischemia, thyroid disease, arrhythmias, and infection. However, this patient's ECG shows a left
bundle branch block pattern, consistent with RV only pacing. Thus, the correct answer is device
interrogation to confirm this finding. On device interrogation, one would specifically assess for LV
pacing and capture.

An echocardiogram is not indicated, as her ejection fraction is already severely reduced and physical

KKUH
Collected by:
Dr Hani Abdullah
Dr Fatma Hadi
Dr Salem Boresa
Dr Rami Elesali
Dr Naeif Almagal
1/1/2020
23

examination discloses no new anticipated changes. A ventilation-perfusion scan is not indicated, as


her shortness of breath appears due to decompensated heart failure and there is no suggestion of
predisposing factors for pulmonary embolism. N-terminal pro–B-type natriuretic peptide (NT-
proBNP) will undoubtedly be elevated, as she has decompensated heart failure, and will not aid in
determining the cause of her decompensation. She may be anemic, but this is not the best next
step, as nothing in the history or physical points to this as a likely diagnosis.

15. The correct answer: D


Key Point
"CRT response is dependent on >90% biventricular pacing. Causes of nonresponse to CRT through
diminished pacing include atrial fibrillation with rapid conduction, inappropriate device programming,
and frequent ventricular ectopy.
CRT non-response can also be due to loss of LV lead capture or poor LV lead position.

Question 16
16.You were asked to see a 71-year-old male for preoperative evaluation prior to craniotomy. He
presented with confusion 1 month after falling and striking his head. Due to the confusion, he has not
taken any of his medications in 4 days. Physical examination demonstrated a blood pressure of
124/72 mm Hg, clear lung sounds, and a regular rhythm. An electrocardiogram was obtained (Figure
1). A computed tomography scan demonstrated a chronic subdural hematoma.

Which of the following is the anatomic site of the arrhythmia on his electrocardiogram?

(Figure 1)

KKUH
Collected by:
Dr Hani Abdullah
Dr Fatma Hadi
Dr Salem Boresa
Dr Rami Elesali
Dr Naeif Almagal
1/1/2020
24

A. Cavotricuspid isthmus.
B. Pulmonary vein.
C. Fossa ovalis.
D. Crista terminalis.
E. Bachmann's bundle.

The patient is in rate-controlled atrial flutter. Typical atrial flutter is characterized by a "saw-tooth"
pattern in the inferior (II, III, aVF) leads and a positive flutter wave in lead V1, reflecting the macro-
reentrant circuit, which includes the isthmus between the inferior vena cava and the tricuspid valve
(Figure 2). This distinction is important because cavotricuspid isthmus–dependent flutter can be
ablated with >90% success.

Atypical right-sided atrial flutter may originate from the fossa ovalis or superior vena cava and would
not produce the same pattern of saw-tooth waves in inferior leads and positive flutter waves in
V1 as cavotricuspid isthmus–dependent flutter.

Islands of pacemaker cells in pulmonary veins are responsible for atrial fibrillation.

The crista terminalis can be responsible for atrial arrhythmias by initiating ectopic atrial beats.
Ectopic rhythms originating from the crista terminalis have the same electrocardiogram appearance
as normal sinus P waves due to the location of the crista terminalis near the sinus node.

Bachmann's bundle is the electrical connection between the right and left atria.

(Figure 2)
Legend: AAo = ascending aorta; CS/ThV = coronary sinus and thebesian valve; CT = crista
terminalis; CTI = cavotricuspid isthmus; ER/EV = Eustachian ridge and valve; IVC = inferior vena
cava; OF = oval fossa; RAA = right atrium appendage; RCA = right coronary artery; RV = right
ventricle; SI = septal isthmus; STV = tricuspid valve; SVC = superior vena cava; TT = tendon of
Todaro.
KKUH
Collected by:
Dr Hani Abdullah
Dr Fatma Hadi
Dr Salem Boresa
Dr Rami Elesali
Dr Naeif Almagal
1/1/2020
25

16. The correct answer: A

Key Point
AFL is a macro–re-entrant circuit most commonly within the RA and is sustained through a region of
slow conduction along a channel or “isthmus” between the IVC and the tricuspid valve, the so called
“cavotricuspid isthmus.” Due to the critical dependence of the arrhythmia on conduction through this
region, the rhythm is classified a “cavotricuspid isthmus-dependent” or CTI-dependent AFL.

Question 17
17. You supervised a treadmill stress test for a 36-year-old male with palpitations and a prior resting
electrocardiogram (ECG) (Figure 1). He has no other medical problems and takes no medications;
he has no known drug allergies. Six minutes into the Bruce protocol, he reported palpitations and
another ECG was obtained (Figure 2). After stopping the treadmill and placing him on the stretcher,
the rhythm continued. He was alert and said the palpitations were just like what he feels when he
exercises at the gym. His pulse was 180 bpm and irregular, and his blood pressure was 104/60 mm
Hg. Vagal maneuvers did not change the rhythm.
Which of the following is the best next step?

(Figure 1)

KKUH
Collected by:
Dr Hani Abdullah
Dr Fatma Hadi
Dr Salem Boresa
Dr Rami Elesali
Dr Naeif Almagal
1/1/2020
26

(Figure 2)

A. Metoprolol.
B. Lidocaine.
C. Amiodarone.
D. Magnesium.
E. Procainamide.

This patient with Wolff-Parkinson-White syndrome has pre-excited atrial fibrillation. Prompt treatment
with intravenous procainamide, intravenous ibutilide, or synchronized cardioversion is critical to
prevent deterioration to ventricular fibrillation; thus, procainamide is the best answer choice.

Agents that slow conduction through the atrioventricular node (such as beta-blockers, calcium
channel blockers, or amiodarone) can lead to preferential conduction along the accessory pathway
and may precipitate ventricular fibrillation.

Magnesium can be a useful adjunctive therapy for polymorphic ventricular tachycardia associated
with long QT syndrome. There are case series of transient slowing of accessory pathway conduction
after intravenous magnesium administration in sinus rhythm, but magnesium does not have an
established role in managing pre-excited atrial fibrillation.

Lidocaine, a class Ic antiarrhythmic drug, is useful for substrate-mediated (monomorphic) ventricular


tachycardia but does not treat atrial fibrillation. Additionally, lidocaine has been reported to
accelerate accessory pathway conduction.

17. The correct answer: E

KKUH
Collected by:
Dr Hani Abdullah
Dr Fatma Hadi
Dr Salem Boresa
Dr Rami Elesali
Dr Naeif Almagal
1/1/2020
27

Question 18
18. A 68-year-old man was referred for evaluation and treatment of paroxysmal atrial fibrillation. He
experiences symptoms attributable to atrial fibrillation infrequently, but they are bothersome and
interfere with his quality of life.

His medical history was notable for hypertension but was otherwise unremarkable. His medications
included apixiban 5 mg BID, lisinopril 10 mg daily, and metoprolol succinate 50 mg daily.

A recent echocardiogram demonstrated normal right and left ventricular function. There was no
significant valve disease. The left atrium was mildly dilated. Estimated pulmonary artery systolic
pressure (PASP) was 25 mm Hg. A nuclear stress test performed 1 year ago in the setting of
atypical chest pain demonstrated normal perfusion at rest and stress.

You decided to prescribe flecainide for pill-in-the-pocket treatment of his atrial fibrillation.

What property of flecainide makes it particularly effective in this setting?


A. Use dependence.
B. Shortening of the action potential.
C. Shortening of the refractory period.
D. Increase rate of membrane depolarization.
E. Increase in delayed afterdepolarizations.

Class Ic antiarrhythmic agents slow conduction by blocking open sodium channels. They dissociate
slowly from the sodium channels during diastole, making them effective at rapid heart rates. This
property of the class Ic antiarrhythmics is called "use dependence" and is responsible for their
efficacy, particularly in supraventricular arrhythmias. This is also why these medications can be used
successfully as part of a "pill-in-pocket" approach for supraventricular tachycardia. Use dependence
is seen most commonly with class Ic drugs (flecanide, propafenone), less commonly with class Ia
drugs (quinidine, procainamide), and rarely in class Ib drugs (mexilitine). Thus, use dependence is
the correct choice.

Sotalol is a class III agent that blocks potassium channels, resulting in prolongation of repolarization,
action potential duration, and the refractory period. Class III agents have "reverse use dependence"
and are generally more effective at slower heart rates. Therefore, sotalol is more effective as a daily
medication for maintenance of sinus rhythm. It is unlikely to be effective at faster heart rates as part
of a "pill-in-pocket" approach.

Class Ib antiarrhythmic drugs increase the rate of membrane depolarization, increase delayed
afterdepolarizations, and shorten the refractory period; flecainide does not.

18. The correct answer: A

Key Point
Class I drugs block inward sodium channels and slow conduction velocity in the myocardium. Class I
drugs exhibit a phenomenon called "use dependence" in which the extent of channel block is

KKUH
Collected by:
Dr Hani Abdullah
Dr Fatma Hadi
Dr Salem Boresa
Dr Rami Elesali
Dr Naeif Almagal
1/1/2020
28

increased at a higher heart rate. These drugs therefore block the sodium channels to a greater
degree at faster heart rates. Proarrhythmic potential of class I drugs can be assessed by performing
treadmill exercise testing to increase the heart rate.

Question 19
19. A 50-year-old male with paroxysmal atrial fibrillation and CHADSVaSC 0 was admitted with two
weeks of symptomatic atrial fibrillation. While in the hospital, he underwent a transesophageal
echocardiogram and cardioversion, and was loaded on amiodarone.

Which of the following is the most appropriate anticoagulation regimen?


A. No anticoagulation.
B. Aspirin plus clopidogrel.
C. Apixaban.
D. Warfarin.
E. Ticagrelor.

Patients with atrial fibrillation lasting more than 48 hours or of uncertain duration require
anticoagulation after cardioversion independent of their CHADSVaSC risk score because of the high
rates of thromboembolism in the 4 weeks following cardioversion. Possible regimens for
anticoagulation in the immediate post-cardioversion period include a direct oral anticoagulant such
as dabigatran, rivaroxaban, apixaban, or edoxaban; or warfarin with a heparin bridge until the
international normalized ratio is >2.0. Clopidogrel plus aspirin was studied in the ACTIVE-W trial and
proven to be inferior to warfarin for reduction of thromboembolic events in patients with atrial
fibrillation. Ticagrelor has never been studied in atrial fibrillation.

19. The correct answer: C


Key Point
Because of the risk of cardioembolic stroke associated with cardioversion, anticoagulation is
recommended independent of the CHA2DS2-VASc score in patients in whom AF has lasted >48
hours or if there is uncertainty regarding onset time.

KKUH
Collected by:
Dr Hani Abdullah
Dr Fatma Hadi
Dr Salem Boresa
Dr Rami Elesali
Dr Naeif Almagal
1/1/2020
29

Question 20
20. Genetic testing for diagnosing long QT syndrome (LQTS) would be most appropriate for which of
the following individuals?

A 53-year-old male with a history of myocardial infarction, left ventricular


A.
ejection fraction 45% on amiodarone, and QTc 510 msec.
A 32-year-old male with QTc 470 msec, systolic murmur with Valsalva
B.
maneuver, and syncope while playing basketball.
A 51-year-old female with QTc 400 msec whose daughter has long QT
C.
syndrome with positive genetic screen.
A 28-year-old male with QTc 520 msec whose brother has suspected long
D.
QT syndrome and negative genetic screen.
A 45-year-old female with frequent premature ventricular complexes,
E.
right bundle branch block, and QTc 485 msec.

The Heart Rhythm Society/European Heart Rhythm Association (HRS/EHRA) Expert Consensus
Statement on the State of Genetic Testing for the Channelopathies and Cardiomyopathies provides
recommendations for when genetic testing for LQTS may be appropriate. Of note, the sensitivity of
genetic testing for suspected LQTS is approximately 80% and 20-25% of patients with LQTS
confirmed by a known genetic mutation may have a normal-range QTc.

• Class I recommendation for comprehensive or LQTS types 1-3 (LQT1-3)-targeted LQTS


genetic testing (e.g., KCNQ1, KCNH2, and SCN5A) for patients with a strong clinical
suspicion for LQTS and QT prolongation.
• Class I recommendation for comprehensive or LQT1-3–targeted LQTS genetic testing for
asymptomatic adult patients with QTc >500 msec (adults) or >480 msec (prepubescents) in
the absence of other clinical conditions that might prolong the QT interval (e.g., electrolyte
abnormalities, hypertrophy, bundle branch block).
• There is also a Class I recommendation for mutation-specific genetic testing for first-degree
relatives of individuals with LQTS and an identified causative mutation.
• There is a Class IIb recommendation for (what may be considered) comprehensive or LQT1-
3–targeted LQTS genetic testing (e.g., KCNQ1, KCNH2, and SCN5A) for any asymptomatic
patient with otherwise idiopathic QTc values >480 msec (adults) or >460 msec
(prepubescents) on serial 12-lead electrocardiograms (ECGs).

The 51-year-old female with normal QTc (400 msec) whose daughter has LQTS with positive
genetic screen should have genetic screening, as her daughter has LQTS and an identified
causative mutation. The 28-year-old male with QTc 520 msec whose brother has
suspected LQTS would not be expected to have a positive genetic screen despite his long
QTc due to the negative screen in his brother (the proband). The 45-year-old female with
QTc 485 msec and 53-year-old male with QTc 510 msec have alternative explanations for
prolonged QTc (right bundle branch block [RBBB] and drug induced, respectively). Finally,
the 32-year-old male with QTc 470 msec, systolic murmur with Valsalva maneuver, and
syncope while playing basketball is more likely to have hypertrophic cardiomyopathy and
does not meet other criteria for LQTS testing.

20. The correct answer: C

KKUH
Collected by:
Dr Hani Abdullah
Dr Fatma Hadi
Dr Salem Boresa
Dr Rami Elesali
Dr Naeif Almagal
1/1/2020
30

Key Point
The success of genetic testing in LQTS depends in large part on the robustness of the clinical
diagnosis with mutations identified in 70-80% of cases when the diagnosis is certain (QTc >480
msec), but dropping significantly in borderline cases.

Question 21
21. A 32-year-old female presented to your office for evaluation of syncope. She has had 3 syncopal
episodes over the past 3 months. The first occurred while exercising on a treadmill, the second while
running to catch the bus, and the third while eating dinner. During the most recent episode, she
bruised her cheek and nose.

She has no past medical history. Her medications include sertraline 50 mg daily and an oral
contraceptive pill. Her family history is unknown, as she was adopted.

On examination, her heart rate was 74 bpm and blood pressure 110/70 mm Hg. Her lungs were
clear, heart was regular with no murmurs, and extremities were without edema.

Laboratories showed sodium 140 mmol/L, potassium 4.2 mEq/L, magnesium 2.2 mg/dL, and
calcium 9.8 mg/dL. An electrocardiogram (ECG) was recorded (Figure 1).

What is the best next step?

(Figure 1)
KKUH
Collected by:
Dr Hani Abdullah
Dr Fatma Hadi
Dr Salem Boresa
Dr Rami Elesali
Dr Naeif Almagal
1/1/2020
31

A. Start nadolol 40 mg daily.


B. Implantable cardioverter-defibrillator implantation.
C. Perform genetic testing.
D. Stop sertraline.
E. Start verapamil sustained release 180 mg daily.

Long QT syndrome (LQTS) is diagnosed in the presence of corrected QT interval (QTc) = 500 msec
or LQTS risk score = 3.5 when secondary causes have been excluded or in the presence of a
pathogenic mutation in one of the LQTS genes. It can also be diagnosed when the QTc is 480-499
msec in a patient presenting with syncope. There are several genetic forms of LQTS, which affect
presentation and response to therapy. Given that syncope is often the result of an arrhythmic event
in patients with LQTS, early recognition and treatment are needed to avoid recurrences, which could
present as cardiac arrest or sudden cardiac death.

This ECG shows a QT of 639 msec with a QTc >480 msec in a patient presenting with syncope; the
diagnosis of LQTS can be made. Beta-blocker therapy, in the absence of contraindications, is
indicated as a first-line therapy in patients with LQTS and suspected arrhythmic syncope. Thus, the
correct answer is nadolol.

There are a number of factors that may cause acquired LQTS, including metabolic disorders of
hypokalemia, hypomagnesemia, and hypocalcemia, and medications such as -azole antifungals,
macrolide antibiotics, antipsychotics, and antiemetics. The website www.crediblemeds.org is a
resource for QT-prolonging drugs. Sertraline does not typically cause LQTS and thus there is no
need to stop this medication.

Implantable cardioverter-defibrillator (ICD) implantation is reasonable in patients with LQTS and


suspected arrhythmic syncope who are on beta-blocker therapy or are intolerant to beta-blocker
therapy. As this patient has not failed beta-blocker therapy, ICD is not indicated.

Verapamil may be prescribed to patients with catecholaminergic polymorphic ventricular tachycardia


(VT) but is not used in LQTS.

Genetic testing would not change management in a patient who has LQTS diagnosed by the
presence of syncope and a QTc >480 msec, and thus is not the best next step.

21.The correct answer: A


Key Point
LQTS can be diagnosed clinically if a patient presents with recurrent syncope and QTc interval >480
msec in repeated ECGs in the absence of secondary causes (e.g., drugs, electrolytes).

KKUH
Collected by:
Dr Hani Abdullah
Dr Fatma Hadi
Dr Salem Boresa
Dr Rami Elesali
Dr Naeif Almagal
1/1/2020
32

Question 22
22. A 23-year-old male law school student presented for evaluation of several months of palpitations
and lightheadedness. These episodes often occur when he is playing hockey but also can occur
during mock trials. His physical examination was unremarkable. He underwent echocardiography,
which showed normal biventricular size and function and no valvular abnormalities. His
electrocardigram (ECG) was normal. His family history was significant for a maternal uncle who died
at age 24 while skiing.

Which of the following mutations is the most likely cause of his symptoms?
A. Plakophilin.
B. Ryanodine receptor.
C. Myosin heavy chain.
D. Transforming growth factor-beta.
E. Fibrillin.

This patient most likely has catecholaminergic polymorphic ventricular tachycardia (CPVT) given his
family history and symptoms with exercise and emotional stress. Although both calsequestrin and
ryanodine receptor mutations can cause this syndrome, ryanodine receptor mutations are much
more common and account for almost half of cases. Transforming growth factor-beta (TGF-β)
mutations are associated with familial thoracic aortic aneurysm diseases. Fibrillin mutations are
associated with Marfan syndrome. Myosin heavy chain mutations are associated with dilated
cardiomyopathy and hypertrophic cardiomyopathy. Mutations in plakophilin are associated with
arrhythmogenic right ventricular cardiomyopathy.

22. The correct answer: B


Key Point
A number of inheritable syndromes associated with increased risk of sudden death are recognized.

Question 23
23. A 23-year-old female with no past history presented to the emergency department after a
syncopal episode. She was eating dinner with her family when she felt faint and passed out, bruising
her forehead and cheek on the side of the table as she fell to the floor. Paramedics were called and
an electrocardiogram (ECG) was obtained (Figure 1).

Her medical history includes exercise-induced asthma and frequent urinary tract infections. She
uses albuterol as needed, takes a daily oral contraceptive, and recently completed a course of
levofloxacin. She does not smoke cigarettes or drink alcohol but smokes marijuana a few
days/week. Her family history is unremarkable.

KKUH
Collected by:
Dr Hani Abdullah
Dr Fatma Hadi
Dr Salem Boresa
Dr Rami Elesali
Dr Naeif Almagal
1/1/2020
33

On examination, she was a young, thin female appearing scared but otherwise in no distress. Her
heart rate was 104 bpm and blood pressure was 96/60 mm Hg. A cardiac examination revealed
premature beats and her examination was otherwise unremarkable.

Laboratories showed thyroid stimulating hormone (TSH) 4.2 U/mL, potassium (K) 3.8 mEq/L, and
magnesium (Mg) 1.9 mg/dL.

Which of the following, in conjunction with levofloxacin, is most likely the cause of her presentation?

(Figure 1)
A. Hypokalemia.
B. Marijuana.
C. Hypomagnesemia.
D. Oral contraceptives.
E. Albuterol.

This patient presented with polymorphic ventricular tachycardia (VT) resulting in syncope. Although
drug-induced polymorphic VT is sometimes regarded as an idiosyncratic event, a number of risk
factors have been identified. Patients with multiple risk factors may face the greatest risk.

A comprehensive list of QT-prolonging drugs can be found at www.crediblemeds.org. Concurrent


use of more than one drug that can prolong the QT interval or use of a QT-prolonging drug with one
that slows drug metabolism due to inhibition of hepatic cytochrome P450 enzymes increases the risk
of polymorphic VT. Other risk factors for polymorphic VT include baseline QT prolongation,
bradycardia, and electrolyte disturbances (especially hypokalemia and hypomagnesemia, and less
often hypocalcemia).

This patient is on two medications with a known risk of polymorphic VT, albuterol and levofloxacin.

KKUH
Collected by:
Dr Hani Abdullah
Dr Fatma Hadi
Dr Salem Boresa
Dr Rami Elesali
Dr Naeif Almagal
1/1/2020
34

This is the correct answer.

Oral contraceptives and marijuana do not prolong the QT interval separately or in combination with
levofloxacin, so these answers are incorrect.

Low potassium and low magnesium can prolong the QT intervals, but her levels would not be
considered low enough to increase the risk and thus these answers are not correct.

23. The correct answer: E


Key Point
Ventricular arrhythmia may be either monomorphic with a single QRS morphology or polymorphic
with changing QRS morphology. The most common cause of polymorphic ventricular tachycardia
(PVT) is acute myocardial ischemia, which may quickly progress to ventricular fibrillation (VF). Less
commonly, PVT is caused by circumstances that prolong the Q-T interval (drugs and ion-channel
disorders). It is important to recognize which common drugs are associated with Q-T prolongation
and discontinue.

Question 24
24. A 26-year-old female with no past medical history came to see you in clinic for follow-up after an
emergency department visit for an episode of syncope. She said that she was exercising when she
began feeling dizzy and nauseated. She tried to walk to the bathroom and then lost consciousness.
Her husband ran to her side and found her awake and oriented, but with a rapid pulse. By the time
emergency medical technicians arrived, her vital signs were normal, and her evaluation in the
emergency department was unremarkable.

On physical examination today, her heart rate was 66 bpm and blood pressure was 108/72 mm Hg.
She appeared well. Jugular venous pressure was normal and lungs were clear. There were no
murmurs. She had no lower extremity edema. Her electrocardiogram (ECG) demonstrated normal
sinus rhythm and right bundle branch block (RBBB) with QRS duration of 128 msec. Electrolytes
were within normal limits. Her echocardiogram showed normal left ventricular (LV) function, mild
right ventricular (RV) enlargement, mild RV dysfunction, and mild tricuspid regurgitation. An exercise
treadmill test in which the patient had multiple runs of the rhythm was performed (Figure 1). There is
no family history of syncope, cardiac arrhythmia, or sudden death.

Which of the following is the most appropriate next step?

KKUH
Collected by:
Dr Hani Abdullah
Dr Fatma Hadi
Dr Salem Boresa
Dr Rami Elesali
Dr Naeif Almagal
1/1/2020
35

A. Genetic testing.
B. Endomyocardial biopsy.
C. Cardiac catheterization.
D. Cardiac magnetic resonance imaging.

The concern is that this young female has arrhythmogenic right ventricular cardiomyopathy (ARVC)
because she has exercise-induced ventricular tachycardia and RV enlargement on echocardiogram,
as well as RBBB on ECG. In order to meet criteria for ARVC, there should be two major criteria, or
one major and two minor criteria, in the categories of ventricular dysfunction, tissue characterization,
arrhythmias, and ECG de/repolarization criteria. The magnetic resonance imaging (MRI) will allow
both evaluation of ventricular dysfunction and size, as well as the tissue and fibrofatty infiltration
(Figure 2). Endomyocardial biopsy would also be helpful for tissue characterization, but is invasive
and subject to sampling error. MRI provides more data with less risk to the patient. Once the
diagnosis is made, you can refer for genetic testing for confirmation and family planning.

KKUH
Collected by:
Dr Hani Abdullah
Dr Fatma Hadi
Dr Salem Boresa
Dr Rami Elesali
Dr Naeif Almagal
1/1/2020
36

Reproduced with permission from Tandri H, Castillo E, Ferrari VA, et al. Magnetic resonance
imaging of arrhythmogenic right ventricular dysplasia: sensitivity, specificity, and observer variability
of fat detection versus functional analysis of the right ventricle. J Am Coll Cardiol 2006;48:2277-84.
24. The correct answer: D
Key Point
The important differential diagnosis in right ventricular outflow tract ventricular tachycardia (VT) is VT
occurring in the situation of arrhythmogenic right ventricular cardiomyopathy (ARVC). One important
difference is that the resting 12-lead echocardiogram (ECG) shows marked T-wave inversion in the
right precordial leads (V1-V3/4) and there may be evidence of an epsilon wave. The 12-lead ECG in
patients with normal heart VT is normal. If ARVC is suspected, computed tomography or magnetic
resonance imaging should be performed to rule out this condition, which is a genetic abnormality.

Question 25
25. A 68-year-old male with a history of hypertension presented to the emergency department for
evaluation of palpitations. An electrocardiogram (ECG) was recorded (Figure 1).

He was advised to undergo catheter-based ablation.

Which of the following best describes the ablation that will be performed?

KKUH
Collected by:
Dr Hani Abdullah
Dr Fatma Hadi
Dr Salem Boresa
Dr Rami Elesali
Dr Naeif Almagal
1/1/2020
37

(Figure 1)

Ablation of the posterior slow pathway most commonly located in the posterior
A.
third of Koch's triangle.
Radiofrequency energy is applied to create a line of ablation from the tricuspid
B.
annulus to the inferior vena cava.
Radiofrequency ablation of the pathway localized by catheter-based mapping of
C.
the atrial and ventricular insertion sites.
The release of a cryoablation balloon delivers confluent lesions that encircle the
D.
ostia of all four pulmonary veins.
Substrate-based extensive ablation of myocardium displaying abnormal
E.
electrogram characteristics.

Catheter ablation using radiofrequency or cryothermal energy is an important therapy in the


management of patients with various types of tachyarrhythmia. Catheter ablation is generally
indicated for the treatment of a recurrent or persistent symptomatic arrhythmia that has been
refractory to medical therapy or for which medical therapy is not tolerated or preferred. For select
arrhythmias known to have a high cure rate with ablation therapy (e.g., atrial flutter, paroxysmal
supraventricular tachycardia, Wolff-Parkinson-White [WPW] syndrome, idiopathic premature
ventricular complexes/ventricular tachycardia), catheter ablation may be indicated as a first-line
treatment.

The ECG shows atrial flutter. Typical (also called isthmus-dependent) atrial flutter utilizes a large
macroreentrant pathway in the right atrium. The cavotricuspid isthmus between the inferior vena
cava and the tricuspid annulus (IVC-TA isthmus) is an obligatory route for typical flutter and, as
such, is the best anatomic target for ablation. Thus, creation of a line of ablation from the TA to the
IVC is the correct answer.

Various methods of catheter ablation have been used for atrial fibrillation, and most focus on
isolating the triggers in the pulmonary veins (PVs) from the vulnerable substrate in the left atrium.
KKUH
Collected by:
Dr Hani Abdullah
Dr Fatma Hadi
Dr Salem Boresa
Dr Rami Elesali
Dr Naeif Almagal
1/1/2020
38

Circumferential PV isolation involves the creation of confluent ablation lesions that encircle the ostia
of all four PVs. The goal of these lesions is to electrically separate the PVs from the left atrium.
Thus, PV isolation is not the correct answer, as it describes the ablation of atrial fibrillation (AF), not
atrial flutter.

The general approach to the catheter ablation of atrioventricular nodal re-entry tachycardia (AVNRT)
is based upon the concept of dual atrioventricular (AV) nodal pathways. The most common AVNRT
circuit involves anterograde conduction down the slow pathway and retrograde conduction up the
fast pathway. The ablation target is the posterior slow pathway since ablation here carries the lowest
risk of AV block, preserves fast pathway function (and a normal P-R interval postablation), and is
facilitated by reliable anatomic and electrophysiologic landmarks of Koch's triangle. Thus, ablation of
the slow pathway describes AVNRT ablation and is not the correct answer.

Patients with recurrent sustained monomorphic ventricular tachycardia (VT) resulting in implantable
cardioverter-defibrillator (ICD) shocks despite treatment with an antiarrhythmic drug may benefit
from catheter-based radiofrequency ablation. VT in patients with a prior myocardial infarction (MI) is
usually due to re-entry in a circuit created by the heterogeneous electrical properties of residual
myocardium in the region of the scar from the infarct. Usually, extensive ablation of myocardium
displaying abnormal electrogram characteristics is performed, so-called substrate-based ablation.
Thus, substrate ablation describes VT ablation and is not correct.

For patients with an accessory pathway and symptomatic arrhythmias including orthodromic
atrioventricular re-entry tachycardia (AVRT), antidromic AVRT, and pre-excited AF or atrial flutter,
catheter ablation is recommended. The location of most accessory pathways can be estimated using
the pre-excitation pattern on the surface echocardiogram. However, more precise localization of the
accessory pathway during catheter-based mapping prior to catheter ablation utilizes several
parameters to determine the atrial and ventricular insertion sites. Thus, ablation of the accessory
pathway seen in WPW syndrome is not the correct answer.

25. The correct answer: B


Key Point
Atrial flutter is a macro–re-entrant circuit most commonly within the RA and is sustained through a
region of slow conduction along a channel or “isthmus” between the IVC and the tricuspid valve, the
so called “cavotricuspid isthmus.” Due to the critical dependence of the arrhythmia on conduction
through this region, the rhythm is classified a “cavotricuspid isthmus-dependent” or CTI-dependent
atrial flutter.

KKUH
Collected by:
Dr Hani Abdullah
Dr Fatma Hadi
Dr Salem Boresa
Dr Rami Elesali
Dr Naeif Almagal
1/1/2020
39

Question 26

26. An 84-year-old woman with paroxysmal atrial fibrillation and hypertension presented to the
hospital after a syncopal episode without prodrome while eating dinner. She regained consciousness
after 20 seconds, and other than a contusion on her cheek, she was unharmed. There was no
incontinence or generalized tonic clonic activity.

Her heart rate was 64 bpm and blood pressure was 146/80 mm Hg. She had a systolic ejection
murmur and her examination was otherwise unremarkable. An electrocardiogram (ECG)
demonstrated normal sinus rhythm with a right bundle branch block (RBBB) and left anterior
fascicular block. An echocardiogram showed normal biventricular function and aortic sclerosis.
Myocardial perfusion stress test revealed no evidence of ischemia.

The patient underwent an electrophysiology (EP) study without inducible ventricular tachycardia
(VT). An intracardiac electrogram was recorded (Figure 1).

Which of the following is the best next step?

KKUH
Collected by:
Dr Hani Abdullah
Dr Fatma Hadi
Dr Salem Boresa
Dr Rami Elesali
Dr Naeif Almagal
1/1/2020
40

A. Single-chamber pacemaker.
B. Cardiac resynchronization therapy pacemaker.
C. Event monitor.
D. Dual-chamber pacemaker.
E. Implantable loop recorder.

The patient developed syncope in the setting of bifascicular block. An EP study is appropriate given
the high likelihood of a malignant cardiac etiology of her symptoms with a negative standard
evaluation. EP study reveals a prolonged H-V interval (normal is 35-55 msec). Current guidelines
give a Class I indication to permanent pacing in the setting of prolonged H-V interval, and a Class IIa
indication to implantation for unexplained syncope in the setting of bifascicular block. Thus,
permanent pacing is indicated.

Further testing with ambulatory ECG monitoring with an event monitor or implantable loop recorder
is not indicated because the etiology was already established on EP study.

In a patient with normal ejection fraction, cardiac resynchronization therapy is not indicated.
However, if she is subsequently noted to have frequent right ventricular pacing (>40%),
consideration of upgrade to cardiac resynchronization therapy (CRT) is reasonable.

An implantable cardioverter-defibrillator is not indicated, as her EP study showed no inducible VT


and her ejection fraction is >35%. As this patient has paroxysmal, not permanent, atrial fibrillation,
she may benefit from AV synchrony, which a dual-chamber pacemaker would provide and thus a
single-chamber pacemaker is not the best choice.

26. The correct answer: D


Key Point
AV block occurs due to a failure of impulse propagation through the cardiac conduction system.
Several types of AV block are recognized; most commonly first-, second-, and third-degree block.
Prognosis of AV block is related to the severity of block. AV block may be due to failure of impulse
propagation in the AV nodal tissues (narrow QRS complex) or within the His-Purkinje system (intra-
or infra-Hisian block). Failure of impulse propagation due to infra-Hisian block may be associated
with a wide QRS complex with bundle branch or fascicular block.

Question 27
27. A 54-year-old female presented to your office with frequent symptoms of palpitations associated
with lightheadedness. She has a history of hypertension and takes lisinopril. She denies any family
history of cardiac disease. She was recently seen in the emergency department with a 12-lead
electrocardiogram (ECG) during her palpitations (Figure 1). She was given 6 mg of adenosine; this
did not cause her rhythm to change, although by report there was evidence of organized atrial
activity during the transient period of atrioventricular (AV) block. Her palpitations resolved
spontaneously while she was in the emergency department and a repeat ECG was performed
(Figure 2).

Which of the following is the most likely etiology of her tachycardia?

KKUH
Collected by:
Dr Hani Abdullah
Dr Fatma Hadi
Dr Salem Boresa
Dr Rami Elesali
Dr Naeif Almagal
1/1/2020
41

(Figure 1)

(Figure 2)

A. Re-entrant tachycardia involving dual pathway physiology within the atrioventricular node.
B. Rapidly firing foci originating from the pulmonary vein.
C. Enhanced automaticity originating within the fascicular system.
D. Re-entrant tachycardia involving an accessory pathway.
E. Abnormal ectopic atrial foci.

KKUH
Collected by:
Dr Hani Abdullah
Dr Fatma Hadi
Dr Salem Boresa
Dr Rami Elesali
Dr Naeif Almagal
1/1/2020
42

Her ECG shows a narrow complex, regular tachycardia with an abnormal P-wave axis consistent
with atrial tachycardia. Administration of adenosine is more likely to convert an AV nodal-dependent
re-entrant tachycardia, such as AV nodal re-entrant tachycardia (AVNRT) or AV re-entrant
tachycardia (AVRT), than atrial tachycardia. The ECG in Figure 2 shows sinus rhythm and there is a
difference in P-wave morphology and axis compared with the supraventricular tachycardia (SVT)
tracing. Atrial tachycardia originates from an ectopic atrial focus. It is the least common cause of
SVT in a patient with a normal baseline ECG. Enhanced automaticity within the fasicular system
would cause fascicular ventricular tachycardia and would not demonstrate rapid atrial activity during
the period of AV block after administration of adenosine. Typical AVNRT involves dual-pathway
physiology within the AV node and may be precipitated by a premature atrial depolarization that is
blocked in one of the pathways. Because the re-entrant circuit involves the AV node, it is more likely
to terminate with administration of adenosine and would not show evidence of organized atrial
activity during the period of AV block after adenosine. Similarly, AVRT involves a re-entrant circuit
consisting of an accessory pathway and the AV node, and is also more likely to terminate with
administration of adenosine. The presence of rapidly firing foci in the pulmonary veins is one
mechanism of atrial fibrillation, which is irregular and would not reveal organized atrial activity during
the period of AV block after administration of adenosine.

27. The correct answer: E


Key Point
PSVT may be due to AVNRT, accessory pathway mediated tachycardia (AVRT), or AT. AT is most
commonly due to a single abnormally firing focus in the atrium. Unlike AVNRT and AVRT, AT is not
dependent on conduction through the AV node. Adenosine is an effective acute treatment of both
AVNRT and AVRT, but is less effective against AT, although use of adenosine in this situation may
produce transient AV block and unmask the mechanism of the tachycardia.

Question 28
28. A 30-year-old male with rheumatoid arthritis presented to your office for evaluation of syncope.
His medications include prednisone 7.5 mg daily, methotrexate 15mg weekly, hydroxychloroquine
200 mg daily, and dapsone 50 mg daily.

His blood pressure was 140/80 mm Hg with a pulse of 80 bpm. His examination was remarkable for
mildly swollen metacarpal joints bilaterally and was otherwise normal.

He became lightheaded in your office and an electrocardiogram (ECG) was obtained (Figure 1).

Which of the following is the most likely cause of the arrhythmia seen?

KKUH
Collected by:
Dr Hani Abdullah
Dr Fatma Hadi
Dr Salem Boresa
Dr Rami Elesali
Dr Naeif Almagal
1/1/2020
43

(Figure 1)
Reproduced with permission from Roediger JE. 2012. Available at:
https://en.wikipedia.org/wiki/Torsades_de_pointes#/media/File:Torsades_de_Pointes_TdP.png.
Accessed 05/31/2018

A. Hydroxochloroquine.
B. Methotrexate.
C. Dapsone.
D. Prednisone.
E. Rheumatoid heart disease.

Acquired long QT syndrome can occur due to a variety of factors including medications and
electrolyte derangements. A comprehensive list can be found at www.crediblemeds.org.

In this patient, hydroxychloroquine is the likely culprit. His QT interval may return to normal once this
medication is stopped. Thus, the first step in assessment of long QT syndrome is identification of
treatable/reversible causes.

Prednisone, methotrexate, and dapsone do not prolong the QT interval and are incorrect choices.

Although rheumatoid arthritis can have an associated cardiomyopathy, there are no other symptoms
or signs of heart failure and this would not explain the ECG findings.

28. The correct answer: A

KKUH
Collected by:
Dr Hani Abdullah
Dr Fatma Hadi
Dr Salem Boresa
Dr Rami Elesali
Dr Naeif Almagal
1/1/2020
44

Key Point
Ventricular arrhythmia may be either monomorphic with a single QRS morphology or polymorphic
with changing QRS morphology. The most common cause of polymorphic ventricular tachycardia
(PVT) is acute myocardial ischemia, which may quickly progress to ventricular fibrillation (VF). Less
commonly, PVT is caused by circumstances that prolong the Q-T interval (drugs and ion-channel
disorders). It is important to recognize which common drugs are associated with Q-T prolongation
and discontinue.

Question 29
29. An 82-year-old female with a history of hypertension and type II diabetes mellitus presented to
the hospital with dyspnea on exertion and lower extremity swelling and redness. She was placed on
vancomycin for presumed cellulitis. Her inpatient medications included aspirin 81 mg daily, atenolol
50 mg daily, amlodipine 10 mg daily, sliding scale insulin, heparin 5,000 units subcutaneous three
times a day, and pravastatin 40 mg daily. She had progressive shortness of breath. Chest computed
tomography (CT) with intravenous contrast showed no pulmonary embolism but did demonstrate
vascular congestion. She was started on furosemide 80 mg intravenous twice a day. The following
day, she was noted to have oliguria.

Which of the following medications should be held at this point?


A. Atenolol.
B. Amlodipine.
C. Heparin.
D. Pravastatin.
E. Aspirin.

The patient was admitted with lower extremity edema and dyspnea due to heart failure with
preserved ejection fraction (EF). However, on presentation she was started on vancomycin for
possible cellulitis. The combination of vancomycin, iodinated contrast, and acute heart failure
precipitated oliguric renal failure. Atenolol is a water-soluble beta-blocker and has a prolonged half-
life in the setting of renal failure. Therefore, it can accumulate to toxic levels in the setting of renal
failure and may cause significant sinus node dysfunction with severe bradycardia. The other
medications listed may be continued in the setting of acute renal failure.

29. The correct answer: A


Key Point
SND is a common and often age-related cause of bradycardia although extrinsic causes such as
drug therapy also may cause apparent SND.

KKUH
Collected by:
Dr Hani Abdullah
Dr Fatma Hadi
Dr Salem Boresa
Dr Rami Elesali
Dr Naeif Almagal
1/1/2020
45

Question 30
30. A 63-year-old male came to the emergency department after an episode of syncope. Earlier in
the day, he had sudden onset of rapid palpitations and took a dose of propafenone; less than an
hour later, he stumbled and fell to the floor. He has a longstanding history of paroxysmal atrial
fibrillation with frequent episodes of symptomatic rapid ventricular response. He has good control of
his symptoms with pill-in-the-pocket propafenone. His other medical history includes hypertension,
benign prostatic hypertrophy, and chronic obstructive pulmonary disease. His current medications
are losartan 100 mg, tamsulosin 0.4 mg, and ipratropium inhaled as needed. His pulse was 58 bpm,
blood pressure was 130/82 mm Hg, and respirations were 12 breaths per minute. A physical
examination disclosed a well-appearing middle-aged male with regularly split S1 and S2, no
murmurs, and normal peripheral pulses. His electrocardiogram (ECG) showed sinus bradycardia
with P-R interval 180 msec and QRS duration 98 msec. Serum chemistries were normal and
troponin was negative. He underwent a stress echocardiogram 1 year ago, exercising 8:30 minutes
and reaching 86% age-predicted maximum heart rate with no ischemic ECG changes and normal
left ventricular ejection fraction (LVEF).

In addition to stopping propafenone, which of the following is the best next step in his care?

A. Thirty-day monitor.
B. Electrophysiology study.
C. Implantable loop recorder.
D. Tilt table test.
E. Coronary angiogram

The correct answer is a 30-day event monitor to assess for sinus pauses or other signs of sinus
node dysfunction after discontinuing the propafenone. Both propafenone and flecainide can
exacerbate underlying conduction system disease, unmasking sinus node dysfunction,
atrioventricular block, or infrahisian block, and increases in the P-R and QRS intervals of ≤25% over
baseline values are commonly seen with the use of these drugs. In this patient who has been using
propafenone without adverse effects for years, it is likely that sinus node dysfunction has gradually
progressed to the point that it is now symptomatic when using propafenone, but more information is
required to determine if invasive electrophysiology studies or pacemaker implantation will be
required.

He has no anginal symptoms and a negative troponin, as well as a negative stress test a year ago,
so there is no indication for coronary angiography at this time.

Tilt table testing would not be expected to add diagnostic value, as his symptoms do not point to
orthostatic or autonomic dysfunction.

An implantable loop recorder is not indicated at this time.

30. The correct answer: A


Key Point
Age-related SND also may be associated with AF, the so-called “tachy-brady” syndrome. This may
be associated with prolonged pauses, which can be symptomatic after termination of AF. A common
KKUH
Collected by:
Dr Hani Abdullah
Dr Fatma Hadi
Dr Salem Boresa
Dr Rami Elesali
Dr Naeif Almagal
1/1/2020
46

scenario would be an elderly patient with persistent AF who develops a prolonged pause one
termination of AF. This can be exacerbated by drugs, especially those that block sodium channels
such as flecainide.

Question 31
31. A 64-year-old male with a history of sinus node dysfunction and complete heart block who
underwent dual-chamber permanent pacemaker 2 years ago came to your office complaining of
intermittent palpitations. During performance of his electrocardiogram by the nurse, he felt the onset
of palpitations, which continued for a minute after the electrocardiogram was completed (Figure 1).

Which of the following interventions is most likely to decrease his symptoms of palpitations?

A. Increasing his lower rate to 80 bpm.


B. Metoprolol 25 mg BID.
C. Ventricular lead revision.
D. Shortening his postventricular atrial refractory period.
E. Decreasing his atrioventricular delay.

The beats are ventricular ectopics and are not related to pacing. There is no evidence of pacemaker
lead malfunction, as there is appropriate sensing and ventricular capture. A trial of an antiarrhythmic,

KKUH
Collected by:
Dr Hani Abdullah
Dr Fatma Hadi
Dr Salem Boresa
Dr Rami Elesali
Dr Naeif Almagal
1/1/2020
47

such as a beta blocker, should be attempted. If unsuccessful and he remains highly symptomatic,
then a ventricular tachycardia ablation could be considered.

31. The correct answer: B


Key Point
In many cases, PVCs are benign and do not require further treatment unless symptomatic.

Question 32
32. A 38-year-old male was admitted to the hospital following an episode of syncope. He had been
seated, watching late-night television, when he became unresponsive. A family member called 911
but he regained consciousness before the paramedics arrived. There was no postictal confusion and
he was reluctant to come to the hospital, but his wife insisted, saying, "This has happened before."
He does not remember details of the prior syncope. His family history is remarkable for an uncle who
died suddenly at age 41. His vital signs were pulse 98 bpm, respirations 14 breaths per minute, and
blood pressure 116/78 mm Hg. A physical examination revealed flat jugular veins, normal heart
sounds, and warm extremities. An electrocardiogram (ECG) was recorded (Figure 1). A bedside
echocardiogram demonstrated normal right and left ventricular size and function.

Which of the following is the most appropriate next step in the management of his syncope?

(Figure 1)

KKUH
Collected by:
Dr Hani Abdullah
Dr Fatma Hadi
Dr Salem Boresa
Dr Rami Elesali
Dr Naeif Almagal
1/1/2020
48

A. Amiodarone.
B. Implantable cardioverter-defibrillator.
C. Verapamil.
D. Tilt table test.
E. Electrophysiology study.

This ECG is consistent with the Brugada type 1 pattern, described in the 2002 consensus statement
as "cove-shaped ST elevation in right precordial leads with J wave or ST elevation of at least 2 mm
(mV) at its peak followed by a negative T wave with little or no isoelectric interval in more than one
right precordial lead (V1-V3)." This type 1 pattern is diagnostic of the Brugada syndrome, and the
recurrent syncope and family history of sudden cardiac death are confirmatory clinical evidence that
intervention with an implantable cardioverter-defibrillator (ICD) is warranted. Provocative testing
such as drug challenge with procainamide or flecainide or programmed electrical stimulation during
electrophysiology testing would be indicated if he had type 2 or type 3 Brugada pattern on ECG
(Figure 2), but in cases of type 1 Brugada, these measures carry risks (of inducing ventricular
tachycardia or ventricular fibrillation) without adding benefits.

Tilt table testing can be useful for evaluating orthostatic intolerance and autonomic dysfunction, but
would not be expected to add to the diagnosis in this patient with unheralded syncope while seated.

Anti-arrhythmic therapy with either amiodarone or verapamil has not been shown to be effective in
preventing sudden cardiac death in these patients, although amiodarone may be considered for
adjunctive therapy if recurrent arrhythmias or in patients who are not candidates for ICD therapy.

KKUH
Collected by:
Dr Hani Abdullah
Dr Fatma Hadi
Dr Salem Boresa
Dr Rami Elesali
Dr Naeif Almagal
1/1/2020
49

(Figure 2)
Precordial leads of a resuscitated patient with Brugada syndrome (BrS) showing all three
echocardiogram (ECG) patterns and dynamic changes over an 8-day period. Arrows indicate J
32. The correct answer: B
Key Point
ICDs are indicated in BrS patients with prior cardiac arrest (Class I) and those with spontaneous
type I pattern and recurrent syncope.

Question 33
33. A 21-year-old male college student was seen in clinic for evaluation of palpitations. These have
occurred sporadically since his teen years but have worsened in the last 2 years. He described
feeling an extra heartbeat followed by racing heartbeats. He has sometimes been able to terminate
these episodes with coughing. His past medical history and physical examination were otherwise
unremarkable.

A resting electrocardiogram (ECG) was obtained (Figure 1).

KKUH
Collected by:
Dr Hani Abdullah
Dr Fatma Hadi
Dr Salem Boresa
Dr Rami Elesali
Dr Naeif Almagal
1/1/2020
50

Which of the following maneuvers/interventions is most likely to make the abnormal finding depicted
in the ECG less apparent?

(Figure 1)

A. Passive leg raise maneuver.


B. Inhaled amyl nitrate.
C. Exercise.
D. Intravenous metoprolol.
E. Valsalva maneuver.

This patient has a delta wave on ECG, which is due to "fusion" of impulses activating the ventricles
via both the atrioventricular (AV) node and via an accessory pathway. His history is suggestive
of paroxysmal re-entry supraventricular tachycardia (SVT). The combination of pre-excitation on
resting ECG and a history of SVT is consistent with Wolff-Parkinson-White (WPW) syndrome.

Exercise may increase conduction through the AV node, thus decreasing the degree of pre-
excitation and the delta wave will diminish.

Interventions that slow AV nodal conduction (such as metoprolol or vagal maneuvers, including the
Valsalva maneuver) will increase the degree of ventricular pre-excitation and make the delta wave
more apparent.

Amyl nitrate decreases preload, whereas a passive leg raise maneuver will increase preload. Under
normal conditions, changing preload will not change AV nodal conduction and thus should not
influence the appearance of the delta wave.

33. The correct answer: C


Key Point
Genesis of the delta wave is due to “fusion” of impulses activating the ventricles via both the AV
node and activation via the accessory pathway. In other words, a delta wave is only present if the
KKUH
Collected by:
Dr Hani Abdullah
Dr Fatma Hadi
Dr Salem Boresa
Dr Rami Elesali
Dr Naeif Almagal
1/1/2020
51

accessory pathway can conduct from the atria to the ventricles. Factors that enhance AV nodal
conduction (such as exercise) cause predominantly more of the ventricles to be activated in a
normal fashion such that the delta wave will diminish with exercise, whereas factors that slow AV
nodal conduction such as drugs will increase the degree of pre-excitation. In other words, the degree
of pre-excitation can change and is not fixed.

Question 34
34.An 82-year-old female with atrial fibrillation presented with syncope. She had been watching
television when she felt a racing heart beat followed by lightheadedness. Her past history includes
hypertension and osteoarthritis. Her medications include apixaban 5 mg twice daily, amlodipine 5 mg
daily, metoprolol succinate 25 mg daily, and flecainide 100 mg twice daily.

On examination, her heart rate was 50 bpm and blood pressure was 140/80 mm Hg. Her lungs were
clear to auscultation. A cardiac examination revealed a soft systolic ejection murmur. Her extremities
were warm, without edema.

An electrocardiogram (ECG) showed sinus bradycardia at 50 bpm with nonspecific ST-T changes.
An echocardiogram showed normal ventricular function, proximal septal hypertrophy with left
ventricular outflow tract velocity of 1.1 m/sec with Valsalva, peak aortic velocity 2.5 m/sec, and left
atrial enlargement.

A 30-day ambulatory ECG monitor demonstrated heart rate 40-110 bpm with paroxysmal atrial
fibrillation and sinus pauses of up to 4 seconds upon conversion to sinus rhythm.

What is the most likely cause of her presentation?

A. Sinus node dysfunction.


B. Aortic stenosis.
C. Chronotropic incompetence.
D. Dynamic outflow tract obstruction.
E. Autonomic dysfunction.

This elderly female with atrial fibrillation has significant sinus pauses consistent with a diagnosis of
tachy-brady syndrome. This is likely caused by flecainide, a Class IC agent, which blocks sodium
channels, resulting in bradycardia. Flecainide prolongs depolarization and slows conduction in the
atrioventricular (AV) node, and profound sinus bradycardia can be induced in patients with pre-
existing sinus node disease.

Her presentation is not consistent with aortic stenosis, as she does not have a significant elevated
velocity across the aortic valve. Nor does she have a significant left ventricular outflow tract gradient
to cause dynamic outflow tract obstruction.

Her history is not consistent with autonomic dysfunction in that her episode did not occur with

KKUH
Collected by:
Dr Hani Abdullah
Dr Fatma Hadi
Dr Salem Boresa
Dr Rami Elesali
Dr Naeif Almagal
1/1/2020
52

changes in position but while seated and preceded by palpitations, which is more consistent with
tachy-brady syndrome.

Chronotropic incompetence would not be expected to cause syncope at rest and, in addition, her
monitor demonstrated an appropriate resting heart rate.

34. The correct answer: A


Key Point
Age-related SND also may be associated with AF, the so-called “tachy-brady” syndrome. This may
be associated with prolonged pauses, which can be symptomatic after termination of AF. A common
scenario would be an elderly patient with persistent AF who develops a prolonged pause one
termination of AF. This can be exacerbated by drugs, especially those that block sodium channels
such as flecainide.

Question 35
35. A 60-year-old woman was admitted to the hospital for palpitations and lightheadedness, and was
referred to you for consultation. For the past 3 months, she has had spells about once a week in
which she suddenly feels very lightheaded, followed by a sensation of breathlessness and anxiety.
Twice in the past month she has fainted completely, one episode which was witnessed by her son.
Her past history includes obesity and fibromyalgia. Her medications are citalopram 20 mg daily and
pregabalin 50 mg 3 times daily. On physical examination, her blood pressure was 130/80 mm Hg,
heart rate was 70 bpm, and weight was 180 pounds (body mass index 32). Cardiovascular
examination revealed an irregular rhythm without murmur. Her abdomen was obese, but the
remainder of the examination was unremarkable. An electrocardiogram showed sinus rhythm at 80
bpm, with a borderline PR interval of 200 msec. An echocardiogram was performed the day prior
with normal findings. Laboratory studies were normal except for potassium of 3.5 mEq/dl. On
telemetry, strips were recorded during the physical examination (Figure 1).

Which of the following is the next step in management?


A. Pacemaker implant.
B. Continued observation.
C. Correct potassium.
D. Discontinue citalopram.
E. Discontinue pregabalin.

These strips show second-degree atrioventricular (AV) block with periods of 2:1 AV block. The
tracing likely indicates Mobitz type 2 block, which suggests that the site of block is below the AV
node (at the level of the His bundle), and therefore is more likely to progress to complete heart block.
She therefore qualifies for a pacemaker. The third strip may show Mobitz type 1 block with PR
prolongation prior to the blocked beat. Nevertheless, a pacemaker would still be indicated for all
types of symptomatic second-degree AV block. Premature ectopic (narrow) beats also are seen

KKUH
Collected by:
Dr Hani Abdullah
Dr Fatma Hadi
Dr Salem Boresa
Dr Rami Elesali
Dr Naeif Almagal
1/1/2020
53

(these do not affect the sinus rate, which is about 75 bpm). Her episodes of syncope are likely
related to her conduction disease. Discontinuing medications or supplementing potassium will not
treat her subnodal conduction disease.

35. The correct answer: A


Key Point
Implantation of a permanent pacemaker is indicated for patients with irreversible symptomatic
bradycardia due to SND or AV block.

Question 36
36. A 66-year-old man presented to the emergency department with palpitations and shortness of
breath. He had a history of persistent atrial fibrillation and underwent radiofrequency ablation last
year. He was previously on sotalol, which was stopped 6 months following his ablation because he
was in sinus rhythm. He is currently taking metoprolol succinate 100 mg daily, lisinopril 20 mg daily,
and apixaban 5 mg twice daily.

An electrocardiogram was obtained (Figure 1).

What is the most likely mechanism of his arrhythmia?

(Figure 1)

A. Delayed afterdepolarizations.
B. Early afterdepolarizations.
C. Non–isthmus-dependent macro re-entry tachycardia.
D. Rapid focal ectopic activity.
E. Re-entry circuit within the triangle of Koch.

KKUH
Collected by:
Dr Hani Abdullah
Dr Fatma Hadi
Dr Salem Boresa
Dr Rami Elesali
Dr Naeif Almagal
1/1/2020
54

Non–isthmus-dependent atrial flutter or atypical flutter describes macro re-entry atrial tachycardias
that are not dependent on conduction through the cavotricuspid isthmus. Non–isthmus-dependent
atrial flutters often occur in patients with atrial scarring from prior heart surgery or ablation, but may
also be idiopathic or occur in any form of heart disease. In this patient with a prior radiofrequency
ablation for atrial fibrillation, his arrhythmia is highly suspicious for an atypical atrial flutter originating
in the left atrium at the site of scar tissue. A surface EKG cannot reliably differentiate between typical
and atypical flutter in these patients. If the patient fails to respond to rate-controlling agents, then an
electrophysiology study with atrial mapping is indicated.

Rapid focal ectopic activity is the mechanism of atrial fibrillation.

Re-entry circuit within the triangle of Koch is the mechanism of atrioventricular nodal re-entry
tachycardia.

Early afterdepolarizations are the mechanism for torsades des pointes and polymorphic ventricular
tachycardia (VT).

Delayed afterdepolorizations can occur with acute myocardial infarction or digoxin toxicity, and may
manifest as bidirectional VT or catecholaminergic polymorphic VT.

36. The correct answer: C


Key Point
Non–isthmus-dependent atrial flutter involves macro–re-entrant circuits elsewhere in the RA or LA
and may occur in a variety of clinical settings, including congenital heart disease, after cardiac
surgery, and after catheter ablation of AF.

Question 37
37. Review the electrocardiogram (ECG) in Figure 1.

What is the correct diagnosis?

KKUH
Collected by:
Dr Hani Abdullah
Dr Fatma Hadi
Dr Salem Boresa
Dr Rami Elesali
Dr Naeif Almagal
1/1/2020
55

(Figure 1)
Reproduced with permission from https://www.healio.com/cardiology/learn-the-heart/ecg-review/ecg-
topic-reviews-and-criteria/. Accessed 05/29/2018.

A. Accelerated idioventricular rhythm.


B. Mobitz type I second-degree heart block.
C. Third-degree heart block.
D. Mobitz type II second-degree heart block.
E. First-degree heart block.

This ECG demonstrates sinus bradycardia with a narrow QRS (first two beats) with a wider
ventricular rhythm subsequently overtaking the sinus rate, consistent with accelerated idioventricular
rhythm (AIVR) at a rate of ~75 bpm. AV dissociation can occur with an accelerated idioventricular
rhythm (AIVR) or complete heart block. The two, however, can be distinguished by the atrial and
ventricular rates.

In complete heart block, the sinus rate is faster than the ventricular rate. In AIVR, the ventricular rate
is faster than the sinus rate. Because the ventricular rate is faster than the atrial rate, this is not
complete heart block but AIVR. As there is no relation between the P waves and QRS complexes,
this is not consistent with first- or second-degree heart block.

37. The correct answer: A


Key Point
AV block should be distinguished from AV dissociation, where the ventricular rate is faster that the
atrial rate. In AV block, the atrial rate is faster than the ventricular rate.

KKUH
Collected by:
Dr Hani Abdullah
Dr Fatma Hadi
Dr Salem Boresa
Dr Rami Elesali
Dr Naeif Almagal
1/1/2020
56

Question 38
38. A 30-year-old female has a sudden onset of palpitations and lightheadedness while playing with
her children in the park. After an hour, with no relief, she presented to the emergency department.
Her vital signs were blood pressure 120/70 mm Hg and heart rate 180 bpm. She was then given
adenosine 6 and 12 mg but without effect. Her electrocardiogram (ECG) was obtained (Figure 1).
She was offered cardioversion but refused.

Which of the following medications is most likely to chemically cardiovert this rhythm?

A. Ibutilide.
B. Verapamil.
C. Metoprolol.
D. Lidocaine.
E. Adenosine (high dose).

In this young patient without structural heart disease, one must first assess hemodynamic stability.
Because she has no chest pain, dyspnea, or hypotension, emergent therapy is not necessary. This
ECG shows a tachycardia with just a mildly wide QRS complex. There is a right bundle branch block
(RBBB)-like morphology. In a young person who is otherwise healthy, this ECG is most consistent
with idiopathic left ventricle (fascicular) ventricular tachycardia, which is a re-entrant tachycardia
involving most commonly the left posterior fascicle, giving an RBBB-like appearance and a superior
axis. This rhythm is highly sensitive to verapamil. It is a low-risk tachycardia and catheter ablation of
the fascicle involved is usually curative.

38. The correct answer: B


KKUH
Collected by:
Dr Hani Abdullah
Dr Fatma Hadi
Dr Salem Boresa
Dr Rami Elesali
Dr Naeif Almagal
1/1/2020
57

Key Point
Distinction of ventricular tachycardia in patients with or without structural heart disease is
importantsince therapeutic approaches are very different. In patients with structural heart disease,
an implantable cardioverter-defibrillator is generally recommended (secondary prevention); in
patients without structural heart disease, drug therapy or catheter ablation may be effective in
reducing or eliminating the arrhythmia.

Question 39
39. A 25-year-old female presented to the emergency department after a fainting spell. She
collapsed without warning and appeared to stop breathing for about 30 seconds, after which she
spontaneously regained consciousness. A similar event occurred 2 months prior after she raced up
three flights of stairs. She has no significant past medical history and her only medication is
azithromycin, which was started 3 days prior for acute bronchitis. On examination, she felt fatigued.
In the emergency department, she lost consciousness and a rhythm strip was recorded (Figure 1).

Which of the following best explains the mechanism for the initiation of this heart rhythm?

(Figure 1)

A. Re-entry due to loss of the epicardial action-potential dome in phase II.


B. Re-entry due to enhanced potassium efflux during phase III of the action potential.
C. Re-entry due to myocardial scar and unidirectional block.
D. Triggered activity due to early afterdepolarizations.

This patient has a prolonged QT interval with polymorphic ventricular tachycardia (torsades de
pointes) degenerating into ventricular fibrillation. The prolonged QT interval triggers early
afterdepolarizations. Azithromycin is known to increase the QT interval and may predispose a

KKUH
Collected by:
Dr Hani Abdullah
Dr Fatma Hadi
Dr Salem Boresa
Dr Rami Elesali
Dr Naeif Almagal
1/1/2020
58

patient with long QT syndrome to this event.

A young female with no prior history is unlikely to have scar-related ventricular tachycardia. Thus,
this is not the correct answer.

Acute myocardial ischemia causes re-entry due to loss of the epicardial action-potential dome in
phase II. This young and otherwise healthy patient is unlikely to have myocardial ischemia and thus
this is not the correct answer.

Enhanced potassium efflux during phase III of the action potential does not occur with azithromycin
and is therefore not the correct answer.

39. The correct answer: D


Key Point
EADs occur in the setting of a prolonged action potential (prolonged QT interval on the ECG) and
can result from drug exposure (e.g., class Ia and class III antiarrhythmic drugs, many
noncardiovascular drugs), gene mutations that cause LQTS, and clinical conditions such as marked
hypokalemia or acute ischemia. EADs can initiate monomorphic or polymorphic VT/VF.

Question 40
40. A 35-year-old male came to your clinic for his symptoms of palpitations. For the past 5 years, he
has felt an irregular beat with frequent ”skips.” He said that in the past 6 months his running capacity
has declined as well, and it takes longer for him to complete his usual 5-mile daily run. He denied
any syncope or chest pain. He has no past medical history. His family history is notable for an acute
myocardial infarction in his father at age 50 years. He takes no regular medications. On physical
examination, his blood pressure was 108/60 mm Hg with a pulse of 70 bpm. He weighed 170
pounds (body mass index was 25.8). The jugular venous pulse was seen at 6 cm and carotid
upstrokes were normal and without bruit. Lung sounds were clear and cardiac auscultation showed a
single S1, physiologically split S2, and a grade 1/6 short systolic murmur. An ectopic beat about every
three beats was heard on auscultation. An electrocardiogram (ECG) was obtained (Figure 1).

In addition to a transthoracic echocardiogram, what is the best next step?

KKUH
Collected by:
Dr Hani Abdullah
Dr Fatma Hadi
Dr Salem Boresa
Dr Rami Elesali
Dr Naeif Almagal
1/1/2020
59

A. Implantable loop recorder


B. Holter monitor
C. Coronary Computed Tomography
D. EP Study

Frequent premature ventricular contractions (PVCs) may result in significant exercise intolerance or
a PVC-related cardiomyopathy. Establishing the burden of PVCs is critical in determine the next
step, including the need for PVC ablation, which can be considered if the PVC burden is over 25%.
Implantable loop recorders are only indicated in patients for whom noninvasive ECG monitoring is
inconclusive. Coronary computed tomography (CT) angiogram would not be the first step in
assessment of ischemia in a low-risk patient. An electrophysiology study (EP) is not indicated
without other findings such as syncope or abnormalities on ambulatory monitoring. Signal-averaged
ECG was previously used for risk stratification of sudden cardiac death in patients with coronary
artery disease, reduced ejection fraction, or unexplained syncope, but is no longer routinely used in
clinical practice.

40. The correct answer: B


Key Point
In such patients, it is important to assess the burden of PVC (the best way to do this is with a 24-
hour Holter monitor) and to rule out any degree of ventricular dysfunction. A 12-lead ECG also is
useful if it shows the same PVC in most of the leads because this can help to determine the site of
the VT origin. A cardiac MRI should be considered in certain situations to help exclude other
underlying structural disease such as ARVD or incidental scarring from remote inflammation.

KKUH
Collected by:
Dr Hani Abdullah
Dr Fatma Hadi
Dr Salem Boresa
Dr Rami Elesali
Dr Naeif Almagal
1/1/2020
60

Question 41
41. A 52-year-old male with a history of paroxysmal atrial fibrillation presented to the emergency
department after an episode of syncope while he was cooking dinner in his kitchen. He reported he
felt lightheaded for a second before collapsing; his wife witnessed the episode, and he woke up and
felt fine 10 seconds later. In the emergency department, his heart rate was 80 bpm, his blood
pressure was 140/76 mm Hg, and he appeared well. His home medications include flecainide 100
mg twice daily and metoprolol 50 mg twice daily. His electrocardiogram (ECG) in the emergency
department (Figure 1) was compared with his prior ECG from last year (Figure 2).

Which of the following most likely explains his current ECG findings and syncope?

(Figure 1)

KKUH
Collected by:
Dr Hani Abdullah
Dr Fatma Hadi
Dr Salem Boresa
Dr Rami Elesali
Dr Naeif Almagal
1/1/2020
61

(Figure 2)
A. Excessive beta-blockers.
B. Ischemia.
C. Increased vagal tone.
D. Brugada syndrome.
E. Torsades de pointes.

This patient's ECG demonstrates type 1 Brugada pattern. Sodium channel blockers (flecainide,
propafenone) can exacerbate the transient ECG abnormalities that occur in patients with Brugada
syndrome who commonly have normal ECGs. Serial ECG testing may demonstrate the intermittent
changes, and use of class 1c agents during electrophysiologic testing can be used to unmask
Brugada pattern. This patient's ECG and history are most concerning for Brugada-induced
polymorphic ventricular tachycardia/ventricular fibrillation. Excessive beta-blockers and vagal tone
may result in syncope, particularly in the setting of a conversion pause from atrial fibrillation, but
these would not explain the ECG changes. There are no ECG findings of ischemia, and the QT
interval is normal. Torsades de pointes would be a concern if the patient were on a class III
antiarrhythmic drug (sotalol, dofetilide) and his QT were prolonged.

41. The correct answer: D


Key Point
BrS is an arrhythmogenic condition in patients with a structurally normal heart and a characteristic
ECG appearance of incomplete/complete RBBB pattern. Familial transmission is autosomal
dominant, but often there is incomplete penetrance.

KKUH
Collected by:
Dr Hani Abdullah
Dr Fatma Hadi
Dr Salem Boresa
Dr Rami Elesali
Dr Naeif Almagal
1/1/2020
62

Question 42
42. A 65-year-old man with a history of coronary artery disease with a remote history of
percutaneous coronary intervention (PCI) for angina symptoms, hypertension, hyperlipidemia, and
type 2 diabetes mellitus presented to your office for evaluation. He reported that he was in his usual
state of health until 2 days prior when he experienced a first syncopal episode while washing dishes.
He does not recall any symptoms prior to or after the event. He denies any chest pain, dyspnea, or
palpitations. He reported being compliant with his medications, which include aspirin, atorvastatin,
lisinopril, and metformin. On examination, his heart rate was 72 bpm and regular, and his blood
pressure was 132/78 mm Hg. His physical examination including neurologic assessment was
unremarkable. An electrocardiogram (ECG) demonstrates sinus rhythm and poor R wave
progression.

In addition to ambulatory ECG monitoring, which of the following is the best next step in this patient's
management?

A. Carotid ultrasound.
B. Electroencephalogram.
C. Echocardiogram.
D. Autonomic testing.
E. Tilt table testing.

Evaluation of left ventricular (LV) structure and function is the most appropriate next step. In patients
with unexplained syncope, arrhythmic causes are more common in patients with LV dysfunction. The
patient described a history concerning for cardiogenic syncope due to a malignant ventricular
arrhythmia, typically characterized as sudden onset in nature and associated with a rapid recovery.
Several factors favor a cardiac cause for syncope in this patient, including age >60 years, male sex,
and known ischemic heart disease. The absence of a prodrome makes reflex syncope less likely
and therefore tilt table testing less appropriate as an initial evaluation. Routine carotid ultrasound
and electroencephalogram (EEG) are not indicated in the evaluation of syncope in the absence of
focal neurologic findings (Class III). This patient's presentation is not consistent with neurogenic
orthostatic hypotension and therefore autonomic testing is not indicated.

42. The correct answer: C


Key Point
Cardiogenic syncope is characterized clinically with sudden loss of consciousness and rapid
recovery. Most cardiogenic syncope is related to bradycardia or a ventricular arrhythmia.

KKUH
Collected by:
Dr Hani Abdullah
Dr Fatma Hadi
Dr Salem Boresa
Dr Rami Elesali
Dr Naeif Almagal
1/1/2020
63

Question 43
43. A 30-year-old male with hypertrophic cardiomyopathy status post–dual-chamber implantable
cardioverter-defibrillator (ICD) for secondary prevention of sudden cardiac death presented to your
office for routine follow-up. He was on metoprolol succinate 50 mg daily. ICD interrogation disclosed
3 episodes of atrial flutter in the past 3 months, the longest lasting 2 hours. He has been
asymptomatic during these episodes.

What is the best next step?


A. Aspirin 325 mg daily.
B. Left atrial appendage occlusion device.
C. No change in medications.
D. Radiofrequency ablation.
E. Rivaroxaban 20 mg daily.

Patients with atrial flutter have the same risk of thromboembolism as patients with atrial fibrillation
(AF); therefore, recommendations for anticoagulation mirror those for patients with AF guided with
the CHADS2VASC score (2). In this patient with hypertrophic cardiomyopathy, the risk of stroke is
unrelated to the CHADS2VaSC score and is high enough that anticoagulation is recommended in all
patients with AF or atrial flutter and hypertrophic cardiomyopathy (Class I).

Catheter-based ablation for atrial arrhythmias is not indicated in an asymptomatic patient. Aspirin
offers inferior thrombotic prophylaxis and thus is not the preferred choice. Left atrial occlusion is only
indicated in patients who do not tolerate anticoagulation.

43. The correct answer: E


Key Point
Risk of stroke in patients with AFL is identical to risk in patients with AF and should be estimated
based on the CHA2DS2-VASc score, as is the case with AF.

Question 44
44. A 32-year-old man presented to your office for evaluation of syncope. One month ago, he
passed out while standing during church services on a hot summer day. He reports feeling
nauseated and diaphoretic at the time.
He has no past history and takes no medications. He has no significant family history of cardiac
disease or sudden death.

On examination, his heart rate was 84 bpm and blood pressure was 122/70 mm Hg. His lungs were
clear, heart regular with no murmurs, and extremities were without edema.
KKUH
Collected by:
Dr Hani Abdullah
Dr Fatma Hadi
Dr Salem Boresa
Dr Rami Elesali
Dr Naeif Almagal
1/1/2020
64

An electrocardiogram (ECG) was recorded (Figure 1).

What do you advise the patient?

(Figure 1)

A. Electrophysiology study to assess for ventricular arrhythmias.


B. Cardiac magnetic resonance imaging.
C. Physical counterpressure maneuvers.
D. Genetic testing.
E. Implantable cardioverter-defibrillator implantation.

This patient's ECG is consistent with a Brugada pattern. Brugada syndrome is a genetic disease
characterized by an increased risk of sudden cardiac death (SCD) and ST elevation with type 1
morphology =2 mm in at least 1 lead among the right precordial leads V1 and V2.

Syncope is a risk factor for cardiac arrhythmic events in patients with Brugada syndrome.
Implantable cardioverter-defibrillator (ICD) implantation is reasonable in these patients; however, the
benefit seems to be limited to patients with suspected arrhythmic syncope. Patients with syncope
consistent with a reflex-mediated mechanism should not undergo the implantation of an ICD.

This patient's history is consistent with classic vasovagal syncope. Given the lack of benefit of ICD
therapy in patients with reflex syncope and the known rate of inappropriate shocks and ICD
complications in patients who receive an ICD, ICD implantation is not recommended when the
syncope mechanism is believed to be reflex mediated. Thus, ICD is not the correct answer.

Management of vasovagal syncope focuses on preventive and abortive measures. Patients with a
syncope prodrome should be instructed to assume a supine position to prevent a faint and minimize

KKUH
Collected by:
Dr Hani Abdullah
Dr Fatma Hadi
Dr Salem Boresa
Dr Rami Elesali
Dr Naeif Almagal
1/1/2020
65

possible injury. In patients with a sufficiently long prodrome, physical counter-maneuvers (e.g., leg
crossing, limb and/or abdominal contraction, squatting) are a core management strategy. In a
randomized, parallel, open-label trial, leg crossing with conventional therapy (e.g., fluid, salt intake,
counseling, and avoidance) was superior to conventional therapy in preventing syncope recurrence.

The value of an electrophysiology study (EP) in assessing the mechanism of syncope or prognosis
in patients with Brugada is unknown. Therefore, EP may be considered only in patients with syncope
suspected to be due to an arrhythmia and is not recommended in patients with reflex syncope.

Genetic testing is not used to make the diagnosis of Brugada syndrome, nor does genetic testing
affect management. Thus, this is not the correct answer.

While cardiac magnetic resonance imaging (MRI) is helpful in some cardiac conditions associated
with arrhythmias, such as sarcoidosis or arrhythmogenic right ventricular cardiomyopathy, it does not
play a role in the diagnosis or management of Brugada syndrome.

44. The correct answer: C


Key Point
Sudden death can be the first manifestation of BrS, although most patients have a history of
recurrent syncope, particularly at rest.

Question 45
45. A 60-year-old female has a history of paroxysmal atrial fibrillation (AF) that has been
symptomatic enough to warrant treatment. She was started on sotalol 80 mg BID, which was titrated
up to 120 mg BID because of ongoing recurrences. She was started on hydrochlorothiazide 25 mg
QD 1 week ago for hypertension. She was brought by ambulance to the emergency department for
multiple episodes of lightheadedness and dizziness. Her blood pressure was 120/76 mm Hg and her
serum potassium was 3.2 mEq/L. Her electrocardiogram was obtained (Figure 1).

Which of the following should be the next step in her management?

KKUH
Collected by:
Dr Hani Abdullah
Dr Fatma Hadi
Dr Salem Boresa
Dr Rami Elesali
Dr Naeif Almagal
1/1/2020
66

(Figure 1)

A. Intravenous potassium and magnesium.


B. Electrical cardioversion.
C. Intravenous metoprolol.
D. Temporary transvenous pacing.
E. Intravenous amiodarone.

Sotalol is a class III antiarrhythmic agent used primarily for the treatment of AF. It blocks the inward
rectifying potassium channel, resulting in a prolongation of the QT interval.

Figure 1 shows nonsustained episodes of torsades de pointes (TdP), accounting for the patient’s
complaints of palpitations and dizziness. Her QTc is 570 msec (QT 400 msec/square root of the RR
interval of 600 msec). The risk of TdP with sotalol is <2%, but is higher in the setting of bradycardia,
female sex, pre-existing QT prolongation, history of heart failure, history of ventricular
tachycardia/ventricular fibrillation, or hypokalemia.

The treatment of sustained TdP in a hemodynamically unstable patient is prompt electrical


defibrillation. In a stable patient, intravenous (IV) magnesium is effective in both the treatment and
prevention of TdP. Temporary transvenous overdrive pacing to treat TdP usually is reserved for
patients not responding to IV magnesium.

Class III antiarrhythmic drugs such as sotalol have reverse-use dependence, such that the QT
lengthens as the heart rate (HR) slows, and the QT shortens as the HR increases. This helps
explain why temporary pacing and IV isoproterenol help treat TdP by increasing the HR and
decreasing the QT interval, whereas slowing the HR with beta-blockers would be contraindicated. IV
amiodarone, which also prolongs the QT interval, would be contraindicated. Therefore, the most
KKUH
Collected by:
Dr Hani Abdullah
Dr Fatma Hadi
Dr Salem Boresa
Dr Rami Elesali
Dr Naeif Almagal
1/1/2020
67

appropriate first steps in treating this patient would be treating her hypokalemia, infusing IV
magnesium and potassium, and discontinuing her sotalol.

45. The correct answer: A


Key Point
Proarrhythmic effect of class III drugs is due to excessive prolongation of the QT interval, leading to
polymorphic VT (torsades de pointes). Proarrhythmic effect of class III drugs is maximal at slower
heart rates.

Question 46
46. A 75-year-old male presented to your office with 2 episodes of near syncope over the past 2
months. The symptoms occurred while lying in bed prior to falling asleep and again while he was
driving to work. His past medical history is significant for hypertension. His medications include
aspirin and amlodipine. He denied exertional chest pain, shortness of breath, or palpitations. On
examination, his blood pressure was 145/80 mm Hg and heart rate was 49 bpm. His jugular venous
pressure was normal, there was no S3 or S4, his lungs were clear, and there was no peripheral
edema. The remainder of his examination was normal. His electrocardiogram (ECG) showed sinus
bradycardia with first-degree atrioventricular (AV) block and no ST or T-wave abnormalities.

Which of the following is the best next diagnostic study for this patient?
A. Cardiac catheterization.
B. Tilt table test.
C. Vasodilator single-photon emission computed tomography.
D. Thirty-day event monitor.
E. Transthoracic echocardiogram.

This older patient is having intermittent near syncope with resting bradycardia, raising the suspicion
of sinus node dysfunction. The best test to determine whether his symptoms are due to bradycardia
or sinus pauses would be an event monitor. Due to the infrequent nature of his episodes, a more
extended monitoring period would be needed than would be afforded by a Holter monitor. This
patient has no signs or symptoms of heart failure, so a transthoracic echocardiogram is less likely to
reveal the etiology of his symptoms. The patient does not have any symptoms suggestive of
ischemia, and thus neither a vasodilator single-photon emission computed tomography (SPECT) nor
cardiac catheterization are indicated. Finally, a tilt table test would not be the best next test, as the
history is not consistent with neutrally mediated presyncope.

46. The correct answer: D

KKUH
Collected by:
Dr Hani Abdullah
Dr Fatma Hadi
Dr Salem Boresa
Dr Rami Elesali
Dr Naeif Almagal
1/1/2020
68

Key Point
SND comprises a number of different manifestations including symptomatic bradycardia, sinus
pauses due to sinus arrest or sinoatrial exit block, and chronotropic incompetence.

Question 47
47.An 18-year-old female with no significant past medical history had been evaluated at a college
sports physical. Her physicial examination was normal. Although she had swum before, at the
second meet of the year, she experienced an episode of sudden cardiac death while swimming.

Which of the following mechanisms explains the likely pathophysiology of the sudden death?
A. Gain of function in the calcium channel.
B. Loss of function in the potassium channel.
C. Loss of function in the sodium channel.
D. Gain of function in the sodium channel.
E. Gain of function in the potassium channel.

A young adult with sudden cardiac death should raise the suspicion of an underlying primary
electrical disorder or hypertrophic cardiomyopathy. There are certain settings that are regarded as
classic for certain syndromes. Of these, long QT syndrome type 1, with a mutation in the IKs
potassium channel leading to a loss of function and thus a delay in membrane repolarization, has a
classic predilection for events with swimming.

47. The correct answer: B


Key Point
Avoidance of strenuous competitive sports is important (Class I).

Question 48
48. A 50-year-old male with a history of paroxysmal atrial fibrillation and coronary artery disease was
initiated on dofetilide. His medications include aspirin 81 mg daily, lisinopril 10 mg daily, simvastatin
40 mg daily, and metoprolol 50 mg twice daily. His heart rate was 55 bpm, blood pressure was
123/75 mm Hg, and the remainder of the examination was unremarkable. His electrocardiogram
(ECG) showed sinus bradycardia at 54 bpm with a nonspecific intraventricular conduction delay,
QRS duration of 105 msec, and QT interval of 430 msec. The laboratory panel showed normal renal
function and a serum potassium level of 5.6 mEq/L.

Which of the following effects of dofetilide may increase the risk for torsades de pointes in this
patient?

KKUH
Collected by:
Dr Hani Abdullah
Dr Fatma Hadi
Dr Salem Boresa
Dr Rami Elesali
Dr Naeif Almagal
1/1/2020
69

A. Decreased potassium efflux exacerbated by hyperkalemia.


B. Increased potassium efflux exacerbated by hyperkalemia.
C. Reverse-use dependence that is exacerbated by bradycardia.
D. Use dependence that is exacerbated by bradycardia.
E. Decreased drug metabolism in the presence of simvastatin.

Dofetilide exhibits reverse-use dependence, with an increased effect at slow heart rates.
Bradycardia in this patient may exacerbate the QT-prolonging effects of dofetilide. Dofetilide does
not have a drug interaction with simvastatin. Dofetilide has many drug interactions, especially those
that prolong the QTc, cause hypokalemia, or cause hypomagnesemia. Verapamil also alters
dofetilide metabolism and increases toxicity.

48. The correct answer: C


Key Point
Class III drugs block outward potassium channel activity (IKs) and prolong refractoriness. These
agents exhibit "reverse use-dependence" in which a greater degree of channel block is seen at
slower heart rates.

Question 49
49. A 50-year-old female with rheumatoid arthritis presented to the emergency department with
syncope. Her current medications are prednisone 7.5 mg daily, methotrexate 15 mg weekly,
hydroxychloroquine 200 mg daily, dapsone 50 mg daily, and nystatin swish and spit daily. On
examination, her heart rate was 84 bpm, blood pressure was 130/80 mm Hg without orthostatic
change, and respiration was 14 breaths per minute. She appeared anxious. Other than swelling of
her hand joints, her examination was normal. An electrocardiogram (ECG) was obtained (Figure 1).

Which medication is most likely responsible for her presentation?

KKUH
Collected by:
Dr Hani Abdullah
Dr Fatma Hadi
Dr Salem Boresa
Dr Rami Elesali
Dr Naeif Almagal
1/1/2020
70

(Figure 1)

A. Prednisone.
B. Nystatin.
C. Methotrexate.
D. Dapsone.

This patient's ECG demonstrates prolonged QTc. Acquired long QT syndrome may be caused by
a number of factors including electrolyte disturbances and medications. A comprehensive list of
drugs that can prolong the QT interval may be found at www.crediblemeds.org.
Hydroxychloroquine contributes to QT interval prolongation. The other medications she is taking
do not significantly affect the QT interval.

49. The correct answer: E

Key Point
Ventricular arrhythmia may be either monomorphic with a single QRS morphology or polymorphic
with changing QRS morphology. The most common cause of polymorphic ventricular tachycardia
(PVT) is acute myocardial ischemia, which may quickly progress to ventricular fibrillation (VF).
Less commonly, PVT is caused by circumstances that prolong the Q-T interval (drugs and
ion-channel disorders). It is important to recognize which common drugs are associated
with Q-T prolongation and discontinue.

Question 50

50. A 52-year-old male came to the emergency department for mild palpitations that started the prior
evening. Palpitations persisted when he awoke the following morning, and therefore he drove himself
to the hospital. He has had 3 prior episodes over the past 2 months, each lasting only 5-10 minutes.
He has had a prior myocardial infarction with a single angioplasty and stent to the left circumflex artery
and an overall left ventricular ejection fraction of 45%. He takes aspirin 81 mg, clopidogrel 75 mg,
ramipril 5 mg daily, bisoprolol 5 mg, and atorvastatin 40 mg daily. He was somewhat anxious
appearing, although not in distress. His blood pressure in the emergency department was
90/65 mm Hg and heart rate was 135 bpm. The lungs were clear with no peripheral edema.
The presenting electrocardiogram (ECG) was obtained (Figure 1).

Which of the following is the best next step in management?

KKUH
Collected by:
Dr Hani Abdullah
Dr Fatma Hadi
Dr Salem Boresa
Dr Rami Elesali
Dr Naeif Almagal
1/1/2020
71

A. Administer intravenous ibutilide.


B. Administer intravenous adenosine.
C. Perform carotid sinus massage.
D. Perform synchronized cardioversion.
E. Administer intravenous metoprolol.

The ECG shows wide complex tachycardia with atrioventricular dissociation diagnostic of ventricular
tachycardia (VT). The patient is hemodynamically stable, although somewhat hypotensive and
anxious. The prior infarct and VT morphology is suggestive of a scar-based VT. Cardioversion is the
best option for this patient (although amiodarone or procainamide may be considered as well).
Ibutilide is appropriate for atrial fibrillation or atrial flutter, whereas carotid sinus massage or
adenosine would be appropriate for supraventricular tachycardia. Metoprolol also would be
ineffective and potentially harmful given the patient's hypotensive state.

50. The correct answer: D


Key Point
Ventricular arrhythmia may be either monomorphic with a single QRS morphology or polymorphic
with changing QRS morphology. The most common cause of polymorphic ventricular tachycardia
(PVT) is acute myocardial ischemia, which may quickly progress to ventricular fibrillation (VF). Less
commonly, PVT is caused by circumstances that prolong the Q-T interval (drugs and ion-channel
disorders). It is important to recognize which common drugs are associated with Q-T prolongation
and discontinue.

Question 51
51. A 45-year-old male saw you in consultation in the office regarding frequent palpitations. He
described pounding heart beats, 1 or 2 per hour throughout the day, which increase during times of
stress. An echocardiogram was normal. An exercise stress test had to be stopped due to increasing
ventricular ectopy, including a five-beat run of ventricular tachycardia that terminated as soon as

KKUH
Collected by:
Dr Hani Abdullah
Dr Fatma Hadi
Dr Salem Boresa
Dr Rami Elesali
Dr Naeif Almagal
1/1/2020
72

exercise stopped; there were no other electrocardiographic (ECG) changes during the stress test. A
12-lead ECG was obtained (Figure 1).

Which of the following is the most likely origin of the premature ventricular contractions (PVCs)?

A. Left ventricular outflow tract.


B. Body of the left ventricle.
C. Right ventricular outflow tract origin.
D. Right ventricle apical morphology.

This patient's palpitations are due to idiopathic ventricular outflow tract PVCs. These are commonly
associated with nonsustained and sustained ventricular arrhythmias during stress or during 24-hour
ECG (Holter) monitoring. Possible origins of this arrhythmia are in the right ventricular outflow tract
(RVOT; more common) or left ventricular outflow tract (LVOT; less common). The PVCs in the
ECG example have a left bundle, inferior-axis morphology. The precordial R-wave transition (totally
positive QRS complexes in lead V3) is earlier than is typically seen in RVOT ventricular tachycardia.
The LVOT (sinuses of Valsalva specifically) is posterior to the RVOT, which results in more positive
voltage and earlier transition in the precordial leads. When idiopathic outflow tract PVCs are more
frequent than 10% of total heart beats, they may be associated with development of
cardiomyopathy. Beta-blockers or nondihydropyridine calcium channel blockers are often effective,
with ablation reserved for patients who are refractory to medical therapy or with cardiomyopathy due
to the high-density arrhythmia.

51. The correct answer: A


Key Point
RVOT VT is important to recognize, as this is a focal VT amenable to curative catheter ablation. This
form of VT has a very characteristic ECG signature (LBBB with inferior axis and late R-wave
progression, beyond V3). Catheter ablation is highly successful in this condition. The VT is NOT
ischemia driven, and therefore, ischemic workup is not required or necessary unless there are other
symptoms such as exertional chest pain with risk factors that make CAD likely.

KKUH
Collected by:
Dr Hani Abdullah
Dr Fatma Hadi
Dr Salem Boresa
Dr Rami Elesali
Dr Naeif Almagal
1/1/2020
73

Question 52
52. A 52-year-old male presented to his cardiologist's office for routine follow-up. He has a history of
ventricular tachycardia and long QT syndrome (LQTS) with a mutation found in the KCNQ1 gene.
He had an implantable cardioverter-defibrillator (ICD) placed 2 years prior. He had been seen by his
primary care clinician the previous day for an upper respiratory infection and was started on
levofloxacin. His other medications include metoprolol succinate, atorvastatin, and aspirin. His
physical examination was unremarkable. His electrocardiogram (ECG) showed normal sinus rhythm
with a QTc of 435 msec.

Which of the following is the best next step in the management of this patient?
A. Increase metoprolol.
B. Stop atorvastatin.
C. Initiate amiodarone.
D. Discontinue levofloxacin.
E. Continue current management.

This patient is genotype positive for LQTS and has a history of ventricular tachycardia. Although his
QTc is currently in the normal range, QT-prolonging medications should still be avoided.
Levofloxacin is known to prolong QT, so discontinuing this medication and using a non-QT
prolonging antibiotic is most appropriate. Amiodarone is not recommended in the treatment of LQTS.
Atorvastatin does not prolong the QT and thus is not the correct answer. Metoprolol succinate is a
first-line treatment for LQTS. While increasing metoprolol may reduce the risk of ventricular
arrhythmias, the correct response here is to discontinue the offending medication.

52. The correct answer: D


Key Point
All genotype-positive/phenotype-negative subjects should avoid QT-prolonging drugs and correct
any electrolyte abnormalities immediately.

Question 53
53. A 63-year-old man came to see you in your clinic. He has a past medical history of coronary
artery disease for which he underwent percutaneous coronary intervention to the mid left anterior
descending and mid right coronary artery 3 years ago. A recent echocardiogram showed a dilated
left ventricle with a left ventricle ejection fraction of 26%. He tries to walk 3 times a week but feels
shortness of breath when trying to walk up a flight of stairs. He is on optimal medical therapy,
including carvedilol and lisinopril. His electrocardiogram showed normal sinus rhythm and left bundle
branch block with a QRS width of 160 msec.

KKUH
Collected by:
Dr Hani Abdullah
Dr Fatma Hadi
Dr Salem Boresa
Dr Rami Elesali
Dr Naeif Almagal
1/1/2020
74

At this point, which of the following is the next best step in his management?
A. Implant a single-chamber implantable cardioverter-defibrillator.
B. Continue current medical therapy.
C. Implant a dual-chamber implantable cardioverter-defibrillator.
D. Implant a biventricular implantable cardioverter-defibrillator.

The correct answer is to implant a biventricular implantable cardioverter-defibrillator (ICD) to


decrease the risk of death or heart failure hospitalization based on the results from MADIT-CRT
(Multicenter Automatic Defibrillator Implantation Trial With Cardiac Resynchronization Therapy) and
RAFT (Resynchronization/Defibrillation for Ambulatory Heart Failure Trial), which demonstrated that
a biventricular ICD in a patient with New York Heart Association class II reduced the composite of
death and heart failure events. In the RAFT trial, the secondary endpoint of mortality also was
achieved with a biventricular ICD compared with ICD alone. Continuing current medical therapy is
incorrect, as the patient meets criteria at this time for a device implant. In the 2012 update to the
2008 American College of Cardiology Foundation/American Heart Association/Heart Rhythm Society
device guidelines, it is now a Class I indication to implant a biventricular ICD in a patient with a left
ventricle ejection fraction of 35%, in sinus rhythm, in New York Heart Association class II, III, or
ambulatory IV, with left bundle branch block and QRS duration >150 msec.

53. The correct answer: D


Key Point
About two thirds of patients respond to CRT with symptomatic improvement. Pre-implant causes of
CRT non-response include non-LBBB native conduction, or a relatively narrow native QRS.

Question54
54. The patient is a 67-year-old male with a history of coronary artery disease for which he
underwent coronary artery bypass graft surgery 5 years ago. A recent echocardiogram showed a
mildly enlarged left ventricle with a left ventricular ejection fraction of 25%. He is able to exercise 4
times a week for 1 hour and also hikes for a couple of hours every weekend. He is on optimal
medical therapy with carvedilol, lisinopril, aspirin, and atorvastatin. His electrocardiogram showed a
normal sinus rhythm with a right bundle branch block pattern and QRS duration of 138 msec.

Which of the following is the most appropriate recommendation for this patient?
A. Continue current medical therapy.
B. Implant an implantable cardioverter-defibrillator.
C. Implant a biventricular implantable cardioverter-defibrillator.
D. Implant a biventricular pacemaker.
E. Initiate sotalol therapy.

KKUH
Collected by:
Dr Hani Abdullah
Dr Fatma Hadi
Dr Salem Boresa
Dr Rami Elesali
Dr Naeif Almagal
1/1/2020
75

The patient meets criteria for a primary prevention implantable cardioverter-defibrillator (ICD) based
on MADIT-II (Multicenter Automatic Defibrillator Implantation Trial II) clinical trial results. Continuing
current medical therapy and initiating sotalol therapy are incorrect, as the patient meets criteria for
device implant and antiarrhythmic therapy is not demonstrated to improve survival. Implanting a
biventricular ICD and implanting a biventricular pacemaker are also incorrect, as the guidelines state
it is a Class III indication for biventricular pacing cardiac resynchronization therapy in patients with
New York Heart Association class I or II and a non-left bundle branch block pattern with QRS
duration of <150 ms.

54. The correct answer: B

Question 55
55. A 32-year-old male presented to the clinic for evaluation of "heart flutters" and lightheadedness.
These symptoms have become more frequent over the last 6 months and are mainly noticeable at
rest or as he is cooling down from exercise. He was able to run 2 miles 3 days a week without
limitations. He would prefer to avoid chronic medical therapy. He has no other medical conditions.
His family history includes myocardial infarction in his father at age 63. His physical examination was
normal. An electrocardiogram (ECG) demonstrated multiple premature ventricular complexes
(PVCs) with a left bundle branch block (LBBB) morphology and inferior axis. A transthoracic
echocardiogram is unremarkable.

A 24-hour Holter monitor revealed 9,240 PVCs as well as multiple runs of nonsustained ventricular
tachycardia (VT).
Which of the following would be the best next step?

A. Exercise stress test.


B. Metoprolol.
C. Cardiac catheterization.
D. Catheter ablation.
E. Propafenone.

This patient's presentation and ECG are consistent with right ventricular outflow tract (RVOT) VT,
which may be seen in individuals with structurally normal hearts. In patients with symptomatic RVOT
VT, beta-blockers or calcium channel blockers may be useful. Catheter ablation has been shown to
be more effective for suppression of RVOT PVCs than antiarrhythmic drugs or beta-blockers, and
may be useful in patients with a preference for avoiding chronic daily therapy. This patient has no
symptoms of ischemic heart disease and his symptoms are not prompted by exertion, so neither a
cardiac catheterization nor exercise stress test are indicated.

55. The correct answer: D


Key Point
RVOT VT is important to recognize, as this is a focal VT amenable to curative catheter ablation. This

KKUH
Collected by:
Dr Hani Abdullah
Dr Fatma Hadi
Dr Salem Boresa
Dr Rami Elesali
Dr Naeif Almagal
1/1/2020
76

form of VT has a very characteristic ECG signature (LBBB with inferior axis and late R-wave
progression, beyond V3). Catheter ablation is highly successful in this condition. The VT is NOT
ischemia driven, and therefore, ischemic workup is not required or necessary unless there are other
symptoms such as exertional chest pain with risk factors that make CAD likely.

Question 56
56. A 75-year-old male presented to the emergency department with palpitations. He has a history of
hypertension and symptomatic bradycardia for which he underwent dual-chamber pacemaker
implantation 2 years prior. His pacemaker has been programmed to the DDD mode 50-120 bpm. An
electrocardiogram (ECG) was obtained (Figure 1).

An intervention was performed, with resolution of the patient's symptoms. An ECG was obtained
after the intervention (Figure 2).

Based on the ECG shown in Figure 2, which of the following interventions was performed?

KKUH
Collected by:
Dr Hani Abdullah
Dr Fatma Hadi
Dr Salem Boresa
Dr Rami Elesali
Dr Naeif Almagal
1/1/2020
77

A. Administering adenosine.
B. Increasing the atrial sensitivity.
C. Programming to VVI mode.
D. Increasing the ventricular pacing output.
E. Programming hysteresis off.

The first tracing (Figure 1) demonstrates ventricular pacing at 110 bpm, consistent with tracking of
either a supraventricular tachycardia or pacemaker-mediated tachycardia. In the second tracing
(Figure 2), the rate has been decreased to 50 bpm with an underlying atrial tachycardia at 110 bpm.
There is no relation between the P waves and paced QRS morphology, consistent with
reprogramming the device to VVI mode, the correct answer.

Adenosine causes transient atrioventricular (AV) block, which would not affect pacemaker function,
and therefore would not resolve the rhythm.

Increasing the ventricular pacer output is not indicated because the pacemaker has adequate
ventricular capture.

Increasing the atrial sensitivity will not resolve the rhythm because there is no evidence of failure to
track atrial rhythms.

Hysteresis promotes intrinsic activity lower than the normal rate by allowing slow but appropriate
intrinsic rhythms. Changing the hysteresis parameters will not help resolve the tachycardia.

56. The correct answer: C

KKUH
Collected by:
Dr Hani Abdullah
Dr Fatma Hadi
Dr Salem Boresa
Dr Rami Elesali
Dr Naeif Almagal
1/1/2020
78

Key Point
Age-related SND also may be associated with AF, the so-called “tachy-brady” syndrome. This may
be associated with prolonged pauses, which can be symptomatic after termination of AF. A common
scenario would be an elderly patient with persistent AF who develops a prolonged pause one
termination of AF. This can be exacerbated by drugs, especially those that block sodium channels
such as flecainide.

Question 57
57. A 29-year-old male with Brugada syndrome underwent ambulatory electrocardiogram (ECG)
monitor for complaints of palpitations. He was found to have several episodes of sustained
ventricular tachycardia (VT). Two episodes occurred the day prior; one was associated with
syncope. He was admitted for implantable cardioverter-defibrillator (ICD) implantation. On postop
day #1, he had 3 ICD shocks. His ICD was interrogated and shocks deemed appropriate for
sustained VT. He was counseled about use of antipyretic medications in the setting of fever,
avoidance of drugs that may induce or aggravate ST-segment elevation, and avoidance of excess
alcohol.

The addition of which of the following is most appropriate in his care?


A. Procainamide.
B. Flecainide.
C. Quinidine.
D. Amiodarone.
E. Propranolol.

Quinidine, a Class Ia antiarrhythmic drug with Ito and IKr blocker effects, prevents induction of
ventricular fibrillation (VF) and suppress spontaneous ventricular arrhythmias in patients with
Brugada syndrome. Quinidine is currently being used in (1) patients with ICD and multiple shocks;
(2) cases in which ICD implantation is contraindicated; or (3) for the treatment of supraventricular
arrhythmias. In this patient, the use of quinidine would be indicated to suppress ventricular
arrhythmias and avoid future ICD shocks (Figure 1). Procainamide and flecainide (sodium channel
blocking agents) can be used to make the diagnosis of Brugada syndrome. Brugada syndrome can
be diagnosed by type I ST-segment elevation occuring spontaneously or after the administration of
these agents. Amiodarone has also been reported to unmask ST-segment elevation and has been
reported to be proarrhythmic in patients with Brugada syndrome. In patients with Brugada syndrome,
beta-blockers can increase ST-segment elevation and are therefore contraindicated.

KKUH
Collected by:
Dr Hani Abdullah
Dr Fatma Hadi
Dr Salem Boresa
Dr Rami Elesali
Dr Naeif Almagal
1/1/2020
79

(Figure 1)

57. The correct answer: C


Key Point
Quinidine may be effective to prevent recurrent arrhythmias in BrS patients with an ICD.

Question 58
58. A 47-year-old female with a history of tobacco use, hypertension, and obesity presented to the
emergency department with sudden onset of palpitations associated with dyspnea and neck
discomfort. On examination, she appeared mildly uncomfortable and vital signs included heart rate
of 170 bpm and blood pressure of 138/66 mm Hg. A 12-lead electrocardiogram (ECG) was obtained,
revealing a regular narrow complex tachycardia with inferolateral ST-T changes. Vagal maneuvers
were performed without effect, so the patient subsequently received 6 mg intravenous (IV)
adenosine with conversion to sinus rhythm and improvement in her symptoms. A repeat ECG was
normal.
Which of the following is the most likely diagnosis?

KKUH
Collected by:
Dr Hani Abdullah
Dr Fatma Hadi
Dr Salem Boresa
Dr Rami Elesali
Dr Naeif Almagal
1/1/2020
80

A. Junctional tachycardia.
B. Atrioventricular nodal re-entry tachycardia.
C. Atrial tachycardia.
D. Atrial flutter.
E. Atrioventricular re-entry tachycardia.

This patient presented with supraventricular tachycardia (SVT). In patients with a normal baseline
12-lead ECG (no pre-excitation or delta wave) such as observed in this patient, the most common
mechanism of a regular narrow complex tachycardia is atrioventricular nodal re-entry tachycardia
(AVNRT). The neck discomfort she reported may be due to cannon A waves, related to pulsatile
reverse flow from right atrial contraction against a closed tricuspid valve. The next most frequent
narrow complex SVT is atrioventricular re-entry tachycardia (AVRT), followed by atrial tachycardia,
which accounts for <10% of cases. Junctional tachycardia is even less common in adults. Finally,
the absence of flutter waves on the ECG makes a diagnosis of atrial flutter unlikely.

58. The correct answer: B


Key Point
In patients with a normal 12-lead ECG (no pre-excitation or delta wave), the most common
mechanism of narrow complex tachycardia is AVNRT followed by AVRT. AT is the least common
form of SVT, accounting for <10% of cases.

Question 59
59.An 80-year-old female with hypertension and diabetes mellitus presented to your clinic with
exertional dyspnea and fatigue. Her current medications included lisinopril 10 mg daily and glyburide
5 mg daily. Her family history was significant for coronary artery disease and hypertension. On
examination, her blood pressure was 145/80 mm Hg with heart rate of 50 bpm. A cardiovascular
examination revealed II/VI early peaking ejection systolic murmur and the reminder of the
examination was unremarkable. She underwent a nuclear stress imaging study on a modified Bruce
protocol; she exercised for 6 minutes and stopped secondary to fatigue. Her maximum blood
pressure and heart rate were 180/90 mm Hg and 96 bpm, respectively. There were no ischemic
electrocardiogram changes and her perfusion images showed no evidence of stress-induced
ischemia.

What is the most likely cause of her symptoms?

A. Deconditioning.
B. Chronotropic incompetence.
C. Aortic stenosis.
D. Hypertension.
E. Microvascular dysfunction.

KKUH
Collected by:
Dr Hani Abdullah
Dr Fatma Hadi
Dr Salem Boresa
Dr Rami Elesali
Dr Naeif Almagal
1/1/2020
81

Sinus node dysfunction (SND) refers to a broad array of abnormalities in sinus node and atrial
impulse formation and propagation. These include persistent sinus bradycardia and chronotropic
incompetence without identifiable causes, paroxysmal or persistent sinus arrest with replacement by
subsidiary escape rhythms in the atrium, atrioventricular (AV) junction, or ventricular myocardium.
This patient's symptoms are related to SND with chronotropic incompetence as evidenced by her
heart rate response during exercise, with a peak heart rate <100 bpm and <70% age-predicted
maximum heart rate.
Although she may also be deconditioned, the blunted heart rate increase in response to exercise
suggests that deconditioning is not her primary problem. Deconditioned patients should have an
exaggerated heart rate response to low levels of exercise.

Although she has a murmur, it was early peaking and her blood pressure response to exercise was
normal, so significant aortic stenosis is unlikely.
Her blood pressure response was normal, not exaggerated, and so would not explain her symptoms.
The normal perfusion study does not suggest ischemia as a factor in her symptoms.

59. The correct answer: B


Key Point
SND comprises a number of different manifestations including symptomatic bradycardia, sinus
pauses due to sinus arrest or sinoatrial exit block, and chronotropic incompetence.

Question 60
60. An 18-year-old woman was referred for evaluation for a family history of long QT syndrome
(LQTS). She felt well and denied any history of fainting, although she noted that her heart pounds
with exercise. She has no past medical history and does not smoke or drink alcohol. Her uncle was
resuscitated from a cardiac arrest and subsequently diagnosed with LQTS based on his postcardiac
arrest electrocardiogram (ECG). On physical examination, her blood pressure was 110/60 mm Hg
with a heart rate of 70 bpm, and the remainder of her examination was normal. She brought in a
copy of an ECG done by her family clinician 2 weeks prior, which showed sinus rhythm at 75 bpm
with a QTc of 460 msec. An ECG in your office today had similar findings, with a QTc of 450 msec.

Which of the following is the next step in management?


A. Stress test.
B. Implantable cardioverter-defibrillator implant.
C. Repeat electrocardiogram standing.
D. Electrophysiologic study.
E. Genetic testing of the Uncle.

This patient has a family history of probable LQTS in her uncle. Her ECG shows borderline QT
prolongation. Genetic testing, starting with her uncle, should be performed. Initiation of beta-blockers
would be reasonable if a diagnosis of LQTS can be made in the 18-year-old patient. There is no role
for electrophysiologic testing and implantable cardioverter-defibrillator (ICD) implant carries a Class
IIb indication only in the presence of strong risk factors for sudden cardiac death, which this patient
KKUH
Collected by:
Dr Hani Abdullah
Dr Fatma Hadi
Dr Salem Boresa
Dr Rami Elesali
Dr Naeif Almagal
1/1/2020
82

has not demonstrated. Stress testing as a provocative test for QT prolongation may be considered in
uncertain cases. However, the clinical use of this test requires more extensive validation and would
not be the best choice.

60. The correct answer: E


Key Point
The success of genetic testing in LQTS depends in large part on the robustness of the clinical
diagnosis with mutations identified in 70-80% of cases when the diagnosis is certain (QTc >480
msec), but dropping significantly in borderline cases.

Question 61
61. A 30-year-old male presented to your office for evaluation of syncope. He has fainted twice in the
past year, both during exercise. He has no other medical problems and takes no medications. His
family history includes a paternal uncle who died at age 34 in a drowning accident. On examination,
his heart rate was 70 bpm, respirations were 12 breaths per minute, and blood pressure was 108/72
mm Hg. His heart and lung sounds were normal. An electrocardiogram (ECG) was obtained (Figure
1).

Which of the following is the best next step in his care?

(Figure 1)

A. Cardiac magnetic resonance imaging.


B. Reassurance.
C. Genetic testing.
D. Provocative testing with intravenous isoproterenol.
E. Electrocardiogram with high precordial leads.

KKUH
Collected by:
Dr Hani Abdullah
Dr Fatma Hadi
Dr Salem Boresa
Dr Rami Elesali
Dr Naeif Almagal
1/1/2020
83

Brugada syndrome is an inherited arrhythmia syndrome characterized by ST elevation in right-sided


precordial leads. Type 1 ST-segment changes, "coved," are diagnostic of the syndrome whereas
type 2 "saddle back" ST changes are equivocal, as seen in this ECG (Figure 2). To make a definitive
diagnosis, type 1 ST-segment elevations must be observed following superior placement of
precordial leads (V1 and V2) in the second intercostal space or after administration of sodium
channel blocking drugs.

This patient has a history of syncope, but the ECG is not diagnostic (i.e., type 2 Brugada pattern)
and so the next step is to perform an ECG with superior placement of leads V1 and V2.

Isoproterenol challenge would be useful for diagnosing ventricular arrhythmias in the setting of
arrhythmogenic right ventricular cardiomyopathy or as an adjunct to tilt table testing for syncope.

Reassurance would be inappropriate for this patient with equivocal ECG findings and a concerning
family history.
Genetic testing and cardiac magnetic resonance imaging (MRI) are not useful in the diagnosis of
Brugada syndrome, which is suggested by the ECG.

(Figure 2)
Precordial leads of a resuscitated patient with Brugada syndrome (BrS) showing all three
echocardiogram (ECG) patterns and dynamic changes over an 8-day period. Arrows indicate J
waves.

61. The correct answer: E

KKUH
Collected by:
Dr Hani Abdullah
Dr Fatma Hadi
Dr Salem Boresa
Dr Rami Elesali
Dr Naeif Almagal
1/1/2020
84

Key Point
Use of a sodium channel-blocking agent (flecainide or ajmaline) can “unmask” BrS and can assist in
the diagnosis of borderline cases. Recording leads V1 and V2 in second and third intercostal space
can also assist in the diagnosis.

Question 62
62. A 20-year-old man presented to the emergency department with palpitations for the past 2 hours.
He has no past medical history. He takes no medications. On arrival, his blood pressure was 108/66
mm Hg with a heart rate of 190 bpm. An electrocardiogram was obtained (Figure 1).

What is the best next step in the management of this patient?

(Figure 1)

A. Ibutilide 1 mg intravenously.
B. Synchronized cardioversion.
C. Digoxin 0.5 mg intravenously.
D. Metoprolol 5mg intravenously.
E. Adenosine 6 mg intravenously.

The patient's resting rhythm reveals a Wolff-Parkinson-White (WPW) pattern and was found on
presentation to be in atrial fibrillation (AF) with rapid ventricular rate, but hemodynamically stable.
Small observational studies support the use of ibutilide or intravenous procainamide for the
treatment of pre-excited AF in patients who are not hemodynamically compromised. Both
medications can decrease ventricular rate by slowing conduction over the accessory pathway and
have the additional benefit of possibly terminating AF.

KKUH
Collected by:
Dr Hani Abdullah
Dr Fatma Hadi
Dr Salem Boresa
Dr Rami Elesali
Dr Naeif Almagal
1/1/2020
85

Prompt direct-current cardioversion is recommended for patients with AF, WPW syndrome, and
rapid ventricular response who are hemodynamically compromised, which is not the situation. While
cardioversion could be considered, it would not be the first option, in an awake and
hemodynamically stable patient.

Administration of intravenous beta-blocker, adenosine, or digoxin (oral or intravenous) in patients


with WPW syndrome who have pre-excited AF is potentially harmful because these drugs accelerate
the ventricular rate.

62. The correct answer: A

Question 3
63. A 56-year-old female was referred for evaluation of intermittent dizziness with occasional near-
syncope. Episodes occur 1-2 times a month, do not have any specific triggers, and last <1 minute.
She has not had a syncopal event, but is often presyncopal. She had no other cardiac symptoms
and was otherwise healthy. Her family history is notable for hypertension and atrial fibrillation in her
mother, and her grandmother had a pacemaker.

Which of the following is the best next diagnostic test for this patient?

A. Patient-activated event monitor.


B. Tilt table test.
C. A 24-hour continuous electrocardiogram (Holter).
D. Implanted loop recorder.
E. Electrophysiologic study.

KKUH
Collected by:
Dr Hani Abdullah
Dr Fatma Hadi
Dr Salem Boresa
Dr Rami Elesali
Dr Naeif Almagal
1/1/2020
86

A patient-activated event monitor can record electrocardiogram (ECG) tracings immediately prior to
and during symptoms, allowing identification of possible arrhythmias. A 24-hour continuous ECG
(e.g., Holter) is not correct: Episodes are likely to be too infrequent to be captured on a 24-hour
Holter. An implanted loop recorder may be considered when noninvasive diagnostic testing is
unrevealing, if symptom frequency is too low to capture on event monitor, or if the patient's symptom
is syncope. Although electrophysiologic study (EPS) allows direct assessment of the cardiac
conduction system, it does not allow correlation of symptoms with an arrhythmia. Tilt table testing
can evaluate for orthostatic symptoms or postural orthostatic tachycardia syndrome, but is not
appropriate for identifying other arrhythmias.

63. The correct answer: A


Key Point
When evaluating patients with lesser degrees of bradycardia, symptom-rhythm correlation is
important. This can be achieved with ambulatory monitoring (Holter or similar). More prolonged
monitoring with an implantable subcutaneous monitor may be needed in some cases.

Question 64
64. A 32-year-old woman presents to the emergency department with palpitations and progressive
shortness of breath for the past two days.

She has no medical history. Her medications include ibuprofen as needed for headaches and oral
contraceptive pills.

On examination, her temperature is 99.4°F, heart rate is 134 bpm and irregular, blood pressure was
90/60 mm Hg, respiratory rate is 20 breaths/min, and oxygen saturation 90% on room air. Her
jugular venous pressure (JVP) is 14 cm H2O. Her lungs are clear, heart is s irregular with a soft
systolic murmur at the left sternal border, abdomen is soft, and extremities are without edema. An
electrocardiogram (ECG) is recorded (Figure 1). A chest X-ray (CXR) shows no infiltrate, effusion, or
edema.

Which test is most likely to elucidate the cause of her symptoms?

KKUH
Collected by:
Dr Hani Abdullah
Dr Fatma Hadi
Dr Salem Boresa
Dr Rami Elesali
Dr Naeif Almagal
1/1/2020
87

(Figure 1)

A. Ventilation-perfusion scan.
B. Thyroid-stimulating hormone.
C. Echocardiogram.
D. Blood cultures.
E. Troponin.

Many potentially “reversible” causes of atrial fibrillation (AF) have been reported, including binge
drinking, cardiothoracic and noncardiac surgery, myocardial infarction (MI), pericarditis, myocarditis,
hyperthyroidism, electrocution, pneumonia, and pulmonary embolism. It is important to assess for
treatable causes because without targeting the underlying cause, control of AF is likely to be
ineffective and the underlying cause may result in significant morbidity and mortality.

This patient is on oral contraceptives, increasing her risk of thromboembolism. She has shortness of
breath with relative tachypnea, hypoxemia, all consistent with possible pulmonary embolism. Her
ECG shows atrial fibrillation. The fact that she has no evidence of lower extremity deep vein
thrombosis (DVT) on examination does not argue against pulmonary embolism, as the venous
thrombosis could have embolized to her lungs, leaving a benign lower extremity examination. Thus,
a ventilation-perfusion scan to assess for pulmonary embolism is the best answer.

Whereas she has a mildly elevated JVP, she has no evidence of pulmonary vascular congestion and
thus does not appear to have heart failure with reduced ejection fracture. The elevated JVP is likely
from right heart strain from the pulmonary embolism. Although the echocardiogram might show signs
of this, it is not the test most likely to directly yield the correct diagnosis.

It is unlikely that a young female without cardiac risk factors presenting with dyspnea and hypoxia is
having an acute coronary syndrome, and thus troponin is not the best answer.

KKUH
Collected by:
Dr Hani Abdullah
Dr Fatma Hadi
Dr Salem Boresa
Dr Rami Elesali
Dr Naeif Almagal
1/1/2020
88

Although hyperthyroidism could cause AF, and could be responsible for the patient's anxious
appearance, it would not explain her other findings and is thus not the best answer.

Her low-grade temperature is likely due to thromboembolic disease, not infection, and thus blood
cultures are not indicated.

64. The correct answer: A


Key Point
Wherever possible, reversible causes for AF should be sought and corrected. These include:
hyperthyroid state, pulmonary embolism, and pericarditis.

Question 65
65. An 81-year-old female was admitted to the hospital with syncope. Two weeks ago, she was
started on paroxetine for depression. Since then, she has felt increasing fatigue when walking or
doing household chores, and she felt presyncopal several times. This morning, she "blacked out"
and fell to the floor on her way to the kitchen. Her home health aide found her on the floor and called
911; when they arrived, her pulse was 41 bpm and she was "groggy." Her past medical history
includes depression, glaucoma, hypertension, hypothyroidism, dyslipidemia, and intermittent
claudication. Her current medications are amlodipine 5 mg, aspirin 81 mg, cilostazol 100 mg,
levothyroxine 75 mcg, lisinopril 20 mg, paroxetine 20 mg, simvastatin 40 mg, and timolol eye drops.
Her vital signs on admission were blood pressure 138/88 mm Hg without orthostatic change, pulse
48 bpm, and respiration 12 breaths per minute. She appeared her stated age with moist mucous
membranes and normal skin turgor. Her cardiac examination was normal except for bradycardia. An
electrocardiogram was recorded (Figure 1).

Interaction between paroxetine and which of the following is most likely responsible for her syncope?

KKUH
Collected by:
Dr Hani Abdullah
Dr Fatma Hadi
Dr Salem Boresa
Dr Rami Elesali
Dr Naeif Almagal
1/1/2020
89

(Figure 1)

A. Amlodipine.
B. Levothyroxine.
C. Cilostazol.
D. Simvastatin.
E. Timolol.

The correct answer is timolol. Timolol eye drops can produce systemic effects including
bradyarrhythmias that mimic sinus node dysfunction. Both paroxetine and timolol are metabolized by
cytochrome P450 2D6 and when coadministered can increase the concentration of the other drug.
Drugs causing bradycardia should be discontinued before making a diagnosis of sinus node
dysfunction, which requires pacing for treatment. Ophthalmic beta-blockers can often be replaced
with prostaglandins or other ophthalmic preparations for glaucoma with resolution of
bradyarrhythmias.

Cilostazol is a PDE-3 inhibitor that produces systemic vasodilation. This can lead to reflex
tachycardia and would not explain bradyarrhythmias.
Amlodipine, levothyroxine, and simvastatin do not have clinically significant interactions with
paroxetine and would not be expected to produce bradycardia.

65. The correct answer: E


Key Point
SND is a common and often age-related cause of bradycardia although extrinsic causes such as
drug therapy also may cause apparent SND.

KKUH
Collected by:
Dr Hani Abdullah
Dr Fatma Hadi
Dr Salem Boresa
Dr Rami Elesali
Dr Naeif Almagal
1/1/2020
90

Question 66
66. A 30-year-old female presented for evaluation of episodes of recurrent syncope. Over the past
20 years, she has fainted 10 times, but in the past 6 months she has had 3 episodes. Typical spells
occur when she sings in the church choir and are preceded by about 30 seconds of nausea and
sweating. She has not injured herself, but is embarrassed, as the last time she fainted the
paramedics were called. She has no other past medical history and takes no medications. On
physical examination, her blood pressure (BP) was 110/60 mm Hg and heart rate was 90 bpm. Her
jugular venous pulse was 5 cm. There was a single S1 and physiologically split S2 without murmur.
The remainder of her examination was unremarkable. An electrocardiogram in the office showed
normal sinus rhythm at a rate of 93 bpm. The following day, she underwent a tilt table study showing
a supine BP of 120/70 mm Hg and a heart rate of 80 bpm. With passive tilt, the BP over the first 5
minutes was 110-120/55-65 mm Hg and heart rates were 80-100 bpm. At 8 minutes into the tilt, she
complained of nausea just like she feels before her usual faints, and then 30 seconds after that she
lost consciousness. Her BP reading obtained about 20 seconds before fainting was 65/40 mm Hg
and heart rate was 40 bpm. At the moment when she fainted, her heart rate was 30 bpm. She was
immediately laid supine, with prompt return of her BP and heart rate to baseline levels.

Which of the following is the best next step in her management?

A. Initiate midodrine.
B. Implant a dual-chamber pacemaker.
C. Avoid triggers for syncope.
D. Initiate metoprolol.

This patient has vasovagal syncope with a well-defined trigger: standing in the choir at church. Her
tilt table study suggests a mixed vasodepressor and cardioinhibitory response, and it reproduced her
symptoms. While a clinical syncopal spell may show a different physiologic response, the first step in
management should be conservative, stressing hydration, salt, and avoidance of triggers. While
other diagnostic tests such as echocardiogram and stress testing are commonly used, the diagnosis
in this patient is clear, without the need for further diagnostic testing. Likewise, electrophysiologic
study can be useful for patients with syncope suspected to be due to a ventricular arrhythmia, but
does not play a role for the workup of vasovagal syncope. If conservative measures fail, then an
implantable loop recorder can be considered to determine if clinical episodes show a marked cardio-
inhibitory response (despite the tilt table findings), as there can be a role for pacing for patients with
prolonged periods of asystole.

66. The correct answer: C


Key Point
Therapy for recurrent vasovagal syncope should center on education and recognition of prodromal
symptoms. Counterpressure maneuvers can be helpful; salt and fluid liberalization may be helpful in
selected patients.

KKUH
Collected by:
Dr Hani Abdullah
Dr Fatma Hadi
Dr Salem Boresa
Dr Rami Elesali
Dr Naeif Almagal
1/1/2020
91

Question 67
67. A 56-year-old male presented to the emergency department with a 12-hour history of
palpitations. He has had similar spells over the last few years, some of which have required
cardioversion. On physical examination, he was mildly anxious. His heart rate was 145 bpm, blood
pressure was 135/88 mm Hg, respiratory rate was 18 breaths/min, and oxygen saturation 99% while
breathing ambient air. A cardiovascular examination revealed tachycardia but no murmurs. The
remainder of his examination was unremarkable. His electrocardiogram (ECG) was recorded (Figure
1). He was given metoprolol 5 mg intravenously and 25 mg PO. Twenty minutes later, he felt much
better. A repeat ECG was normal and did not reveal any structural abnormalities.

What is the best therapy in the management of this patient?

(Figure 1)

A. Amiodarone.
B. Digoxin.
C. Flecainide PRN.
D. Catheter ablation.
E. Metoprolol.

This patient's ECG demonstrates typical atrial flutter, for which the best treatment is catheter
ablation, which has a 95% success rate. Achieving ventricular rate control in atrial flutter is
characteristically difficult with atrioventricular nodal blocking agents. Given the high success rate
with catheter ablation, either chronic (amiodarone) or episodic (PRN flecainide) use would not be
ideal.

67. The correct answer: D


Key Point
Catheter ablation is the preferred treatment option for recurrent symptomatic PSVT – success rates

KKUH
Collected by:
Dr Hani Abdullah
Dr Fatma Hadi
Dr Salem Boresa
Dr Rami Elesali
Dr Naeif Almagal
1/1/2020
92

exceed 95% in good centers. Drug therapy (prophylaxis or as the occasion arises with drugs that
block AV node or antiarrhythmic drugs) is rarely effective or helpful.

Question 68
68. A 75-year-old man presented to the emergency department with symptoms of palpitations
associated with dizziness. He has a history of coronary artery disease and underwent a 3-vessel
coronary artery bypass graft (CABG) 5 years ago. His most recent echocardiogram 3 months ago
showed an ejection fraction of 20% with a dilated left ventricle and hypokinesis of the mid anterior
and apical segments. On examination, he was in mild distress but alert and oriented. His blood
pressure was 110/55 mm Hg. An electrocardiogram (ECG) was obtained (Figure 1).

Which of the following is the underlying mechanism of his arrhythmia?

(Figure 1)

A. Delayed afterdepolarizations.
B. Early afterdepolarizations.
C. Re-entrant circuit.
D. Abnormal automaticity.
E. Aberrancy in setting of supraventricular tachycardia.

The ECG shows a wide complex tachycardia with positive concordance across the precordial leads
and a monophasic R wave in V1 consistent with ventricular tachycardia (VT). The most common
mechanism of ventricular tachycardia in patients with coronary artery disease is scar-mediated re-
entrant VT. Delayed afterdepolarizations are a type of triggered activity typical of idiopathic outflow
tract VT. Early afterdepolarizations are seen in torsades de pointes in setting of long QT syndrome.

KKUH
Collected by:
Dr Hani Abdullah
Dr Fatma Hadi
Dr Salem Boresa
Dr Rami Elesali
Dr Naeif Almagal
1/1/2020
93

Abnormal automaticity can be seen in fascicular ventricular tachycardias. The 12-lead ECG is not
consistent with supraventricular tachycardia (SVT) with aberrancy.

68. The correct answer: C


Key Point
Monomorphic ventricular tachycardia (MVT) is most commonly due to a re-entry mechanism around
an infarcted scar tissue. Less commonly, MVT is due to a single focus in patients without structural
heart disease.

Question 69
69. A 54-year-old female presented to your office for a second opinion regarding management of
atrial fibrillation. She was diagnosed with atrial fibrillation 5 months ago. She was advised by her
primary cardiologist to consider catheter ablation due to episodes of uncomfortable palpitations and
exercise intolerance that occurred during episodes of atrial fibrillation on an ambulatory
electrocardiogram (ECG) monitor.

Her other medical problems include hypertension, diabetes mellitus, dyslipidemia, and obstructive
sleep apnea. Her medications include hydrochlorothiazide (HCTZ) 12.5 mg daily, metoprolol
succinate 50 mg once daily, metformin 500 mg twice daily, rivaroxaban 20 mg daily, atorvastatin 40
mg daily, and zolpidem 5 mg at bedtime. She reported that she wears a continuous positive airway
pressure (CPAP) mask nightly. She drinks 1 alcoholic beverage nightly and admitted to smoking 1/2
pack cigarettes daily, although she is trying to quit.

On examination, she was 5'2" and 160 lbs (body mass index [BMI] 29.3 kg/m2). Her blood pressure
was 128/70 mm Hg, heart rate 76 bpm. Her lungs were clear, heart regular without murmurs, and
extremities were without edema. ECG showed normal ventricular and valvular function with no
pulmonary hypertension. A treadmill stress test showed no ischemia at adequate heart rate. She
would like to avoid catheter ablation and asked what other interventions might help.

What intervention do you recommend?


A. Switch hydrochlorothiazide to valsartan 80 mg daily.
B. Change continuous positive airway pressure to bilevel positive airway pressure.
C. Weight loss of 15-20 lbs.
D. Smoking cessation.
E. Switch metformin to glyburide 5 mg daily.

Weight management is associated with a reduction in atrial fibrillation (AF) symptom burden. In
the LEGACY (Long-Term Effect of Goal-Directed Weight Management in an Atrial Fibrillation
Cohort) trial, 335 patients with BMI >29 kg/m2 were offered weight management assistance and
followed for 5 years. Weight loss of ≥10% of body weight resulted in a sixfold greater
probability of arrhythmia-free survival at 5 years. Thus, 15-20 lbs weight loss is the correct
answer.

KKUH
Collected by:
Dr Hani Abdullah
Dr Fatma Hadi
Dr Salem Boresa
Dr Rami Elesali
Dr Naeif Almagal
1/1/2020
94

Treatment of obstructive sleep apnea (OSA) with CPAP appears to favorably impact AF
management.3,4 However, there is no evidence that switching CPAP to bilevel positive airway
pressure (BiPAP) for the treatment of OSA would help and this is not the correct answer.

Although smoking cessation has numerous benefits, the relationship between smoking cigarettes
and AF recurrence is not established and this is not the correct answer.

Whether different antihypertensive medications affect the development of AF independent of


blood pressure reduction is not clear. In the GISSI-AF (Gruppo Italiano per lo Studio Della
Sopravvivenza Nell'Infarto Miocardico–Atrial Fibrillation) trial, valsartan was not superior to
placebo in secondary prevention of AF at 1 year. In the ANTIPAF (Angiotensin II-Antagonist in
Paroxysmal Atrial Fibrillation) trial, olmesartan did not decrease AF burden compared to
placebo in patients without structural heart disease. Thus, switching HCTZ to an angiotensin-
receptor blocker (ARB) when the blood pressure is already optimally controlled is not the best
answer.

The presence of diabetes mellitus is a risk factor for the development of AF. An observational
study from Taiwan assessed the effects of metformin on the development of AF. In 645,710
patients over a 13-year follow-up, fewer patients taking metformin developed AF, suggesting
that metformin had a protective effect on the development of AF in patients with diabetes
mellitus. Thus, switching metformin to a different agent is not the right answer.
69. The correct answer: C
Key Point
Chronic contributors to AF, including hypertension, obstructive sleep apnea, and obesity, should be
addressed in all patients. Not infrequently, rate control and correction of these comorbidities can
resolve symptoms.

Question 70
70.A 77-year-old female with hypertension was brought to the emergency department by her
husband after a syncopal episode at home. Prior to the event, she had been standing in the kitchen
preparing dinner and feeling well; she denied any warning symptoms. The event was unwitnessed.
She only recalled awakening on the floor. There was no history of syncope. Her medications
included aspirin 81 mg and amlodipine 5 mg. An electrocardiogram was recorded (Figure 1). The
patient remained asymptomatic and initial laboratories were unremarkable.

Which of the following is the best next step in this patient's management?

KKUH
Collected by:
Dr Hani Abdullah
Dr Fatma Hadi
Dr Salem Boresa
Dr Rami Elesali
Dr Naeif Almagal
1/1/2020
95

(Figure 1)
Modified from Roediger JE. Available at: https://en.wikipedia.org/wiki/

A. Start fludrocortisone.
B. Discontinue amlodipine.
C. Permanent pacemaker implantation.
D. Thirty-day event monitor.
E. Coronary angiography.

This patient experienced unheralded syncope and has second-degree atrioventricular (AV) block,
Mobitz II. In patients with symptomatic irreversible bradycardia due to sinus node dysfunction or AV
block, placement of a permanent pacemaker is indicated. An event monitor may be beneficial to
diagnose the cause of syncope in a patient with vague symptoms and no identifiable conduction
system disease. Discontinuing amlodipine would not be beneficial, as it has no significant action on
the sinus or AV nodes, and the conduction abnormality in this case is infranodal. Fludrocortisone
might be useful to treatment orthostatic hypotension but is not indicated for symptoms due to
conduction system disease. Finally, coronary angiography would not be indicated in this patient
without angina or other evidence of myocardial ischemia.

70. The correct answer: C


Key Point
Patients with symptomatic irreversible bradycardia due to SND or AV block require permanent
pacemaker implantation.

KKUH
Collected by:
Dr Hani Abdullah
Dr Fatma Hadi
Dr Salem Boresa
Dr Rami Elesali
Dr Naeif Almagal
1/1/2020
96

Question 71
71.A 28-year-old male was referred by his primary care clinician for evaluation of a murmur and an
abnormal echocardiogram. He exercises regularly and denied any exertional symptoms. He is an
avid cyclist, takes no medications, and has no relevant medical history. His family history is
unremarkable.

On examination, his heart rate was 62 bpm with blood pressure of 135/80 mm Hg. Cardiac
auscultation revealed a soft, II/VI systolic ejection murmur loudest at the right upper sternal border
that augments with rising from a squatting position.

An electrocardiogram (ECG) revealed normal sinus rhythm and left ventricular hypertrophy with
repolarization changes. An echocardiogram demonstrated normal biventricular function with normal
left ventricle (LV) size. There was a severely thickened interventricular septum measuring 2.5 cm.
There was systolic anterior motion of the mitral valve with a peak left ventricular outflow tract (LVOT)
velocity of 3.8 m/sec; this increased to 5.0 m/sec with Valsalva.

Which of the following is the best next step?


A. Treadmill stress test.
B. Implantable loop recorder.
C. Repeat echocardiogram in 1 year.
D. Electrophysiology study.
E. Dobutamine stress test.

Risk factors for sudden death in hypertrophic cardiomyopathy include: sudden death in a first-degree
relative, maximal septal thickness >30 mm, unexplained syncope, low blood pressure during
treadmill stress testing, and nonsustained ventricular tachycardia (NSVT) on Holter. In this patient
with hypertrophic cardiomyopathy (HCM), more data about his risk of sudden death, including the
blood pressure response to exercise, are needed; thus, this is the correct answer. The degree of
LVOT obstruction is dynamic and affected by loading conditions and autonomic tone. Although some
studies have suggested an association between LVOT obstruction and sudden cardiac death, the
gradient alone is not a basis for decisions about implantable cardioverter-defibrillator (ICD) therapy
(Figure 1).

Repeating an echocardiogram in 1 year is the not the best answer because additional risk
stratification is required at this time.
Electrophysiology study is not used for risk stratification in HCM.
Dobutamine stress testing offers no additional information in patients with HCM. Additionally,
dobutamine would be expected to worsen the LVOT gradient, which could be dangerous.

Although an ambulatory rhythm monitor (i.e., Holter or event monitor) may be useful to assess for
NSVT as part of his risk stratification, there is no indication for an implantable recorder in this patient.

KKUH
Collected by:
Dr Hani Abdullah
Dr Fatma Hadi
Dr Salem Boresa
Dr Rami Elesali
Dr Naeif Almagal
1/1/2020
97

(Figure 1)
*SCD risk modifiers include established risk factors and emerging risk modifiers

71. The correct answer: A


Key Point
In patients with other cardiomyopathies, such as hypertrophic cardiomyopathy, arrhythmogenic RV
cardiomyopathy, and sarcoidosis, and inherited channelopathies, such as long QT syndrome,
catecholaminergic polymorphic VT, and Brugada syndrome, ICD implantation is a Class I indication
for those patients who have survived a cardiac arrest due to VT/VF. An ICD may be considered to
be beneficial (Class IIa indication) in subgroups of patients with high risk factor profiles, for example,
hypertrophic cardiomyopathy with unexplained syncope, LV wall thickness =30 mm, or sudden
cardiac death in a first-degree relative.

KKUH
Collected by:
Dr Hani Abdullah
Dr Fatma Hadi
Dr Salem Boresa
Dr Rami Elesali
Dr Naeif Almagal
1/1/2020
98

Question 72
72. A 52-year-old male was admitted with palpitations. His electrocardiogram (ECG) was obtained
(Figure 1). He has a history of hypertension and is active at baseline without symptoms of chest pain
or dyspnea. His examination was notable for tachycardia, jugular venous pressure of 5 cm H20, no
appreciable cardiac murmurs, clear lungs, and no lower extremity edema.

In addition to an echocardiogram, which of the following tests is indicated?

(Figure 1)

A. Metanephrines.
B. D-dimer.
C. Lyme antibody.
D. C-reactive protein.
E. Thyroid stimulating hormone.

In patients with atrial fibrillation, reversible causes should be investigated, including hyperthyroidism,
pericarditis, pulmonary embolism, and electrolyte abnormalities. An echocardiogram is an
appropriate test to determine if there is structural heart disease. Lyme disease can be associated
with atrioventricular (AV) block but is not considered a typical cause of atrial fibrillation. D-dimer for
pulmonary embolism and c-reactive protein testing for pericarditis are not indicated in patients
without symptoms of these diseases.

72. The correct answer: E

KKUH
Collected by:
Dr Hani Abdullah
Dr Fatma Hadi
Dr Salem Boresa
Dr Rami Elesali
Dr Naeif Almagal
1/1/2020
99

Key Point
Wherever possible, reversible causes for AF should be sought and corrected. These include:
hyperthyroid state, pulmonary embolism, and pericarditis.

Question 73
73. A 25-year-old male with no significant past medical history presented to his primary care clinician
for a pre-employment physical examination. He has been active and has no complaints. His family
history was unavailable, as he was adopted. On physical examination, his blood pressure was
110/60 mm Hg with a heart rate of 60 bpm, and his examination was within normal limits. An
electrocardiogram (ECG) was obtained (Figure 1).

Which of the following activities would you recommend as possibly safe?

(Figure 1)

A. Hockey.
B. Raquetball.
C. Swimming.
D. Diving.
E. Golf.

Athletes with a definite diagnosis of arrhythmogenic right ventricular cardiomyopathy (ARVC) should
not participate in most competitive sports, with the possible exception of low-intensity class 1A
sports (Class III, Level of Evidence C).

KKUH
Collected by:
Dr Hani Abdullah
Dr Fatma Hadi
Dr Salem Boresa
Dr Rami Elesali
Dr Naeif Almagal
1/1/2020
100

Class 1A sports are low in static and dynamic loads. Examples include billiards, bowling, cricket,
curling, golf, and riflery. All other options include higher dynamic and/or static loads (Figure 2).

(Figure 2)
Reproduced with permission from Levine BD, Baggish AL, Kovacs RJ, et al. Eligibility and
disqualification recommendations for competitive athletes with cardiovascular abnormalities: Task
Force 1: classification of sports: dynamic, static, and impact: a scientific statement from the
American Heart Association and American College of Cardiology. J Am Coll Cardiol 2015;66:2350-5.
73. The correct answer: E
Key Point
If arrhythmogenic right ventricular cardiomyopathy is diagnosed in an index case, family screening
should be considered. Implantable cardioverter-defibrillator therapy may be warranted. Patients
should avoid vigorous exercise, as this can clearly accelerate disease progression.

Question 74
74. A 72-year-old female presented with complaints of weakness and fatigue. She has a past history
significant for atrial fibrillation managed with rate control strategy, peripheral vascular disease, and
hypertension. She recently was discharged from the hospital following a recurrent episode of atrial
fibrillation requiring cardioversion. She was unclear what her home medications are, but knows she
was taking several new medications for her heart.

KKUH
Collected by:
Dr Hani Abdullah
Dr Fatma Hadi
Dr Salem Boresa
Dr Rami Elesali
Dr Naeif Almagal
1/1/2020
101

Upon physical examination, she was confused and lethargic. Her blood pressure was 91/54 mm Hg
and pulse was 103 bpm. Physical examination was remarkable for a tachycardic rhythm and no
murmurs. She complained of blurred vision. An electrocardiogram (ECG) was obtained in the
emergency department (Figure 1).

The recent addition of which of the following medications to this patient's medication regime is most
likely responsible for her clinical findings and ECG changes?

A. Amlodipine.
B. Nadolol.
C. Dronedarone.
D. Labetalol.
E. Apixaban.

Dronedarone is a potent inhibitor of P-glycoprotein. When administered concomitantly with digoxin, it


may lead to substantial increases in digoxin steady-state levels. Coadministration of digoxin and
dronedarone requires a dose reduction in digoxin (as is also the case with amiodarone
coadministration). The ECG demonstrates bidirectional ventricular tachycardia, a classical finding in
digoxin toxicity. The patient is digoxin toxic and should be treated with digoxin immune fab (ovine).
The other agents are not inhibitors of P-glycoprotein and would not be expected to result in the
clinical condition described.

KKUH
Collected by:
Dr Hani Abdullah
Dr Fatma Hadi
Dr Salem Boresa
Dr Rami Elesali
Dr Naeif Almagal
1/1/2020
102

74. The correct answer: C


Key Point
Elimination of antiarrhythmic drugs is via kidneys and hepatic P450 system. Important interactions
include amiodarone and warfarin (increased INR), and P-glycoprotein a renal transporter for drugs
such as digoxin.

Question 75
75. A 54-year-old female with a history of hypertension and surgical atrial septal defect repair
approximately 10 years prior presented to your office for urgent evaluation. She had been
experiencing palpitations that began suddenly the evening prior and that are associated with fatigue
and decreased exercise tolerance. Her only medication is lisinopril 20 mg daily. Her blood pressure
was 124/76 mm Hg, heart rate 100 bpm and regular. The remainder of her physical examination was
unremarkable. An electrocardiogram (ECG) was recorded (Figure 1).

Which of the following is the best next step in this patient's management?

(Figure 1)
Reproduced with permission from Ferreira R, Primo J, Adão L, et al. Late atypical atrial flutter after
ablation of atrial fibrillation. Rev Port Cardiol 2016;35:doi: 10.1016/j.repc.2015.10.005.

A. Catheter ablation.
B. Transesophageal echocardiogram.
C. Intravenous flecainide.
D. Start diltiazem.
E. Direct current cardioversion.
KKUH
Collected by:
Dr Hani Abdullah
Dr Fatma Hadi
Dr Salem Boresa
Dr Rami Elesali
Dr Naeif Almagal
1/1/2020
103

Although definitive diagnosis frequently requires electrophysiology study and intracardiac mapping,
the history of prior surgical atrial septal defect repair and the concordant ECG flutter wave polarities
suggest that this patient has atypical atrial flutter. Atypical atrial flutter is a macroreentrant atrial
arrhythmia that does not utilize the cavo-tricuspid circuit present in typical atrial flutter, and therefore
is classified as “non–isthmus-dependent” atrial flutter. As the patient has been in symptomatic atrial
flutter for <48 hours, proceeding with direct-current (DC) cardioversion without performing
transesophageal echocardiogram (TEE) is reasonable. Compared to DC cardioversion, intravenous
flecainide and other antiarrhthmic medications exhibit poor cardioversion efficacy. Cather ablation
can be considered, but in general is more difficult in atypical atrial flutter, and the current guidelines
recommend reserving ablation for recurrent atypical atrial flutter after failure of at least 1
antiarrhythmic agent. Pursuing an acute rate control strategy by starting diltiazem is unlikely to be
very effective in this patient, as atrial flutter is notoriously difficult to rate control. Finally, as patients
with atrial flutter are thought to have the same thromboembolism risk as patients with atrial
fibrillation, they therefore should be anticoagulated similarly.

75. The correct answer: E


Key Point
Non–isthmus-dependent atrial flutter involves macro–re-entrant circuits elsewhere in the RA or LA
and may occur in a variety of clinical settings, including congenital heart disease, after cardiac
surgery, and after catheter ablation of AF.

Question 76
76. A 67-year-old female presented to your office with fatigue and dizziness. She had hypertension
and coronary artery disease with a prior right coronary artery stent. Her medications include aspirin
81 mg daily, atorvastatin 20 mg daily, and lisinopril 10 mg daily. On physical examination, her heart
rate was 60 bpm and blood pressure was 126/80 mm Hg with no orthostatic changes. There was no
nystagmus and the Epley maneuver did not elicit vertigo. She had a split S2. The remainder of her
examination was unremarkable. Her electrocardiogram (ECG) was recorded (Figure 1).

Which of the following is the best next step?

KKUH
Collected by:
Dr Hani Abdullah
Dr Fatma Hadi
Dr Salem Boresa
Dr Rami Elesali
Dr Naeif Almagal
1/1/2020
104

(Figure 1)

A. Thirty-day event monitor.


B. Coronary angiography.
C. Permanent pacemaker.
D. Electrophysiological study.
E. Implantable loop recorder.

This patient's ECG demonstrates right bundle branch block (RBBB). Her symptoms of dizziness may
be due to intermittent advanced heart block, and ambulatory ECG monitoring may be helpful for
correlating her symptoms with ECG findings. An event recorder would be the best next step.

It would be premature to consider an implantable loop recorder prior to a noninvasive evaluation.

As the patient does not have evidence of symptomatic bradycardia or high-degree atrioventricular
(AV) block yet, there is no indication for a permanent pacemaker.

Electrophysiology study is not indicated without further evidence of high-degree AV block warranting
further invasive investigation of the conduction system.

The patient does not have any subjective or objective findings of ischemia to warrant coronary
angiography at this time.

76. The correct answer: A


Key Point
When evaluating patients with lesser degrees of bradycardia, symptom-rhythm correlation is
important. This can be achieved with ambulatory monitoring (Holter or similar). More prolonged
monitoring with an implantable subcutaneous monitor may be needed in some cases.

KKUH
Collected by:
Dr Hani Abdullah
Dr Fatma Hadi
Dr Salem Boresa
Dr Rami Elesali
Dr Naeif Almagal
1/1/2020
105

Question 77
77. A 16-year-old female high school soccer player presented to your office for evaluation after an
abnormality was detected on her electrocardiogram (ECG), which was done as part of her high
school's athletic screening program (Figure 1).

What is the most appropriate next step?

(Figure 1)

A. Isoprotenolol challenge.
B. Procainamide challenge.
C. Exercise stress test.
D. Genetic testing.
E. Reassurance.

The ECG (Figure 1) demonstrates evidence of pre-excitation, likely as a result of an accessory


pathway. In this case, the patient has Wolff-Parksinon-White (WPW) pattern, as she is asymptomatic
but has an ECG pattern consistent with WPW. The prevalence of a WPW pattern on ECG is
estimated at 0.13-0.25% in the general population. WPW syndrome (pre-excitation on ECG
associated with symptomatic arrhythmia) is 10-100 times less common. Noninvasive exercise testing
for risk stratification is the next appropriate step (Class I recommendation). During exercise stress,
complete and abrupt disappearance of the delta wave is suggestive of a low-risk pathway (Figure 2).
Persistent pre-excitation, including gradual shortening of the delta wave, suggests a higher-risk
pathway capable of conducting at higher heart rates, placing the patient at risk for arrhythmias,
including ventricular fibrillation. An alternative approach is to proceed directly to electrophysiologic
study including procainamide challenge (Class IIa), but a noninvasive approach is preferable as the
first step. Isoproterenol may be useful for provocation of ventricular arrhythmias such as are seen in
arrhythmogenic right ventricular cardiomyopathy or as an adjunct to tilt table testing for syncope, but
KKUH
Collected by:
Dr Hani Abdullah
Dr Fatma Hadi
Dr Salem Boresa
Dr Rami Elesali
Dr Naeif Almagal
1/1/2020
106

is not useful in WPW patients. Genetic testing does not have an established role in a WPW pattern.
It is not appropriate to reassure the patient without further risk stratification.

(Figure 2)
During exercise treadmill testing, this patient abruptly lost pre-excitation at a heart rate of 117 bpm.
Beginning abruptly with the beat under the arrow, the PR interval normalizes , and the QRS changes
from pre-excited to narrow.

77. The correct answer: C

Key Point
Genesis of the delta wave is due to “fusion” of impulses activating the ventricles via both the AV
node and activation via the accessory pathway. In other words, a delta wave is only present if the
accessory pathway can conduct from the atria to the ventricles. Factors that enhance AV nodal
conduction (such as exercise) cause predominantly more of the ventricles to be activated in a
normal fashion such that the delta wave will diminish with exercise, whereas factors that slow AV
nodal conduction such as drugs will increase the degree of pre-excitation. In other words, the degree
of pre-excitation can change and is not fixed.

KKUH
Collected by:
Dr Hani Abdullah
Dr Fatma Hadi
Dr Salem Boresa
Dr Rami Elesali
Dr Naeif Almagal
1/1/2020
107

Question 78
78.A 22-year-old male with arrhythmogenic right ventricular cardiomyopathy presented to the clinic to
establish care. His diagnosis was made based on the findings of symptomatic nonsustained
ventricular tachycardia, an electrocardiogram with T-wave inversions in leads V1-V3, and a cardiac
magnetic resonance imaging with an elevated right ventricular end-diastolic volume and regional
hypokinesis. He subsequently underwent implantation of an implantable cardioverter-defibrillator.
During discussion of management of his condition, he asked for advice regarding lifestyle
interventions.

Which of the following lifestyle modifications do you suggest?


A. Limit alcohol.
B. Avoid air travel.
C. Limit competitive sports participation.
D. Limit caffeine.
E. Restrict fluid intake.

Arrhythmogenic cardiomyopathy is a progressive, heritable disease of cardiac muscle resulting in a


high risk of ventricular arrhythmias and sudden death, even when cardiac structural changes are
minimal. Based on the increased risk of cardiac arrest and sudden death in athletes with
arrhythmogenic cardiomyopathy, patients are advised to avoid competitive sports and endurance
training. Physical activity may accelerate structural progression of arrhythmogenic cardiomyopathy;
in a study of heterozyogous plakoglobin- deficient mice, endurance training accelerated the
development of arrhythmias and right ventricular dysfunction. Although caffeine, alcohol,
dehydration, and mild hypoxia may trigger arrhythmia in individuals, there are no known specific
links to arrhythmogenic cardiomyopathy.

78. The correct answer: C


Key Point
If arrhythmogenic right ventricular cardiomyopathy is diagnosed in an index case, family screening
should be considered. Implantable cardioverter-defibrillator therapy may be warranted. Patients
should avoid vigorous exercise, as this can clearly accelerate disease progression.

Question 79
79. A 65-year-old woman presented for follow-up in the heart failure clinic. She was able to do her
usual activities, but was limited when trying to walk upstairs. She denied any chest pain. Her past
medical history includes systolic heart failure (left ventricular ejection fraction 30%) for the past 3
years, coronary artery disease with an anterior wall myocardial infarction 5 years ago, type 2
diabetes mellitus, hyperlipidemia, and hypertension. Her medications include lisinopril 20 mg daily,
carvedilol 25 mg BID, atorvastatin 40 mg daily, aspirin 81 mg daily, and metformin 850 mg BID. On
examination, her blood pressure was 100/70 mm Hg and pulse was 70 bpm. Her examination was
otherwise unremarkable and euvolemic. Her electrocardiogram showed sinus rhythm at 70 bpm,
with a left bundle branch block (LBBB) and QRS duration of 155 msec.
KKUH
Collected by:
Dr Hani Abdullah
Dr Fatma Hadi
Dr Salem Boresa
Dr Rami Elesali
Dr Naeif Almagal
1/1/2020
108

In addition to referring her to cardiac rehabilitation, which of the following is the best next step in this
patient's care?

A. Refer for cardiac transplantation.


B. Add torsemide.
C. Implant cardioverter defibrillator.
D. Implant biventricular defibrillator.

The guidelines for biventricular pacing specify the type of QRS complex, QRS duration, and New
York Heart Association (NYHA) functional class. Class I indication for defibrillator includes an EF of
35% or less with NYHA class II or III symptoms.

This patient also has a class I indication for biventricular pacing with class II, III, or ambulatory IV
heart failure, LBBB, and QRS duration of 150 ms or greater, which is why a defibrillator alone would
not be the best choice.

She is euvolemic and does not need diuresis.


Her NYHA class II symptoms and good functional capacity do not merit advanced therapy (left
ventricular assist device or heart transplantation) at this time.

79. The correct answer: D

Question 80
80. A 75-year-old female with hypertension and atrial fibrillation presented to your office for
consultation. She felt well with no symptomatic palpitations or dyspnea. Her medications include
carvedilol 6.25 mg twice daily, lisinopril 10 mg daily, and rivaroxaban. On examination, her blood
pressure was 105/70 mm Hg, her heart rate was 98 bpm, and she was afebrile. Cardiac examination
revealed irregularly irregular rhythm and the remainder of her examination was unremarkable. Her
electrocardiogram showed atrial fibrillation at approximately 100 bpm and left bundle branch block. A
nuclear myocardial perfusion scan performed 6 months prior showed a normal left ventricular
ejection fraction (LVEF) without perfusion defects.

Which of the following is the most appropriate next step in the management of this patient?
A. Add dofetilide.
B. No change in therapy.
C. Add diltiazem.
D. Add digoxin.
E. Atrial fibrillation ablation.

KKUH
Collected by:
Dr Hani Abdullah
Dr Fatma Hadi
Dr Salem Boresa
Dr Rami Elesali
Dr Naeif Almagal
1/1/2020
109

This patient has asymptomatic, rate-controlled atrial fibrillation. In the absence of symptoms and with
normal LVEF, a strategy of rate control and anticoagulation is reasonable.
The RACE-II Study compared rate control with a target resting heart rate <80 bpm to a rate control
strategy with a target resting heart rate <110 bpm in patients with permanent atrial fibrillation and
found similar rates of major adverse events with both strategies. Therefore, adding additional rate-
controlling drugs such as diltiazem or digoxin may be associated with increased side effects without
clear benefits in a low-risk, asymptomatic patient.

The AFFIRM Study and RACE-I Study both demonstrated that rate and rhythm control strategies
have similar major adverse event rates. Similarly, ablation of atrial fibrillation would not be expected
to have meaningful benefits for this asymptomatic patient with normal LVEF.

80. The correct answer: B


Key Point
The main goal in treating AF is to address symptoms thought to be related to AF. Rate or rhythm
control seems to be equivalent in terms of mortality. Selection of a therapeutic strategy takes into
account factors that include age, symptoms, duration of AF, evidence of tachycardia-mediated
cardiomyopathy, or difficulty in achieving adequate rate control.

Question 81
81.A 42-year-old woman with fibromyalgia presented to your office with weakness, fatigue,
lightheadedness, and palpitations. Her wearable heart rate monitor reported sinus rhythm with heart
rates ranging from 90 to 120 bpm at rest that she said correlated with her symptoms. She had
previously been evaluated with normal thyroid function tests, blood counts, and electrolytes. Her
electrocardiogram is shown (Figure 1). She had previously tried metoprolol, atenolol, and nebivolol,
which all caused debilitating fatigue.

Which of the following is the best treatment?

KKUH
Collected by:
Dr Hani Abdullah
Dr Fatma Hadi
Dr Salem Boresa
Dr Rami Elesali
Dr Naeif Almagal
1/1/2020
110

(Figure 1)

A. Ivabradine 5 mg BID.
B. Propafenone 225 mg BID.
C. Digoxin 0.125 mg daily.
D. Radiofrequency ablation.
E. Cardiac rehabilitation.

While ivabradine is not FDA-approved for treatment of inappropriate sinus tachycardia, the 2015
heart rhythm society expert consensus statement on the diagnosis and treatment of postural
tachycardia syndrome, inappropriate sinus tachycardia, and vasovagal syncope (Sheldon RS et al
Heart Rhythm 2015) gives ivabradine a Class IIa recommendation for this indication: "Ivabradine can
be useful for treating patients with IST." Ivabradine holds considerable promise for the treatment of
IST. The drug blocks the If current and has a dramatic and generally well-tolerated effect on heart
rate. At doses of 5– 7.5 mg twice daily, the drug slows the heart rate by 25–40 bpm. The strongest
evidence to date comes from a randomized crossover study in which 21 patients with IST were
randomized to placebo or ivabradine 5 mg twice daily for a total of 12 weeks. ß-Adrenergic blockers
are not usually effective and can cause adverse effects. Digoxin and propafenone are not
recommended for treatment of IST. Other treatments have been suggested, including
fludrocortisone, volume expansion, pressure stockings, phenobarbital, clonidine, psychiatric
evaluation, exercise training, and erythropoietin, but have not proven useful and are not specific
recommended in the guidelines. Modification or ablation of the sinus node can be performed IST.
However, there is a high rate of symptom recurrence, and the complication rates are significant,
including the need for permanent pacing, phrenic nerve paralysis, and transient superior vena cava
syndrome. Given the young age of the patients and the highly invasive nature of ablations, we do not
recommend that they be part of routine care.

KKUH
Collected by:
Dr Hani Abdullah
Dr Fatma Hadi
Dr Salem Boresa
Dr Rami Elesali
Dr Naeif Almagal
1/1/2020
111

81. The correct answer: A


Key Point
If (for “funny current”) is located in the SA node and drives sinus rate.

Question 82
82. A 64-year-old male presented to the clinic for evaluation of an abnormal electrocardiogram
(ECG) (Figure 1). His past medical history was significant for hypertension, for which he is treated
with lisinopril. He denied cardiac symptomatology. He is active, works full-time as a car salesman,
and plays golf regularly. His examination was unremarkable. His laboratories, including thyroid
stimulating hormone (TSH), were normal.

Which of the following is the best next step?

A. Carotid sinus massage.


B. Echocardiogram.
C. Vasodilator perfusion imaging stress test.
D. Dual-chamber pacemaker.
E. Cardiac magnetic resonance imaging.

The ECG is consistent with sinus rhythm with 2:1 atrioventricular (AV) block. The differential is
Mobitz type 1 (Wenckebach) second-degree AV block versus Mobitz type 2 second-degree AV

KKUH
Collected by:
Dr Hani Abdullah
Dr Fatma Hadi
Dr Salem Boresa
Dr Rami Elesali
Dr Naeif Almagal
1/1/2020
112

block. His ECG is more suggestive of Mobitz type 1 second-degree block based on the long PR
interval and narrow QRS interval, but the diagnosis still needs to be established because higher-
grade AV block would merit further investigation and potential pacemaker placement.

Noninvasive vagal and sympathetic maneuvers can help to distinguish one from the other because
Mobitz type 1 block is usually AV nodal, and Mobitz type 2 block is usually in the His-Purkinje
system. In a patient with AV nodal block, carotid sinus massage will slow the sinus rate and worsen
AV conduction, which will worsen the AV block. The slowing of sinus rate and depression of AV
conduction will have a protective effect on the distal conduction system, so conduction in the His-
Purkinje system may improve. Thus, carotid sinus massage is the correct answer.

Alternatively, exercise (sympathetic stimulation) will improve AV nodal conduction and worsen His-
Purkinje conduction. Thus, an exercise stress test may be helpful but a vasodilator perfusion
imaging stress test would offer no further information and is contraindicated based on his conduction
disease.
Although the detection of structural heart disease may help guide patient therapy, it is not the best
next diagnostic maneuver to assess the degree and level of AV block.
If the site of block is within the AV node in an asymptomatic patient, a pacemaker is not indicated.
Cardiac magnetic resonance imaging (MRI) may help establish the etiology if infraHisian block is
identified, but further information is needed before proceeding with an MRI.

82. The correct answer: A


Key Point
AV block occurs due to a failure of impulse propagation through the cardiac conduction system.
Several types of AV block are recognized; most commonly first-, second-, and third-degree block.
Prognosis of AV block is related to the severity of block. AV block may be due to failure of impulse
propagation in the AV nodal tissues (narrow QRS complex) or within the His-Purkinje system (intra-
or infra-Hisian block). Failure of impulse propagation due to infra-Hisian block may be associated
with a wide QRS complex with bundle branch or fascicular block.

Question 83
83.A 73-year-old man with a dual-chamber pacemaker placed 5 years ago for sinus node
dysfunction presented to your clinic with increasing dependent edema and paroxysmal nocturnal
dyspnea. His past medical history includes hypertension, diabetes mellitus, and obesity. His current
medications are aspirin 81 mg, atorvastatin 80 mg, lisinopril 10 mg, metoprolol succinate 100 mg,
and glyburide 5 mg. His height was 5'6", weight 200 lbs, pulse 50 bpm, blood pressure 110/80 mm
Hg, and respiration 12 breaths per minute. The jugular veins were not visible, the lungs were clear,
the heart was regular with paradoxical splitting of S2, and the ankles were swollen with chronic
venous stasis changes.

An echocardiogram showed dilated left ventricle with severe global left ventricular systolic
dysfunction, estimated left ventricular ejection fraction 25%. There was moderate functional mitral
regurgitation

KKUH
Collected by:
Dr Hani Abdullah
Dr Fatma Hadi
Dr Salem Boresa
Dr Rami Elesali
Dr Naeif Almagal
1/1/2020
113

What is the best next step?

A. Endomyocardial biopsy.
B. Pacemaker interrogation.
C. Polysomnography.
D. Cardiac magnetic resonance imaging.
E. Serum and urine immunofixation electrophoresis.

The correct answer is pacemaker interrogation. Although the device was placed for sinus node
dysfunction, it would be important to assess his percentage of RV pacing. He may have developed
heart block since implantation or have been programmed inappropriately resulting in a high
percentage of RV pacing. Chronic right ventricular pacing may result in the development of heart
failure, occurring in up to 12% of patients who are paced >20% of the time, and this should be
considered since it is reversible. If you are not able to decrease the percentage of RV pacing by
programming changes, upgrade to a biventricular pacing system is indicated in such patients.
Although other causes of cardiomyopathy such as amyloid are possible, testing for amyloid would
not be the first step. Obstructive sleep apnea can worsen heart failure but would be not be likely to
explain his severe left ventricular dysfunction. Cardiac magnetic resonance imaging can be useful
for diagnosing infiltrative heart disease but must be performed with caution in patients with pacing
systems. Furthermore, conditions such as cardiac sarcoidosis or hemochromatosis would be unlikely
to present in a patient at this age. Endomyocardial biopsy is generally low-yield and not indicated in
this situation.

83. The correct answer: B


Key Point
Prolonged RV apical pacing may in some cases cause a deleterious effect on ventricular function:
DAVID trial. Ventricular pacing alone may also increase the likelihood of AF.

Question 84
84. A 76-year-old male with a history of atrial fibrillation, hypertension, diabetes mellitus, stroke 2
months prior, and a dual-chamber pacemaker for tachy-brady syndrome was scheduled to undergo
pacemaker generator change. His medications include Lisinopril 20 mg daily, metformin 850 mg
twice daily, metoprolol succinate 50 mg daily, and warfarin 5 mg daily.

Which of the following is the most appropriate periprocedure anticoagulation management strategy?
A. Hold warfarin 5 days prior.
B. Replace warfarin with dabigatran (150 mg twice daily).
C. Continue warfarin.
D. Bridge with enoxaparin.
E. Replace warfarin with aspirin (325 mg daily).

KKUH
Collected by:
Dr Hani Abdullah
Dr Fatma Hadi
Dr Salem Boresa
Dr Rami Elesali
Dr Naeif Almagal
1/1/2020
114

Management strategies for periprocedure anticoagulation are based on the risk-benefit assessment
including the indication for anticoagulation and the risk of bleeding from the procedure. In this case,
the patient has a CHA2DS2-VASc score of 6 with a recent prior stroke and therefore continuing
warfarin is the most appropriate strategy.

Withholding all anticoagulation is not appropiate is this high-risk patient.


Substituting with aspirin offers inferior thromboembolic protection and is thus not recommended.

Enoxaparin carries a higher postprocedure bleeding risk as demonstrated in the BRUISE


CONTROL (Bridge or Continue Coumadin for Device Surgery Randomized Controlled
Trial) and would not be indicated.

There is no evidence that periprocedure dabigatran offers less bleeding risk than continuing current
therapy with warfarin and therefore is not the correct choice.

84. The correct answer: C


Key Point
The CHA2DS2-VASc and HAS-BLED score decisions are used to balance the need for long-term
anticoagulant therapy against the risk of bleeding, respectively. These scoring systems are best
utilized in patients with nonvalvular AF, and do not apply to other special populations, such as those
with hypertrophic cardiomyopathy.

Question 85
85. A 43-year-old male with a history of tobacco use and hypertension presented to the emergency
department with chest discomfort that radiated to the back. His chest pain began an hour prior to his
arrival while eating dinner with his family. He denied any shortness of breath, palpitations, or
nausea. His family history is notable for his father dying suddenly at age 50. On physical
examination, he was diaphoretic. His heart rate was 90 bpm and blood pressure was 150/95 mm Hg.
There were no murmurs and his lungs were clear. His abdomen was mildly distended with normal
bowel sounds. His initial electrocardiogram showed normal sinus rhythm with no ST or T-wave
abnormalities. His first troponin was negative. He was placed on a monitor. Ten minutes later, he
became unresponsive. His rhythm strip was obtained (Figure 1).

Which of the following is the most likely etiology of the rhythm shown?

(Figure 1)
KKUH
Collected by:
Dr Hani Abdullah
Dr Fatma Hadi
Dr Salem Boresa
Dr Rami Elesali
Dr Naeif Almagal
1/1/2020
115

A. Long QT syndrome.
B. Brugada syndrome.
C. Acute myocarditis.
D. Myocardial ischemia.
E. Arrhythmogenic right ventricular cardiomyopathy.

The rhythm strip shows ventricular fibrillation (VF). The most common cause of VF and polymorphic
ventricular tachycardia is acute myocardial ischemia. The patient presented with chest pain due to
an acute coronary syndrome. His ongoing chest pain with a normal ECG can be seen with an acute
circumflex artery occlusion. He has multiple risk factors for coronary disease including tobacco use,
hypertension, and family history of sudden cardiac death (presumably due to a myocardial
infarction). Acute myocarditis, long QT syndrome, arrhythmogenic right ventricular cardiomyopathy
(ARVC), and Brugada syndrome can cause VF but are less compatible with his clinical presentation.

85. The correct answer: D


Key Point
Ventricular arrhythmia may be either monomorphic with a single QRS morphology or polymorphic
with changing QRS morphology. The most common cause of polymorphic ventricular tachycardia
(PVT) is acute myocardial ischemia, which may quickly progress to ventricular fibrillation (VF). Less
commonly, PVT is caused by circumstances that prolong the Q-T interval (drugs and ion-channel
disorders). It is important to recognize which common drugs are associated with Q-T prolongation
and discontinue.

Question 86
86.A 33-year-old male with no significant past medical history was admitted to the hospital with three
days of fevers and chills. His current medication regimen includes multivitamins. His family history
was significant for hypertension. On admission, he was febrile with a temperature of 39.0°C and
blood pressure of 100/60 mm Hg. His physical examination revealed diminished breath sounds in
left lower lung fields and was other wise unremarkable. A chest radiograph confirmed the diagnosis
of community-acquired pneumonia. A previous outpatient electrocardiogram (ECG) was available
(Figure 1); his admission ECG was also recorded (Figure 2).

What is the most appropriate next step in his management?

KKUH
Collected by:
Dr Hani Abdullah
Dr Fatma Hadi
Dr Salem Boresa
Dr Rami Elesali
Dr Naeif Almagal
1/1/2020
116

(Figure 1)

(Figure 2)

A. Aggressively treat fever.


B. Genetic testing.
C. Implantable cardioverter defibrillator.
D. Provocative testing.
E. Stress echocardiography.

KKUH
Collected by:
Dr Hani Abdullah
Dr Fatma Hadi
Dr Salem Boresa
Dr Rami Elesali
Dr Naeif Almagal
1/1/2020
117

Brugada syndrome (BrS) is an example of a channelopathy, a disease caused by alteration in the


transmembrane ion currents that form the action potential. Patients with BrS are prone to developing
ventricular tachyarrythmias that may lead to syncope, cardiac arrest, or sudden cardiac death
(SCD). Diagnosis of BrS requires the characteristic ECG pattern along with clinical history such
as unexplained syncope or family history of premature unexplained sudden
death. Febrile illnesses, alcohol or cocaine use, and certain medications may unmask a Brugada
pattern on ECG. The most important intervention at this point is to treat the fever.

Several pathogenic genes have been identified as associated with the disease, but SCN5A is the
most prevalent, present in nearly 30% of cases.1 Mutation-specific genetic testing for this patient is
not recommended, as he has a diagnostic type I Brugada pattern on his ECG but does not have a
history of unexplained syncope or family history of premature unexplained sudden death, and thus
does not meet criteria for BrS. In patients who are asymptomatic or with nondiagnostic ECGs, it
would be reasonable to proceed with intravenous administration of Na+ channel blocking drugs like
ajmaline or flecainide to elicit a type I Brugada pattern. An implantable cardioverter-defibrillator
(ICD) is not indicated in asymptomatic BrS patients with a drug-induced type I ECG pattern or on the
basis of family history of SCD alone.2 Stress testing would not add diagnostic information in this
setting.

86. The correct answer: A


Key Point
Asymptomatic BrS patients without a spontaneous type I pattern (or type 2 or 3 pattern) are at low
risk, and an ICD is NOT indicated, even if an Na channel-blocking drug evokes a type I pattern.

Question 87
87. A 22-year-old man with surgically repaired tetralogy of Fallot presents for evaluation of
palpitations for the past few months with heart rates of 120-130 bpm lasting a few hours at a time.
He notes that his exercise capacity has worsened as well.

He wore a cardiac event monitor prior to this visit, which demonstrated a 2-hour episode of atrial
flutter with variable atrioventricular (AV) conduction, correlating with his symptoms.

His cardiac examination today reveals a regular rate and rhythm, no gallop, and a grade 2/6 systolic
ejection murmur. Electrocardiogram demonstrates sinus rhythm with a first-degree AV block and
right bundle branch block with QRS 140 msec.

Which of the following is the most appropriate next step?


A. Transthoracic echocardiogram.
B. Sotalol 120 mg twice daily.
C. Radiofrequency catheter ablation.
D. Metoprolol 25 mg twice daily.
E. Diagnostic cardiac catheterization.

KKUH
Collected by:
Dr Hani Abdullah
Dr Fatma Hadi
Dr Salem Boresa
Dr Rami Elesali
Dr Naeif Almagal
1/1/2020
118

While treatment with medications or ablation may eventually be appropriate in this patient, the most
important first step is to reassess his structural and hemodynamic status. The development of atrial
tachycardia, atrial flutter, or atrial fibrillation in adult congenital heart disease patients is often
associated with progressive hemodynamic deterioration of the underlying disease. Thus,
echocardiogram is warranted. Surgical treatment of the hemodynamic problems does not eliminate
atrial arrhythmias, and ablation of atrial arrhythmias alone could allow significant hemodynamic
issues to progress and potentially deteriorate. Successful treatment involves assessing both the
arrhythmia and the contributing hemodynamic changes and addressing both when indicated and
feasible. Cardiac catheterization may be appropriate to perform more detailed hemodynamic
assessment, but in light of his unchanged physical examination, echocardiography will provide
important information without the need for an invasive procedure.

Early experience in adults with unoperated atrial septal defects and atrial arrhythmias demonstrated
the importance of an integrated approach for arrhythmia and hemodynamic problems; similar
principles apply to patients with tetralogy of Fallot, Ebstein’s anomaly, and single-ventricle
physiology; these patients are at highest risk of arrhythmia development, with concurrent
hemodynamic abnormalities. Beta-blockers may decrease catecholamine-related triggers and
provide AV nodal blockade during recurrent atrial arrhythmias. One study of adults with transposition
of the great arteries and prior atrial switch repairs with implanted defibrillators demonstrated
supraventricular tachycardia preceding ventricular tachycardia in 50% of patients; use of beta-
blocker medications in this population was associated with decreased incidence of appropriate
defibrillator shocks.

Observational studies on the use of sotalol in ACHD patients report freedom from recurrent atrial
tachycardia in 41-46% of patients during short-term follow-up. Use of either medication in the setting
of significant sinus node dysfunction may exacerbate bradycardia and requires careful monitoring.
Initiation of sotalol in this population is recommended during inpatient monitoring for proarrhythmia
for 48-72 hours. Multiple observational and multicenter studies have demonstrated acute success
rates of 65-100% for treatment of supraventricular tachycardia associated with ACHD. Acute
success rates vary by tachycardia mechanism and type of congenital heart disease and repair.

87. The correct answer: A


Key Point
Non–isthmus-dependent atrial flutter involves macro–re-entrant circuits elsewhere in the RA or LA
and may occur in a variety of clinical settings, including congenital heart disease, after cardiac
surgery, and after catheter ablation of AF.

Question 88
88. A 60-year-old woman came to the emergency department with palpitations. She described
irregular and racing heart beats as well as exertional shortness of breath starting the prior day . She
has also had 1-2 weeks of a tremulous and jittery feeling, sweating, and difficulty sleeping. Her past
medical history includes hypertension, asthma, and depression with anxiety.
KKUH
Collected by:
Dr Hani Abdullah
Dr Fatma Hadi
Dr Salem Boresa
Dr Rami Elesali
Dr Naeif Almagal
1/1/2020
119

Her current medications are lisinopril 10 mg, hydrochlorothiazide 25 mg, and extended-release
venlafaxine 150 mg. Her vital signs were pulse 115 bpm, respiration 15 breaths per minute, and
blood pressure 125/62 mm Hg. Her physical examination revealed an anxious-appearing female.
The lung fields showed mild expiratory wheezes and her heart rate was rapid and irregular without
murmurs or rubs.

The electrocardiogram is shown (Figure 1). A bedside ultrasound showed hyperdynamic left
ventricular systolic function and a trivial pericardial effusion. Laboratories included potassium 4.1
mEq/L, creatinine 0.9 mg/dL, thyroid-stimulating hormone 0.1 mIU/L (normal 0.5-5.0 mIU/L), and
free thyroxine 19.8 ng/dL (normal 0.7-1.9 ng/dL).

Which of the following would be the most appropriate for restoring sinus rhythm in this patient?

(Figure

A. Amiodarone.
B. Colchicine.
C. Disopyramide.
D. Propylthiouracil.
E. Propranolol.

The correct answer is treatment of hyperthyroidism with either propylthiouracil or methimazole. Atrial
fibrillation is a common complication of hyperthyroidism, occurring in 5-15% of cases. Antiarrhythmic
drugs, including disopyramide and amiodarone, and electrical cardioversion are unlikely to be
successful in hyperthyroid states and normalization of the thyroid function is usually associated with
spontaneous return of normal sinus rhythm. Additionally, amiodarone must be used cautiously in

KKUH
Collected by:
Dr Hani Abdullah
Dr Fatma Hadi
Dr Salem Boresa
Dr Rami Elesali
Dr Naeif Almagal
1/1/2020
120

patients with present or prior thyroid disease because of its iodine component. Beta-blockers are
first-line agents for rate control in atrial fibrillation associated with hyperthyroidism, but would not be
the best choice in this patient with active wheezing. Colchicine is effective for treating pericarditis,
which can also trigger atrial fibrillation, but this patient does not have the triad of pleuritic or
positional chest pain, ST elevations, and pericardial friction rub to support a diagnosis of pericarditis.
The 2014 American College of Cardiology/American Heart Association/Heart Rhythm Society
(ACC/AHA/HRS) Guideline for the Management of Patients With Atrial Fibrillation adds that "many
potentially 'reversible' causes of AF have been reported, including binge drinking, cardiothoracic and
noncardiac surgery, myocardial infarction (MI), pericarditis, myocarditis, hyperthyroidism,
electrocution, pneumonia, and pulmonary embolism. It is important to recognize that there are few
data to support the notion that patients with AF that occurs in the setting of one of these potentially
'reversible' conditions are, in fact, cured of AF after effective treatment or elimination of the condition.
Since long-term follow-up data are not available in these clinical scenarios and AF may recur, these
patients should receive careful follow-up."

88. The correct answer: D


Key Point
Wherever possible, reversible causes for AF should be sought and corrected. These include:
hyperthyroid state, pulmonary embolism, and pericarditis.

Question 89
89. A 50-year-old male underwent routine appendectomy. Shortly following extubation in the
recovery room, he was briefly unresponsive and a tracing (Figure 1) was retrieved from the monitor.
He presently was sleepy but responsive, acknowledging abdominal pain. A review of preoperative
evaluation showed a normal stress electrocardiogram 6 months before and no prior history of heart
disease. He received a dose of fentanyl 30 minutes ago. The last serum potassium was 3.6 mEq/dl.

Which of the following should be done prior to hospital discharge?

KKUH
Collected by:
Dr Hani Abdullah
Dr Fatma Hadi
Dr Salem Boresa
Dr Rami Elesali
Dr Naeif Almagal
1/1/2020
121

A. Catheter ablation.
B. Implantable cardioverter-defibrillator placement.
C. Chronic amiodarone therapy.
D. Coronary angiography.
E. No additional evaluation is required.

The tracing shows sinus rhythm for five beats, followed by a premature ventricular contraction that
initiates polymorphic ventricular tachycardia (VT) degenerating to ventricular fibrillation (VF) (Figure
1). Marching the QRS complexes forward into the arrhythmia does not suggest artifact and the
patient was briefly unresponsive, consistent with an episode of hypotension, with spontaneous
termination of the arrhythmia, which can occur with polymorphic VT and even VF. Polymorphic VT
due to QT prolongation is a consideration, but the QT interval is not prolonged. Furthermore, the
episode is not preceded by a pause or transient slowing of heart rate, as is usually seen with
acquired long QT syndrome. The most likely cause is myocardial ischemia, which may be transient
and needs to be evaluated. Thus, coronary angiography is the correct answer and should be
immediately considered, taking into consideration risks imposed by the recent surgery and any
additional evidence suggesting an acute coronary syndrome. VF due to an acute ischemic syndrome
usually does not warrant placement of an implantable cardioverter-defibrillator. Catheter ablation is
rarely performed for polymorphic VT, and then only for recurrent episodes with premature ventricular
contraction triggers that are not due to acute ischemia. Although acute intravenous amiodarone
therapy could be considered, chronic oral therapy should not be initiated at this time.

89. The correct answer: D


Key Point
Ventricular arrhythmia may be either monomorphic with a single QRS morphology or polymorphic
with changing QRS morphology. The most common cause of polymorphic ventricular tachycardia
(PVT) is acute myocardial ischemia, which may quickly progress to ventricular fibrillation (VF). Less
commonly, PVT is caused by circumstances that prolong the Q-T interval (drugs and ion-channel
disorders). It is important to recognize which common drugs are associated with Q-T prolongation
and discontinue.

Question 90
90. A 26-year-old female with no history presented to your office for evaluation of palpitations. For
the past year, she has had episodes of a racing heart lasting ≤30 minutes, most often at night,
without clear triggers. The episodes seem to start and stop abruptly and are associated with
lightheadedness. She does not drink caffeinated beverages or alcohol. She denied use of illicit
substances, vitamins, or supplements.

On examination, she was a healthy young female in no distress. Her heart rate was 68 bpm and
blood pressure 110/60 mm Hg. Her examination was unremarkable.
She had an episode in your office, which you captured on electrocardiogram (ECG) (Figure 1).

KKUH
Collected by:
Dr Hani Abdullah
Dr Fatma Hadi
Dr Salem Boresa
Dr Rami Elesali
Dr Naeif Almagal
1/1/2020
122

What do you recommend to this patient?

(Figure 1)

A. Catheter ablation.
B. Digoxin 0.125 mg daily.
C. Cardioversion.
D. Education on vagal maneuvers.
E. Metoprolol succinate 25 mg daily.

The ECG (Figure 1) shows atrial tachycardia. Atrial tachycardia originates from a discrete atrial
origin discharging at a rate that is generally regular at 100-250 bpm. Catheter ablation is preferred
over medical therapy, as it is more effective (Figure 2). Thus, ablation is the correct answer.

Metoprolol would not be preferred over catheter ablation. Digoxin is generally ineffective for atrial
tachyarrhythmias and has not been studied in focal atrial tachycardia.

Vagal maneuvers are not indicated, as this is not vasovagal syncope.

Cardioversion is not indicated, as the arrhythmia is paroxysmal and spontaneously converts to sinus
rhythm with brief episodes.

KKUH
Collected by:
Dr Hani Abdullah
Dr Fatma Hadi
Dr Salem Boresa
Dr Rami Elesali
Dr Naeif Almagal
1/1/2020
123

(Figure 2)
Reproduced with permission from Page RL, Joglar JA, Caldwell MA, et al. 2015 ACC/AHA/HRS
guideline for the management of adult patients with supraventricular tachycardia: a report of the
American College of Cardiology/American Heart Association Task Force on Clinical Practice
Guidelines and the Heart Rhythm Society. J Am Coll Cardiol 2016;67:e27-e115.

90. The correct answer: A


Key Point
Catheter ablation is the preferred treatment option for recurrent symptomatic PSVT – success rates
exceed 95% in good centers. Drug therapy (prophylaxis or as the occasion arises with drugs that
block AV node or antiarrhythmic drugs) is rarely effective or helpful.

Question 91
91. A 76-year-old woman has permanent atrial fibrillation. She has a history of hypertension,
diabetes, dyslipidemia, and osteoarthritis. Her medications include apixaban 5 mg twice daily,
metoprolol succinate 25 mg twice daily, metformin 500 mg twice daily, and atorvastatin 40 mg daily.

She would like to stop anticoagulation because she bruises easily.

What is this patient's estimated annual stroke risk without anticoagulation?

KKUH
Collected by:
Dr Hani Abdullah
Dr Fatma Hadi
Dr Salem Boresa
Dr Rami Elesali
Dr Naeif Almagal
1/1/2020
124

A. 21%.
B. 7%.
C. 1%.
D. 15%.
E. 3%.

There is a class I indication for using the CHA2DS2-VASc score to quantify annual stroke risk in
patients with non-valvular atrial fibrillation. This patient has a CHA2DS2-VASc score of 5 (Figure 1),
which corresponds to an annual stroke risk of 6.7%. Thus, 7% is the best answer.

(Figure 1)
*These adjusted stroke rates are based on data for hospitalized patients with AF and were published
in 2001. Because stroke rates are decreasing, actual stroke rates in contemporary nonhospitalized
cohorts might vary from these estimates.

†Adjusted stroke rate scores are based on data from Lip and colleagues. Actual rates of stroke in
contemporary cohorts might vary from these estimates.

KKUH
Collected by:
Dr Hani Abdullah
Dr Fatma Hadi
Dr Salem Boresa
Dr Rami Elesali
Dr Naeif Almagal
1/1/2020
125

91. The correct answer: B


Key Point
The CHA2DS2-VASc and HAS-BLED score decisions are used to balance the need for long-term
anticoagulant therapy against the risk of bleeding, respectively. These scoring systems are best
utilized in patients with nonvalvular AF, and do not apply to other special populations, such as those
with hypertrophic cardiomyopathy.

Question 92
92.A 40-year-old female was admitted to the hospital with palpitations, diaphoresis, and
lightheadedness. In the emergency department, she was hypotensive and tachycardic; an
electrocardiogram (ECG) was obtained (Figure 1). She was cardioverted to sinus rhythm with relief
of symptoms. Her evaluation included a normal basic metabolic panel, blood counts, and troponin.
Her echocardiogram was normal with normal chamber sizes and no valvular lesions. Her cardiac
magnetic resonance imaging (MRI) did not show any areas of delayed gadolinium enhancement or
structural abnormalities of the heart chambers.

Which of the following is the best next step in her care?

(Figure 1)

A. Propranolol.
B. Cardiac sympathetic denervation.
C. Implantable cardioverter-defibrillator.
D. Mexilitine.
E. Ablation of accessory pathway.

Sustained monomorphic ventricular tachycardia (VT) in the setting of a structurally normal heart
does not necessarily benefit from implantable cardioverter-defibrillator therapy, and preventive
therapy of the arrhythmia is most appropriate. To this end, beta-blockers, calcium channel blockers,
KKUH
Collected by:
Dr Hani Abdullah
Dr Fatma Hadi
Dr Salem Boresa
Dr Rami Elesali
Dr Naeif Almagal
1/1/2020
126

and antiarrhythmic medications as well as catheter ablation are considered first-line therapies
(Figure 2). This patient has right ventricular outflow tract (RVOT) VT, which can be treated with beta-
blockers, calcium channel blockers, Class Ic antiarrhythmic drugs, or catheter ablation. Cardiac
sympathetic denervation could be effective for catecholaminergic polymorphic ventricular
tachycardia or long QT syndrome, but not for this patient with RVOT VT. Mexilitine, a Class Ib
antiarrhythmic drug, is used for re-entrant or scar-mediated ventricular arrhythmias and would not be
expected to be effective in preventing RVOT VT. Catheter ablation of accessory pathway tracts is
used for Wolff-Parkinson-White syndrome, which is not the correct diagnosis here.

(Figure 2)
ICD = implantable cardioverter-defibrillator; SHD = structural heart disease; VT = ventricular
tachycardia.
92. The correct answer: A
Key Point
Distinction of ventricular tachycardia in patients with or without structural heart disease is
importantsince therapeutic approaches are very different. In patients with structural heart disease,
an implantable cardioverter-defibrillator is generally recommended (secondary prevention); in
patients without structural heart disease, drug therapy or catheter ablation may be effective in
reducing or eliminating the arrhythmia.

KKUH
Collected by:
Dr Hani Abdullah
Dr Fatma Hadi
Dr Salem Boresa
Dr Rami Elesali
Dr Naeif Almagal
1/1/2020
127

Question 93
93. A 21-year-old male college hockey player was referred for evluation of intermittent palpitations
and a recent episode of near syncope during practice. He reported infrequent episodes of
palpitations dating back ≥10 years. However, for the past couple of years, he has noted that his
symptoms have been more frequent and occur at least once a month. He describes a "rapid and
strong beating in my chest," which is occasionally associated with lightheadedness and presyncope,
although he has never lost consciousness. His symptoms last only "a couple of minutes" and
generally resolve with Valsalva. He has not been able to participate in athletics since his most recent
episode of near syncope and is anxious to return as soon as is safely possible.

He has no prior relevant medical history and takes no medications.

An electrocardiogram (ECG) was obtained (Figure 1).

What is the most appropriate next step?

(Figure 1)

A. Prescribe verapamil.
B. Order a 48-hour Holter monitor.
C. No further intervention or therapy.
D. Referral for catheter ablation.
E. Referral for treadmill stress test.

KKUH
Collected by:
Dr Hani Abdullah
Dr Fatma Hadi
Dr Salem Boresa
Dr Rami Elesali
Dr Naeif Almagal
1/1/2020
128

Based on the patient's ECG, which demonstrates pre-excitation, and his symptoms, he is considered
to have Wolff-Parkinson-White (WPW) syndrome. Catheter ablation is recommended as first-line
therapy for therapy for symptomatic patients. Multiple large studies have demonstrated the safety
and efficacy of this approach. The success rate for catheter-ablation is approximately 90-95% but
varies based on the location of the pathway and the presence of multiple pathways.

Noninvasive (exercise stress test, Holter monitor, procainamide infusion) or invasive


(electrophysiology study) risk stratification techniques can be useful in asymptomatic patients with
WPW pattern on ECG. However, the patient described in this case is symptomatic and has WPW
syndrome. Therefore, further risk stratification is not indicated in this case.

If he were not interested in proceeding with ablation, medical therapies could be considered. In the
absence of structural heart disease, one could consider flecanide or propafenone (Class IIa).
Amiodarone, beta-blockers, diltiazem, verapamil, sotalol, and dofetilide are also options, although
these medications receive a class IIb recommendation in the 2015 American College of
Cardiology/American Heart Association/Heart Rhythm Society (ACC/AHA/HRS) Guideline for the
Management of Adult Patients With Supraventricular Tachycardia (Figure 2).

(Figure 2)
AVRT = atrioventricular re-entry tachycardia; ECG = electrocardiogram; pt = patient; SHD =
structural heart disease.
93. The correct answer: D
Key Point
Catheter ablation is the preferred treatment option for recurrent symptomatic PSVT – success rates
exceed 95% in good centers. Drug therapy (prophylaxis or as the occasion arises with drugs that
block AV node or antiarrhythmic drugs) is rarely effective or helpful.

KKUH
Collected by:
Dr Hani Abdullah
Dr Fatma Hadi
Dr Salem Boresa
Dr Rami Elesali
Dr Naeif Almagal
1/1/2020
129

Question 94
94. A 64-year-old female presented to the cardiology clinic for evaluation of exertional dyspnea and
fatigue. She denied any syncope but has had a few episodes of sudden lightheadedness. Her
medical history includes hypertension and diabetes mellitus, both of which are well controlled. Her
medications include aspirin, amlodipine, hydrochlorothiazide, metformin, and atorvastatin. On
examination, her heart rate (HR) was 66 bpm, blood pressure was 127/82 mm Hg, and body mass
index (BMI) was 34 kg/m2. She was referred for cardiopulmonary exercise stress testing. She was
able to complete 5 minutes of a Bruce protocol with a baseline HR of 64 bpm and maximum HR of
98 bpm. She reached her anaerobic threshold. The test was terminated due to patient fatigue. She
denied any anginal symptoms and there were no ischemic electrocardiogram changes.

Which of the following is most likely responsible for her symptoms?


A. Deconditioning.
B. Sinus node dysfunction.
C. Baroreceptor response.
D. Obesity.
E. Medication side effect.

This patient's cardiopulmonary exercise testing (CPET) was notable for a low maximum achieved
HR (63% maximum predicted HR) in the setting of meeting anaerobic threshold, consistent with
chronotropic incompetence, which is one manifestation of sinus node dysfunction. The most
commonly accepted definition of chronotropic incompetence includes a failure to reach 80%
maximum predicted HR with exercise. Sinus node dysfunction may also present with sinus pauses,
which may be responsible for the reported history of presyncope. Obesity, deconditioning, and
baroreceptor response would not be expected to cause inadequate HR response to exercise.
Whereas beta-blockers and nondihydropyridine calcium channel blockers may blunt HR response,
amlodipine should not have this effect.

94. The correct answer: B


Key Point
SND comprises a number of different manifestations including symptomatic bradycardia, sinus
pauses due to sinus arrest or sinoatrial exit block, and chronotropic incompetence.

KKUH
Collected by:
Dr Hani Abdullah
Dr Fatma Hadi
Dr Salem Boresa
Dr Rami Elesali
Dr Naeif Almagal
1/1/2020

You might also like